APPENDIX.

EXAMINATION PAPEES.

ISTO-lSTl. EXAMINATIONS

THE JULY TERM, 1870.

MATRICULATION EXAMINATION.

GEEEZ.

Professor Irving.

SMITH, Smaller Greek Grammar. XENOPHON, Anabasis, Books IV. V. VI.

\_N.B.—The principal parts of a verb are the first persons singular indicative of its present future and perfect active, ofits perfect passive and second aorist active ; or if these be not in use, then of the present future perfect and aorist employed. In parsing a verb give its tense mood voice and prin­ cipal parts. In parsing a substantive or an adjective give its gender number case and nomina­ tive and genitive singular. No abbreviation of any Greek word is to be used. You need not accentuate.']

1. Translate literally, omitting no word and indicating by brackets all that you supply— Ma^OjUEi'wu ce avrdy rat inrooov^iiviov, deoiv nc airoTc /.trixaii/v cruirtipiag cioioaiv. 'EJaJriVijc yap uytka/iiptv olicia TUV kv Se^iy, orov. 01) tvai^avroQ. IV EXAMINATION PAPERS,

'Sic, o' av-r] £,vvirrnrTev, itpevyoi' 01 airb rwc ev Ce^itf vhtuiv. 'flc 3' tfiadtv 6 Seroevyov ovv cat nl rWo rouriiii' rail' OIKIWV. Oi 0£ (.aro arofia. Oi) 'in fjtovoi tXvTTOvv Kal cijXoi ijirar, Jn iitiKuaovTai iv TT) c£uou .re Kal Karafiaaei. 'EiraiOu TapayytWet (popeiv i,i)Xa, oaoi krvyytivov eloi SITES riSy fitXojy, elg rd jxiaov lavriiiiv rat TWV iroXt/iibiv.

2. In the foregoing point out (a) a Genitive Absolute, (6) an Indefinite Pronoun, (c) a Neuter Adjective used for an adverb, {d) an Historic Present. Why is (1) lti,iq Feminine Singular, (S) piXoiv Genitive, (3) ai)-oTc Dative, avikafi\j/tv Aorist and not imperfect ?

3. Decline throughout the substantives Uvapaau-, VOOQ, iropda, v\a.^; the adjective TTOXVC ; the pronouns oS-oc and rig, and the participle Odg. {Four at least must be done without mistake.)

4. Write down these tenses throughout—Second Aorist Indicative Middle of cuptKviofiai, First Aorist Indi­ cative Active of oWw, Imperfect Indicative Active of Jjut (I go), First Aorist Imperative Middle of

5. Write down the First Person Plural of these tenses —Second Aorist Indicative Active of ip^o/iai, Im­ perfect Indicative Middle (contracted) of ^opt'oi, Future Indicative Active of pivu, Second Aorist MATRICULATION, J.T. 1870. V

Optative Active of tf.ifla.tya>, First Aorist Sub­ junctive Active of arifiaivto, Future Optative .Middle of Xafjflayut; and the principal parts of alaOayo/jui, flaXXut, heLKWfii, hvva/iai, QUID, Xdirw, Ttavw, inrivtw, rpiflut, tptliyoj. ( Three of the former and five of the latter must be done ?vithout mistake.)

G. Give the meaning and the derivation of airetpog, (Kppoavyrj, flaSqy, Kevoraipiov, Ki'Tjfilc, fiovapyna, va- parao-ceiv, aKr\voo>, aKvrtvog, -)/i.Tti>vlaKOQ.

7, Parse BeSovXevKtvat, c£ayayi>v, ijfiiav, tTnrear;, KC^W- piafiivovc, jXtyaXv, tzaytcn, Traptt^ov, aw^rjcdc, rd- XOvq.

8. Translate the following, and give the rule for all cases, other than the nominative, occurring in them—

(1) eyo> Eifu} o cKptXoficvoc. AtijtTTTrou ayovrog TOVTUV TOV avion. (2) ru>v 'ApKaOwv ol fiev TtBydatv, ol Si Xonrol iirl Xoipov ni'oc TroXioprauJTai. (3) tv-^ofxai Covvai fioi rove Otovc. a'inoy TIVOQ vfiiv ayaOov yu'tadat. (4) evdvg ayaKaXEtraficvot rjj o-d\;rtyy( aTfijEuav, Kal (KpiKovro abdrifitpby iirl ru arpaTomCov. VI EXAMINATION PAPERS,

LATIN. Professor Irving. SMITH, Smaller Latin Grammar. CESAR, Gallic War, III. IV. \_N.B.—In parsing a verb give tense, mood, voice, and principal parts, viz., the first person singular present and perfect indicative {active if in use), the supine in -um, and the present infinitive {active if in use). In parsing a substantive or an adjec­ tive give gender, number, case, and nominative and genitive singular. No abbreviation of any Latin word is to be used.'] 1. Decline throughout the substantives dies, filius, mare, pes; the adjectives acer, prudens ; the nu­ meral duo; and the pronoun quis. {Foitr at least must be done without mistake.)

2. Write down throughout Past-Perf. Ind. Act. of cognosco, Fut. Subj. Act. of edo, Imper. Act. of eo, Fut. Perf. Ind. Pass, of recipio,-Perf. Subj. Act. of sto, Past-Imp. Subj. Act. of volo. {Four at least must be done without mistake.)

3. Write down the principal parts of audeo, claudo, evado, exuo, moveo, pello, peto, sustineo, iitor, venio; and the second person singular of these tenses—Past-Perf. Subj. Act. of cado, Fut. Imp. Act. of do, Perf. Ind. Act. of eripio, Past-Imp. Subj. Puss, of fero, Fut. Ind. Pass, of pono, Perf. Ind. of soleo. {Six of the former and four of the latter must be done without mistake.) MATRICULATION, J.T. 1870. Vli

4. Parse these nouns—acie, ccdificiis, arborum, fibulis, plurimos, vici; and these verbs—administrandi, daretur, impensus, incolerent, rescidit, ulcisceretur. (Six at least must be parsed according to instruc­ tions without mistake.) 5. Translate literally— Hacconfirmataopinione timoris, idoneum quon­ dam hominem et callidum delegit, Galium, ex his, quos auxilii causa secum habebat. Huic magnis praemiis pollicitationibusque persuadet, uti ad hostes transeat, et quid fieri vclit edocet. Qui ubi pro perfuga ad eos venit, timorem llomanorum proponit, quibus angustiis ipse Cajsar a Venetis prematur docet, neque longius abesse, quin proxima nocte Sabinus clam ex castris exercitum educat, et ad CtBsarem auxilii ferendi causa proficiscatur. Quod ubi auditum est, conclamant omnes occa- sionem negotii bene gerendi amittendam non esse; ad castra iri oportere. Multiu res ad hoc consilium Gallos hortabantur: supeiiorum dierum Sabini cunctatio, perfuga? confirmatio, inopia cibariorum, cni rei parum diligenter ab his erat provisum, spes Vcnetici belli, et quod fere libentcr homines id, quod volunt, credunt. 6. Point out in the foregoing {a) an impersonal verb, {b) a passive used impersonally, (c) an ablative of time, {d) a dative of advantage, {e) a subjunctive of result, and {f) a relative pronoun referring to a sentence. 7. Translate the following, and give the rule for every case other than the nominative occurring in them— (1) Equitatu quo plurimum valebant agmen adorti sunt. Vlll EXAMINATION PAPERS,

(2) A quibus quum paucorum dierum iter abesset legati venerunt. (3) Omnia deerant qure ad reficiendas naves usui erant. (4) Tn quibus aliquid consilii aut dignitatis fuit eo pervenerant. 8. Give the meaning of each of the following, the word from which each is derived and its meaning, the termination by which each is formed and its mean­ ing—amicitia, civitas, frumcntum, graviter, mer- cator, pedester, proximus, vexo. 9. (a) What are the Latin Demonstrative Pronouns and their peculiar meanings ? Construct an ex­ ample of the use of each. {b) Name some of the verbs which in Latin are followed by an Accusative and Infinitive. 10. Give the Latin for {a) Cajsar is said to have been of tall stature, {b) How many books can you buy for eighteen pence (use as for a penny) ? {c) The speech of the general was more true than pleasant to his soldiers, {d) The senate sent ambassadors to Ctesar, to ask what he was going to do next year.

ENGLISH. Professor Irving. 1. Write out, correctly spelt, the following:— The armie wh: now bekaim supream in the stait wos difrent from eny that haz sins bene scan MATRICULATION, J.T. 1870. IX

among us. At presunt the pay of the comun soljer iz not sutch az can sedyoos anny butt the umblest clas of Inglish laybrers from there cawl- ing. A barryer allmost impassible seperaits him from the comishuned offiscr. The grate majorities of thoze hoo rize hie in the servise rise bye per- chace. So iioomrus and ecstensiv ar the remoat dependensies of Ingland that evry man who enlists in the h'ne must ecspekt to pas meny yeres in oxsile and sum in klymaits unfaivrable two thee helth and viggre of the Europeean rase. 2. Parse every word in the following according to Morell's scheme:—Indeed, the Parliament itself abhorred its old defenders more than its old enemies. 3. Give the general and the detailed analysis of the following :—The private, distinguished for intel­ ligence, might hope to attain high command. 4. Correct the following, and explain the mistake in each:— (1) Nor heaven nor earth have been at peace to­ night. (2) Every one, both masters and men, know this to be false. (3) This tax has no right to be levied. (4) This paper has the largest circulation of any in the world. 5. State briefly the meaning of the following Eng­ lish words, all of which occur in one page of Macaulay's History: — Inextricable, unrepealed, bigoted, synod, permanent, penury, conscientious, assemblv. a3 X EXAMINATION PAPERS,

6. Give from Morell the meaning of all prefixes and suffixes occurring in the foregoing, and name the source whence each is derived.

7. Write down the first, the second, and the third persons singular of the present and of the past in­ dicative of these verbs—To be, can, to go, to lie, shall, to transfer.

8. State, according to Morell's system, the exact re­ lation which the sentence in italics bears to the other sentence in each of the following examples : (1) He knew not whither to go. (2) //' you go away, I shall not remain. (3) Wliere the bee sucks, there lurk I. (4) Man proposes, but God disjioses.

9. Give, with examples, the common modes by which plural nouns are in English formed from their singulars.

10. Write a brief but full description of an orange.

11. Explain the distinction drawn by Crombie be­ tween primary, primitive ; near, nearly ; human, humane; universal, general; proposition, pro­ posal; genii, geniuses.

12. "Derivatives are generally construed like their primitives." Explain and illustrate this rule of Crombie's. MATRICULATION, J.T. 1870. XI

PEENCH.

Professor Irving.

\fUhe first five questions must be answered to the satis­ faction of the Examiner.']

1. Write down throughout the present indicative of s'etcndre, the present subjunctive of avoir, the imperfect subjunctive of produire, the present in­ dicative of soutenir, the future of changer, the imperfect subjunctive of etre. {Four of these must be done without mistake.)

2. Write down the 1st person singular of the present of the imperfect and of tho preterite definite (all indicative), and of the present subjunctive of the verbs absoudre, conquerir, ecrire, fairo, mettre, nattre, pouvoir, prendre, voir, vouloir; and the 3rd singular present indicative of appeler, the 3rd singular imperfect subjunctive of dormir, the 3rd plural present indicative of s'asseoir, the 2nd plural preterite definite of poindre and of taire, and the 3rd plural imperative of suivre. {Five of the former and three of the latter must he done without mistake.)

3. Translate literally— Le roi Auguste, bien aise de punir 1'eveque do Posnanie avec bienseancc, et de plaire h le cour de Rome, contre laquelle il se serait eleve en tout autre temps, remit le prelat polonais entre les mains du nonce. L'evCque, apres avoir vu pillor sa maison, fut porte par des soldats chez le Xll EXAMINATION PAPERS,

ministre italien, et envoy6 en Saxe, ou il mourut. Le comte de Hoorn essuya dans le chftteau oil il etait renferme le feu continuel des ennemis: enfin, la place n'etant pas tenable, il se rendit prisonnier de guerre avec ses quioze cents SueVlois. Ce fut lu le preuiier avantage qu'eut le roi Auguste, dans le torrent de sa mauvaise fortune, centre les amies victorieuses de son ennemi.

4. Put into French— {a) How long has your father been in Paris ? {b) Did your friend speak of me in his last letter ? (c) Yes, and in all the letters which I have re­ ceived, he sends you his compliments ? {d) We must not believe all that we hear, {e) Fetch me pen, ink, and paper. {/) You are wrong in thinking that vou want a hor.=e.

5. Give the French for a battle, his face, the garrison, his wisdom, difficult, equally, hardened, regular, to become, to lie down, to seek, to try; and the English of accable, se brouiller, s'ecarter, cve'que, fevrier, menuisier, sechement, souffler, les vagues, veiller.

6. Translate— Cependaut cette armee suedoise, sortie de la Saxe si triomphante, n'ctait plus; la moitie avait peri de misure, 1'autre moitie etait esclave ou massacree. Charles XII. avait perdu en un jour le fruit de neuf ans de travaux et de prts de cent combats : il f'uyait dans une mechanic caleche, ayant i\ son cote le major general Hord, blesse dangereusement; le reste de sa troupe suivait, les uns a pied, les. autres h cheval, quelques-uns Xlll

dans des charrettes, h travers un desert oil ils ne voyaient ni huttes, ni tentes, ni hommes, ni animaux, ni chemins; tout y manquait, jusqu'a, 1'eau memo. C'etait dans le commencement de juillet. Le pays est situe au quarante-septieme degr6; le sable aride du desert rendait la chaleur du soleil plus insupportable ; les chevaux tom- baient; les hommes etaient pres de mourir de soif. Un ruisseau d'eau bourbeuse fut 1'unique ressource qu'on trouva vers la nuit; on remplit des outres de cette eau, qui sauva la vie h la petite troupe du roi de Suede. Apres cinq jours de marche il se trouva stir le rivage de fleuve Hippanis, aujourd'hui nomme le Bogh par les barbares, qui ont.defigure jusqu'au nom de ces pa}'S, que des colonies grecques firent fleurir autrefois. Ce fleuve se joint a quelques milles de lit au Borysthenc, et tombe avec lui dans la mer Noire.

7. {a) Write down six French adjectives, masculine and feminine, to show the different ways of forming feminines. {b) Write down, with their meanings, the possessive pronouns used in French. (c) Write down, with their meanings, six adverbs of comparison used in French.

8. Put into French— (1) It is not the scarcity of money, but that of men and of talents that makes an empire weak. (2) They killed the Russians to get rid of those prisoners with whom they knew not what to do. XIV EXAMINATION PAPERS, -

(3) It was a triumph nearly such as that of the old Romans. (4) lie took a black wig to disguise himself, for he used always to wear his own hair.

ARITHMETIC.

Professor Wilson.

All working required in obtaining a, result must he sent in as part of the answer. Every result must be reduced to its simplest form. Wliere the an­ swers are not exact, they must be worked to three places of decimals. Five at least must be done correctly.

1. Write down in words the quotient and the remain­ der obtained by dividing Twenty-one thousand two hundred and thirty-one billions three hundred thousand nine hundred and thirty millions two hundred and ten thousand by Seven hundred thou­ sand seven hundred millions thirty thousand four hundred. In this case an integral quotient only is required. o A reservoir leaks uniformly "025 pints in every se­ cond. How many gallons will leak out between noon on the 1st January and midnight on the 31st December in the present year ? MATRICULATION, J.T. 1870. XV

3. A clock is set correctly at noon on the 19th of January in the present year. It is ten minutes fast at 7 a.m. on the 1st July. Supposing it to have gained uniformly, what is the gain in each twenty-four hours ?

4. Assuming the pressure of the atmosphere to be 14-75 lbs. on every square inch of surface, find the whole pressure on the surface of a cube whose edge is six feet.

5. Supposing the value of the copper coinage reduced so that £1 contained 1000 farthings, find the loss which would result in a year in the tolls of a bridge over which 1500 persons passed daily paying a halfpenny each.

6. A metre is 39-37079 inches. Express a mile in metres.

7. A litre contains 61-027 cubic inches, and a gallon contains 277-274 cubic inches. Find how many pints there are in a litre.

7 C_i. i •) in .f i) 8. Reduce -n-A y-, f ,\ \, to a decimal. " vs ~ TV — it (i + TV

n „ , -03 - -02997 + -00003 to a vul ar 9. Eeduce .00539 + -00048 - -00491 ° fraction in its lowest terms.

10. A square field contains 100 acres: find the length of one side. XVI EXAMINATION PAPERS,

ALGEBEA.

Professor Wilson.

All work required in answering a question must he sent in as part of the answer. Every result must be reduced to its simplest form. Hix at least must be done.

1. The numbers a and x are multiplied together, so also are b and y; the latter product is subtracted from the former, and the remainder divided by the sum of the two products; write down the alge­ braical expression for the result.

2. Twice the number a is multiplied three times in succession by the number b ; also the number b is multiplied three times in succession by twice the number a; the two products so obtained are mul­ tiplied together ; find the result.

3. The sides of a rectangular block of land are 2a and 26 chains: A-person starting from one angle to walk round the boundary proceeds for 3« — h chains; he then turns back and walks b — a chains; he then goes forward again 5b — a chains; how much further has he to go to reach his start­ ing point ?

4. Reduce to its simplest form 1 1 i a- + -x a x x a x a MATRICULATION, J.T. 1870. XVII

5. Multiply together 4.a-x — 8a3 and Qax {2a + x) + 8a"

6. Substitute Sax for y in the expression a* — x* + 2x-y — y« a8 — Sax + x"

7. Substitute 2a2x3 for y in the expression y* — Say3 + oa-y- — a3y

8. Substitute a- + b* for x, and «« — b- for y in the expression a* — 2aix + x- x" + 2xy + y2 b* + 2b2y + y1 ' x2 — 2xy + y2

9. A person starting from a certain place walks x miles northwards; then he turns round and walks half-way back to his starting point; he again walks x miles northwards, and then half-way back to his starting point; a third time he walks x miles northwards, and then half-way back to his starting point. Express his distance from his starting point after his third return towards it.

10. Solve the equation (* + 1) {x +^3) = {x + 2)2 ~{x-2)

11. Solve the equation a{x-b + l)-l-x + {l-a){b-l) XV111 EXAMINATION PAPERS,

EUCLID.

Professor Wilson.

Algebraical Symbols must not be used.

Book I.

Three of these must be written out. 1. If two triangles have two sides of the one equal to two sides of the other each to each and have like­ wise their bases equal the angle contained by the two sides of tho one shall be equal to the angle contained by the two sides equal to them of the other.

2. To draw a straight line at right angles to a given straight line from a given point in the same.

3. To make a triangle of which the sides sh;ill be equal to three given straight lines; but any two what­ ever of these must be greater than the third.

4. If a straight line falls upon two parallel straight lines it makes the alternate angles equal to one another and the exterior angle equal to the interior and opposite upon the same side and likewise the two interior angles upon the same side together equal to two right angles.

5. In any right-angled triangle the square which is described upon the side subtending the right angle is equal to the squares described upon the sides which contain the right angle. MATRICULATION, J.T. 1870. XJX

Book II.

Two of these must be written out.

If a straight line is bisected and produced to any point the rectangle contained by the whole line thus produced and the part of it produced to­ gether with the square of half the line bisected is equal to the square of the straight line which is made up of the half and the part produced.

If a straight line is divided into any two parts the squares of the whole line and of one of the parts are equal to twice the rectangle contained by the whole line and that part together with the square of the other part.

To divide a given straight line into two parts so that the rectangle contained by the whole and one of the parts shall be equal to the square of the other part.

HISTOEY.

Professor Hearn.

Candidates arc to select any two, but not more, of the subdivisions of each question.

I.—Describe the geographical position of each of the following places; and state, with their respective XX EXAMINATION PAPERS,

dates, the principal historical events connected with each place :— 1. Amphipolis: Mantinea: Potidaea. 2. Carrhae : Clupea : Syracuse. 3. Athelney: Dunkirk : Torres Vedras.

II.—Give, with the proper dates, some account of each of the following persons :— . 1. Aristotle: Callicratidas : Demetrius Po- liorcetes. 2. C. Flaminius : C. Marius : Tarquiuius Priscus. 3. Canute: Duke of Somerset (the Protector): Prince Eugene.

III.—Give, with the proper dates, some account of each of the following events :— 1. The Mutiny of Alexander's Army: The Battle of Crannon: The Return of the Heracleidre. 2. The Battle of Actium: The Formation of the First Triumvirate: The Restoration of the Achrean Exiles. 3. The Massacre of St. Brice's Day: The Black Hole of Calcutta : The Grand Remonstrance.

IV.—State the principal events in each of the following- years :— " 1. B.C. 501: 401: 301. 2. B.C. 321: 221: 121. 3. A.D. 1297: 1497: 1697. GEOGEAPHY.

Professor McCoy.

1. Trace the boundaries of the area in the Northern Hemisphere in which wheat, oats, and barley will flourish.

2. Trace the north and south boundaries of the belt of the earth in which parrots are found.

3. Describe in as much detail as you can the basin of the Amazon, with the names and courses of the principal tributaries.

4. Give as many of the mountain chains as you can, having an approximately east and west direction, in South America.

5. Give as many of the lakes and inland seas as you can of Asia.

6. What oceanic currents pass round the Cape of Good Hope ?

7. Explain the methods of Government in the Chinese

Empire and Japan respectively.

8. Draw a rough outline map of Africa.

9. Fill in on the above outline map the boundaries of the principal political divisions, by dotted lines, XXli EXAMINATION PAPERS, MATRIC. J.T. 1870. < indicating each by a number referring to a sepa­ rate list of the names. 10. Fill in on the above map die courses of the princi­ pal rivers, indicating each by a letter referring to a separate descriptive list. 11. Mark down the positions of the principal islands near the African seaboard. EXAMINATIONS

IN THE OCTOBER TERM, 1870.

MATRICULATION EXAMINATION.

GEEEK. Professor Irving.

SMITH, Smaller Greek Grammar. XENOPHON, Anabasis, Books IV. V. VI. \_N.B.—The principal parts of a verb are the first persons singular indicative of its present future and perfect active, of its perfect passive and second aorist active; or if these be not in use, then of the present future perfect and aorist employed. In parsing a verb give its tense mood voice and principal parts. In parsing a substan­ tive or an adjective give its gender number case and nominative and genitive singular. No ab­ breviation of any Greek word is to be used. You need not accentuate.']

1. Translate literally— 'AKOvaag ravra 6 KXiavBpoe Ettrtv on Ac'&Tnrov fiey OVK iwaiyoiri, ti ravra trcTrottjKioc t"ij]' ov jxivroi 'itpri yofiii^eiy, ohB' ei irafnrovrip'oc; cir] At'^tmroc, fliav XXIV

Xpiji'ai Ttaa-)(£iv avrov, aXXa Kptfttvra, dtrwcp Kal i)fieic vvv OXLOVTE, rijg 0IKJ]Q rvyeiv. Nvv JIEV OVV OLTTITC raraXt7ro>T£C rovrov TOV ixvcpa. brav o' iyi> KEXEVOU), TTCIpEOTE TTpOQ T>]V KplUlV. A'tTlWfXat BE OVTE rf]y errpariav OVTE aXXo>' ovBiva £rt, LTTEI yt OVTOQ aires o/xoXoyEl cupEXEirOai TOV at'Opa. 0 BE aqiatpE- 6EI£ E'IITEV Eyi), <5 KXtavopE, EI rai O'IEI [IE aCiKOvyra. TI (iyEcrOai, OVTE ETratoy ovciva OVTE iflaXXoy, dXX' elirov on Btifxoma ftr/ ra irpoflara' i\v yap TO>V urpa- Tiiorwv Boyfin, E'L Tie, U~6TE ?; crrpaTia E£WL, lola Xr/'i^oiTO, Bjjfioota Eiyat ra XrjajOh'Ta.

2. In the foregoing point out (a) a Temporal Parti­ ciple, {b) a Protasis containing an optative mood, (c) an Indirect Middle, {d) a Partitive Genitive governed by a verb. Why is (1) Iwaivolti optative, (2) KEXEVITO) aorist instead of present, (3) EZIOI optative, (4) tit) (third line from end) singular instead of plural ?

3. Decline throughout the substantives aXc BIKT] vboe (contracted forms only), the adjective ayadug, the definite article b, and the participle XE'^OIC. {Four must be done without mistake.)

4. Write down throughout these tenses 2nd Aor. Ind. Act. yiyywo-Kw, Imp. Ind. Act. BlouifjLi, Put. Ind. Act. KOfii£w, 1st Aor. Imp. Pass. Xt'yw, 1st Fut. Ind. Act. raaaos, Pres. Imp. Pass, rjyuaw (con­ tracted forms only). {Three must be done without mistake.)

5. Write down the 3rd person singular of 1st Aor. Ind. Act. a'tpoi, 2nd Aor. Opt. Act. 'lo-ropi, Pres. MATRICULATION, O.T. 1870. XXV

Opt. Act. bpaw (contracted form), 1st Aor. Opt. Mid. iravoj, 1st Aor. Imp. Act. TTXEKW, Perf. Imp. Pass, ratro-to; and the principal parts of otUvvfii, Kaiw, Xafxflavoi, yo/ii£w, TTOIEIO, (pEpto, (pfiyoi, (pvXatnro). {Three qftlie former and four of the latter must be done without mistake.)

0. Parse anoBiBpaKOTEg, Efjfltflw, ij)(6riTav, ZvXXEyijyai, ayyEtotg, trXoiov, vflpsi, vypoTrfra. {Four must be done correctly.)

7. Give the meaning and the full derivation of adpoi^w, alyiaXog, avSpairoBoy, CEiXog, Bvufjn'l, EVKXEI'IC, fitya- Xi)yopE~ty, TTEXraoTJjc.

8. Translate and give the rule for every case other than the nominative in (1) dig diriflijaav ol 'ApKil- Bcg rfjg WKTog (2) /liyovci BE oh fitag fjfiipag cirri TCI ETriTi'iSEia (3) KcpaaovyTtwy da-fyat avrovg Et)fj)0i;yuEi'.

9. («) How are Reflexive Pronouns made in Greek ? {b) Name two modes of comparison of adjectives. (c) Distinguish the meaning of 6 ayadog cwi'ip, and o ariip ayaOog. {d) What is the difference in meaning between the moods of the Aorist and those of the Present ? LATIN.

Professor Irving.

SMITH, Smaller Latin Grammar. CESAR, Gallic War, III. IV.

\_N.B.—In parsing a verb give tense, mood, voice, and principal parts, viz., th'e first person singular present and perfect indicative {active if in use), the supine in um, and the present infinitive {active if in use). In parsing a substantive or an adjec­ tive give gender, number, case, and nominative and genitive singular. No abbreviation of any Latin word is to be used.]

1. {a) Translate literally— Quibus ex navibus cum essent expositi milites circiter trecentr. atque in castra contendercnt, Morini, quos Ctttsar in Britanniam proficiscens pacatos reliquerat, spe prredEe adducti, primo non ita magno suorum numero circunistctemnt, ae, si sese interfici nollent, arma ponere jusscrunt. Cum illi orbe facto sese dcfenderent, celeriter ad cla- niorem hominum circiter millia sex convenei'unt. Qua re nuntiata, Cawar omnem ex castris equifa- tum suis auxilio misit. Interim nostri milites impetum hostium sustinuerunt, atque amplius hoi-is quattuor fortissime pugnaverunt, et, paucis vulneribus acceptis, complures ex his occiderunt. {b) In the foregoing point out {a) an ablative abso­ lute {b) an ablative of instrument (c) a. Dativus Commodi {(l) a Perfect used as a Past Indefinite {e) an accusative of direct object (f) a Protasis. MATRICULATION, O.T. 1870. XXV11

2. Decline throughout, eques, falx, iste, iter, ullus, velum. ( Three must be done right.)

3. Write down throughout Past-Perf. Subj. Act. cogo, Perf. Ind Act. facio, Pres. Subj. Pass, moneo, Imperative Act. nolo, Pres. Ind. Act. possum, Imperative Pass, teneo. • {Three must be done right.)

4. {a) Write down the principal parts of claudo, do, fugio, gero, jubeo, peto, tendo, verto, and {b) Write down the 1st Person Plural of these tenses Fut. Ind. Pass, abdo, Past-Imp. Subj. Act. fero, Fut.-Perf. Ind. Act. jungo, Pres. Ind. Act. obsideo, Perf. Subj. Act. possum, Perf. Ind. Pass, utor. {Four in {a) and three in {b) must be done right.)

5. Parse these nouns, mquinoctii, loca, longinquiori- bus, pulverem, triplici, usui; and these verbs, consuerint, discesserunt, gesturus, petissent, succisus, ventitaret. {Six must be done right according to instructions.)

6. Give the meaning of each of the following, the word whence it is derived and its meaning, the termination that forms it and its meaning, gu- bcrnator, flumen, pedestris, perfidia, quisquam, ratio.

7. Translate the following and give the rule for every case other than the nominative found in thera— (a) Hac audita pugua, maxima pars Aquitanire sese Crasso dedidit. 62 XXV111 EXAMINATION PAPERS,

{b) Legati hfec so ad suos relaturos dixerunt, et post diem tertium reversuros.

8. {a) Give the Latin for these phrases (1) We must use new words (2) I desire to hear from you what you think (3) Let us imitate our ancestors (4) I scarcely knew what you were going to say. {b) Explain what is meant by the Supine of a Verb, and by an accusative of nearer Definition. {c) Name two Neuter Passive Verbs, and two In­ ceptive Verbs.

ENGLISH.

Professor Irving.

{The first five questions must be answered satisfactorily.)

1. Write out the following correctly spelt— Of the noomcrus pamflets in witch the kaws of the Koart and the Church wair eegorly and ankshusly pleeded befour the Puritan, now the arbitre of the fait ov his porsckyooters, wun oanly iz rememberd the leter two a Discentre. In this masterlie litle tiakt awl the argyuments wich cood kouvins a Nonconformist that it wos his deauty and his iutrest to profur an aliaus with the Church two aim allians with the Kourt waire kondeuced into the smalcst kumpas, arainged in the moast purspickuus ordar, and enfoursed with MATRICULATION, O.T. 1870. xxix

an clokwens ernest indede yet nevur in its ut- moast veheemens transgresing the limmitts of exackt good sence and breding.

2. Analyse the following— (1) Last came, and last did go, The Pilot of the Galilean lake. (2) Fame is no plant that grows on mortal soil. (3) Weep no more, shepherds; For Lycidas your sorrow is not dead, Sunk though he be beneath the watery floor.

3. Parse every word in the following according to Morell's scheme— Together both, ere the high lawns appeared, We drove afield.

4. Write a description of a pocket-knife, such as would be intelligible to a person who had never seen one.

5. Correct whatever you think wrong in the follow­ ing, and give your reason for thinking it wrong— {a) The persons whom he stated had acted of this fashion were summoned. {b) When you have fallen that tree, fetch your axe home with you. (c) Neither the one, or the other expedient was availed of.

6. Give the meaning and the etymology of the fol­ lowing—amarantus, due, ore, pansy, praise, re­ compense. XXX

7. Give the meaning of the following words; and that of the prefix or affix by which each is formed— afield, homely, mitred, ruth, thankless, weanling.

8. Give the correct name according to Morell of each of the sentences or phrases in italics— (1) We shall reap if we faint not. (2) The eye was made^r seeing. (3) It is doubtful where Attila was buried. (4) Remote from towns he ran his godly race. 9. Give Crombie's criticism on-the following— (1) I had rather not. (2) He gave me a book. (3) No one, him except, was wiser than Solomon.

10. State and illustrate the four canons for decision between two forms of expression with the same meaning both of which are sanctioned by usage.

FEENCH.

Professor Irving.

VOLTAIRE, Charles XIL, Books III. IV. V. DE FIVAS, Grammar. TRANSLATION.

{The first four questions must be answered satisfactorily.)

1. Write down throughout the Preterite Definite of aller, the Future Absolute of courir, the Present MATRICULATION, O.T. 1870. XXX1

Indicative Interrogative of donner, the Impera­ tive of 6tre, the Present Indicative of tenir, and the Present Indicative of vouloir. {Three must be done correctly.)

2. Writedown (a) the 1st Persons Singular Present and Imperfect Indicative and Imperfect Sub­ junctive of avoir, croire, dire, cteindre, faire, finir, suivre, voir: and {b) the 3rd Sing. Future Absolute of acquerir, the 2nd Plur. Pre­ sent Indicative of appeler, the 3rd Sing. Pre­ sent Subjunctive of boire, the 3rd Sing. Present Indicative of croitre, the 2nd Sing. Imperative of dormir, and the 2nd Plur. Imper­ fect Subjunctive of mettre. {Four in {a) and three in {b) must be done correctly.)

3. Translate literally— Une partie de cet argent servit a manager des intrigues dans le serail, ;\ acheter la faveur des vizirs, ou a procurer leur perte: il rdpandait I'autre partie avec profusion parmi ses officiers, et les janissaires qui lui servaient de gardes a, Bender. Grothusen, son favori et tresorier, etait le dispen- sateur de ses liberalites: c'etait un homme qui, contre 1'usage do ceux qui sont en cette place, aimait autant il donner que son maitre. Il lui apporta un jour un compte de soixante mille ecus en deux lignes: " Dix mille ecus donnes aux Suedois et aux janissaires par les ordres genereux de sa majeste, et le reste mange" par moi." " Voilji, comrae j'aime que mes amis me rendent leurs comptes,ditce prince: Muliern me fait lire des pages enticres pour des sommes de dix mille francs ; j'aime raieux le style laconique de Grothusen." XXX11 EXAMINATION PAPERS,

Un de ses vieux officiers, soupgonne d'etre un peu avare, se plaignit a lui de ce que sa majeste don- nait tout il Grothusen. " Je ne donne de 1'argent, repondit le roi, qu'il ceux qui savent en faire usage." Cette geuerosite le reduisit souvent h n'avoir pas de quoi donner. Plus d'economie dans ces liberalites eilt ete aussi honorable et plus utile ; mais c'etait le defaut de ce prindo de pousser h 1'exces toutes les vertus. 4. Translate into French— (1) The least excusable of all errors'is that which is voluntary. (2) There is nothing w-hich one gives so liberally as advice. (3) He said that he was an Englishman and a mer­ chant. (4) Was it he who spoke ? No it was I. (5) Your letter of the sixteenth of June reached me on the eleventh of August. (6) Here is a gentleman to whom you wish to speak. 5. Give the French for, this conquest, her fear, my knee, his name, this shoulder: to conquer, to escape, better, immediately, useful; and the Eng­ lish of au-delfl, boue, convenir, dore, epuiser, gue, ouvrier, pluttit, recrue, supplice. 0. Translate into French— {a) He forbade the soldiers going away to pillage, under penalty of death. {b) Remember, my friend, that if I have taken away a kingdom from King , I have taken nothing of it for myself. MATRICULATION, O.T. 1870. XXxiii

(c) The Swedish king had helped his friends and overwhelmed his enemies without exacting the smallest fruit of his victories.

. How is the feminine of adjectives usually formed in French ? Name three principal classes of excep­ tions, with two examples of each.

. {a) Give with their meanings three verbs that take no preposition before an infinitive following. (J>) " The past participle of neuter verbs conjugated with avoir never varies." Explain the reason. (c) Name two classes of adjectives which in French always follow the substantive, and give two ex­ amples of each class.

ARITHMETIC.

Professor Wilson.

All working required in obtaining a result must be sent in, as part of the answer. Every result must be reduced to its simplest form. When an answer is not exact it must be worked to three places of decimals. 1. Write down in words the quotient and the re­ mainder obtained by dividing Five thousand and seventy-two billions seven hundred thousand six hundred and twenty-eight millions by Two hun­ dred and fifty-three thousand five hundred millions seven hundred and eighty-one thousand. In this case an integral quotient only is required. 63 XXXIV EXAMINATION PAPERS,

2. A person buys oranges at eleven pence a dozen and sells them thirteen for a shilling : how many must he sell to gain a profit of £100 ? 3. A river brings down mud which is deposited at its mouth in a layer one ten-thousandth of an inch in thickness every hour: what thickness will be de­ posited between noon on the 1st of January 1869 and noon on the 28th September 1871 supposing the deposition to go on uniformly ? 4. Supposing an alloy of twent}'-two parts gold and two parts silver to cost £3 17s. lid. an ounce and silver to cost 5s. Gd. an ounce; what is the cost of the gold ? 5. A clock is right at 4 a.m. on the 1st of January 1870 and gains uniformly 2 minutes 5 seconds in 24 hours : when will it next shew correct time ? 6. A metro is 39-37079 inches: express a kilometre in miles. *.7. A litre contains 61-027 cubic inches, and a gallon contains 277-274 cubic inches: supposing a bottle to be one-sixth of a gallon express a bottle in litres.

to a 8. Reduce l37fT» + 5 a + J +-^ + W decimal.

n T, j '47 - (-5 - -0303) t0 a vul ar frac 9. Reduce -0873 _ (.0083 + -06) S " tion in its lowest terms. 10. Find the number of }'ards of fencing required to enclose a square paddock of 40 acres. ALGEBEA.

Professor Wilson.

All work required in answering a question must be sent in as part of the answer. Every result must be reduced to its simplest form.

The numbers a and x are multiplied together and also the numbers b and y and twice the latter pro­ duct is subtracted from three times the former; and the remainder is then divided by the product of the sum and the difference of a and b : Write down the algebraical expression for the result.

T Twice the number x is multiplied three times in succession by three times the number y and three times the number y is subtracted from the result: Write down the algebraical expression which when multiplied by itself will give the remainder.

3. The circumference of a circular race course is 4a chains: A person starts from the winning post and rides Sa — b chains, then turns back and rides 2a + b chains, then turns again and rides forwards 5a — 26 chains and again turns back and rides a — 46 chains: How far is he then from the winning post and in which direction ?

4. Reduce to its simplest form 2J + 3^26 1 + 6a6

2b3a-2b 1 + b*«6 XXXVI EXAMINATION PAPERS,

5. Multiply together ? — ? and SaUSaz + z2) + (3a« + 4az)z« z a

r <%.,;< 7b°- . «« - 76* + 12aa; -I- 10a;2 o. Substitute etor arm . 6« a + 2x

7. Substitute 6ab2 for x in -— — SX + 5- - — 5a2 « a? a- 8. Substitute 1 , , 1 , , . a2 + 62 - 2a6 tor a and for 6 in ^-^ x — y x + y a- + 0s + "aO 9. A person walks x miles at five miles an hour and x miles further at three miles an hour, he then returns home at an uniform rate and perforins the 2a; miles in the same time it took him to walk out: Find the rate at which he walked home.

10. Solve the equation {x — l){x-2) = {x- 3)« + 2{x - 1)

11. Solve the equation {a + bx) - 2{a - x) = a{l — x) — 3(6 - a>) MATRICULATION, O.T. 1870. XXXVii

EUCLID.

Professor Wilson.

Algebraical Symbols must not be used.

Book I.

Three of these must be written out.

The angles at the base of an isosceles triangle are equal to one another, and if the equal sides be produced tho angles on the other side of the base will be equal.

2. To draw a straight line perpendicular to a given straight line of an unlimited length from a given point without it.

3. The angle which is opposite to the greater side of a triangle is greater than that which is opposite to the less side.

4. If two triangles have two angles of the one equal to two angles of the other each to each and one side in the one equal to ono side in the other, viz. the sides adjacent to the equal angles in each triangle „ the other sides shall be equal each to each.

5. To describe a parallelogram that shall be equal to a given triangle and have one of its angles equal to a given rectilineal angle. XXXV111 EXAMINATION PAPERS,

Book II.

Two of these must be written out.

6. If a straight line is divided into any two parts the rectangle contained by the whole and one of the parts is equal to the rectangle contained by the two parts together with the square on the aforesaid part. •

7. If a straight line is bisected and produced to any point the rectangle contained by the whole line thus produced and the part of it produced together with the square on half the line bisected is equal to the square on the straight line which is made up of the half and the part produced.

8. To divide a given straight line into two parts so that the rectangle contained by the whole and one of the parts shall be equal to the square on the other part.

HLSTOEY. Professor Hcarn.

Candidates are to select any two, but not more, of the subdivisions of each question.

I.—Describe the geographical position of each of the following places; and state, with their respective MATRICULATION, O.T. 1870. XXXIX

dates, the principal historical events connected with each place:— 1. Athos : Byzantium : Persepolis. 2. Alesia: Cremera: Tarsus. 3. Aboukir : Dunbar: Navarino.

II.—Give, with tho proper dates, some account of each of the following persons :— 1. Euclides: Perdiccas : Thimbron. 2. Archimedes: A. Corn. Cossus : Viriathus. 3. The person from wrhom Adelaide was named : The person from whom Mel­ bourne was named : The person from whom Sydney was named.

III.—Give, with the proper dates, some account of each of the following events :— 1. The Death of Demosthenes : The Battle of OEnophy ta : The Voyage of Nearchus. 2. The Battle of Munda: The Formation of the Second Triumvirate : The First Consulate of Marius. 3. The Battle of Fontenoy : The Emigration of the Pilgrim Fathers : The Affair of the Shannon and the Chesapeake.

IV.—State the principal events in each of the following years:— 1. B.C. 494: 387 279. 2. B.C. 494: 390 279. 3. A.D. 800: 1400 1800. xl EXAMINATION PAPERS,

GEOGEAPHY.

Professor McCoy.

1. Describe the periodical currents of the Pacific and Indian Oceans, with their times and directions. 2. Describe the American rivers flowing into the Arctic Ocean, with their associated lakes.

3. Trace the boundary of the geographical distribu tion of tigers at the present time.

4. Draw a rough outline map of Asia.

5. Mark out and name, on the above map, the chief political divisions.

6. Mark on the above outline map and name the chief rivers running south.

7. Mark with numbers on the above map, and name on a corresponding list the principal islands.

8. Mark on the above map each of the seas.

9. Give any general and political information in your text-book relative to India.

10. Trace the northern limit of the growth of the vine in the Northern hemisphere. ORDINARY EXAMINATIONS, O.T. 1870.] xli

ORDINARY EXAMINATIONS FOR THE DE­ GREES OF B.A., LL.B., AND M.B., AND FOR THE CERTIFICATE OF C.E.

JUNIOR GREEK. Professor Irving.

HOMER, Odyssey, Books XXI.-XXIV. PLATO, Apology.. SMITH, Student's Greek Grammar. 1. Translate literally either (a) or (6), and either (c) or {d)— {a) "iic apa tpiitviicrag elpvcnraTO ipaayavov 6i,v XaXi.Eoi', afiipOTEpwdEy iiKa-)QiEvov, UXTO B' ETC avru SfjEpBaXia la^iov Ii B' ci/xapTi) Blog 'OBvcrcEvg 'lor aTroirpo'Ulg /5UXXE crrijBog Ttapa juafoc, 'Ev BE ol ijirtiTt trij^E Ooor fliXog' EV B' apa xctpbg i>diryayoy I)KE ya^d^E, 7TEptppr]Bi)g BE TpairE^r) KaTTTTfiTE Bii'TjdEig, and C E'iSara \EVEV kpa^E Kal SE'TTOC a/MpiKvTTEXXoy' o Sk yfiova TVTTTE fiETw-io Qvfjio dym^ioy, TTIJOI OE Opovov dfiipoTEpoiaiv Aam'^ur ETiyacrcre' /car ofdaXfiwy B' i\vT &)(Xvg.

(6) 'iic (fiaTO, rip B' ETI fidXXoy hip' ifiepov Spcre yvoto' KXaiE B' iywv aXo^oc Ov/.tapla, KEOva iBv'tav. Sic B' or' dy dcnrewtog yi] yi^OfiEvoicn

LTaOpot B' i^Efvyov TroXiijg dXbg ijjrEipoyBE N>])(6flEVOt, TroXXlJ SE TTEpl -)(po\ TETpOlpEy &X/J.t], Aairdawi B' EirEJiav yairjg, KaKOTqra ifivyovTEg' iir apa rrj aavacrrbg Eijy vocrig Eiaopoixrri, Astpijg B' ov mo ird/jtrray diplETO TTI}^EE XEVKUI,

{6) 'AvaXdfliaftEy ovy il, dpyfig, rig f) Karrjyopla Early, k\ »)C f) Efiri BiafloXi) ylyovev, r) Si) cai viard/iiiy ME'- Xnrog fiE Eypdif/aro rfjv ypaiptjy TavTijy. EIEV. rl BI) Xiyoyrsg BiiflaXXoy ol BiafldXXoyrcg; &airtp ouy KarriySptoy n)y avTuifjioatav Bel dvayywvat avr&y. Swirpdrijc dBiKt'i Kill irEpiEpyd^Erai ^qriuv rd TE UTTO yijc rai rd Eirovpavia, Kal Toy f)TTh) Xoyov KpeirTio TTOidy, Kal aXXouc radra raura BiBd>v. Toiaurij rig icrri' raura yap kwpdTE Kal avrol ey TT) 'Apiaro- ipdvovg KiitfiaBia, 'SnoKpdrn rtvd EKE'I TTEpiipEpofiEvoy, ipdiTKOvrd TE aEpoflaTEly Kal dXXi/v •roXXi-;!' ipXvapiay cpXvapovyra, S)V iyi> OVBEV OVTE jJtiya OVTE crfitKpbv iripi errata).

(a) "lows ovy avrig E'LTTOI, Stywy BE Kal ifa^r^lav ayiav, (5 Saitrpartc, ov%oiog T EUEL i}fiiv E^cXdiov £r\v; Tourt Br} Ecrn irdyriav y^aXETrwraTOv rrtiaai Tivag v/iHy. lav TE yap Xiyia, on rii Oeiji dvEtdEly TOVT' EITTI Kal Sid TOVT' aBvyarov f]Ov)(iav dyEiy, ov TTEIVEOQE ^IOI v VfiE~ig Efiou aKoiiETE BiaXEyofiivov Kal Efiavrby Kal&XXovg ti,ETat,ovTog, b Be dyE^Eraorog fliog oh fliurbg dvdpit- rrii), ravra in ijTToy TTEIUEUQE fioi Xiyovn. ra BE I^EI fiiy ovroig, oig eyo> (^VfJti, <5 dyBpig, ITEIOEIV BE OV p'dBiov.

Write down the fourth and the eighth lines in {a) and in (6)—marking the feet and the quantity of every syllable in them. ORDINARY EXAMINATIONS, O.T. 1870. xliii

2. Parse these verbs aTrarjjfl/JfE, dvolcru), yEyovwg, iyyuiKag, ttopdrE, I'I^IOV, djrwGOLtEy, E^ii/jrrai, JJpEE, diJKE, TrXriyEVTii, \alpovcrt. {Six must be done correctly thus. Give tense mood voice and principal parts, namely, first person singular present future perfect and second aorist active and perfect passive, if in use; if not, of the present future perfect and aorist employed..)

3. (a) Decline BiBovg, ovrog, xpfjfia. (6) Point out any irregularity of form in £rjy, BdTToy, p.yj]ari\pEaai, •^in^iEtadE.

4. {a) Name two classes of adjectives which in Greek require a genitive. Give with their meanings two words of each class. (6) Name two uses of the accusative case in Greek, and give an example of each use.

5. (a) Quote or construct an example of the use of VTO with each of the three cases it governs, and give the English. (6) How does the meaning of av-og vary with the position of the article ?

6. (a) Name and exemplify three of the uses of the Greek Subjunctive in principal (or simple) sen­ tences. (6) How are the two Principal Conjugations in Greek distinguished ? What are they severally called and why ?

7. {a) Give the meaning of the following Greek ter­ minations forming nouns, and an example of each -EIOV -Evg -iaKog -rpoy. xliv EXAMINATION PAPERS,

(6) What is the meaning of the following Verbal Roots in Greek ? What the form of the present in what they are found ? Quote one or two English words embodying each. <5o- Xsy- CTKETT- ray-.

8. Translate and explain fully the construction in— {a) viroXdfloi dv oiv Tig v/jtuiy (6) dfiEXijirag loi-irEp ol TTOXXOI (c) dyBp&y fxvr)ini)piov KEyoXi^/xivog {d) aropEijov Xiyog oippa Kal avrbg Xi£n/j.ai

9. {a) Where and what were, Paros, Pylus, Olympus, Thebes ? (6) Who were Agamemnon, Hephtestus, Hesiod, Minos ? <

10. Derive explain fully the formation and give the meaning of avrorryfEBidliEiv, Birrog, Kdi/iaiBoTroiog, XvcrtTeXiiv, avvTErayfXEvuig, iPpovnirriig, ddarog, dvd- Orjfia, iipnjxipwg, tvEpyeaiq, 7roXvKfM)TOg, ravvaiTTTEpog.

JUNIOE LATIN. Professor Irving.

CICERO, Speeches against Catiline. VIRGIL, yEneid VII. VII I. IX. SMITH, Student's Latin Grammar. 1. Translate literally— (a) Sed cur tarn din de uno hoste loquimur, et de eo hoste, qui jam fatetur se esse hostem, et quem, ORDINARY EXAMINATIONS, O.T. 1870. xlv

quia, quod semper volui, mums interest, non tirneo; de his, qui dissimulant, qui Romre rema­ nent, qui-nobiscum sunt, nihil dicimus ? Quos quidem ego, si ullo modo fieri possit, non tarn ulcisci studeo, quam sanare, et ipsos placare rei- publicas; neque, id quare fieri non possit, si me audire volunt, intelligo. Exponam enim vobis, Quirites, ex quibus generibus hominum istaj copice comparentur; delude singulis medicinam consilii atque orationis meae, si quam potero, afferam. Unum genus est corum, qui, magno in acre alieno, majores etiam possessiones habent, quarum amore aduucti dissolvi nullo modo possunt.

(6) In animis ego vestris omnes triumphos meos, omnia ornamenta honoris, monumenta glorias, laudis insignia condi et collocari volo. Nihil me mutum potest delectare, nihil taciturn, nihil de- nique ejusmodi, quod etiam minus digni assequi possint. Memoria vestra, Quirites, nostra; res alentur, sennonibus crescent, litteraruin monu- mentis inveterascent et corroborabuntur: ean- demque diem intelligo, quam spero teternam fore, et ad salutem Urbis et ad memoriam consulatus mei propagatam : unoque tempore in hac republica duos cives exstitisse, quorum alter fines vestri imperii non terra?, sed cojli regionibus terminaret, alter ejusdem imperii domicilium sedemque ser- varet.

(c) Namque ferunt fama Hippolytum, postquam arte novercte Occident patriasque explerit sanguine pcenas Turbatis distractus equis, ad sidera rursus iEtheria et superas cteli venisse sub auras, Poeoniis revocatum herbis et amore Dianas. xlvi EXAMINATION PAPERS,

Turn Pater omnipotens, aliquem indignatus ab umbris Mortalem infernis ad lumina surgere vitse, Ipse repertorem mediciine talis et artis FulminePhcebigenam Stygiasdetrusit ad undas. At Trivia Hippoh'tum secretis alma recondit Sedibus, et nymphai Egeriae nemorique relegat, Solus ubi in silvis Italis ignobilis nevum Exigeret, versoque ubi nomine Virbius esset. Unde etiam templo Trivise lucisque sacratis "" Cornipedes arcemur equi, quod litore currum Et juvenem monstris pavidi effudere marinis.

{d) Turris erat vasto suspectii et pontibus altis, Opportuna loco; summis quam viribus omnes Expugnare Itali summaque evertere opum vi Certabant, Troes contra defendere saxis Perque cavas densi tela intorquere fenestras. Princeps ardentem conjecit lampada Turnus, Et flammam adfixit lateri; qiuc plurirna vento Corripuit tabulas et postibus hcesit adesis. Turbati trepidare intus, frustraque malorum Velle fugam. Dum se glomerant, retroque re- sidunt In partem, quae peste caret, turn pondere turris Procubuit subito, et ctulum tonat omne fragore. Semineces ad terram, immani mole secuta, Confixiquo suis telis et pectora duro Transfossi ligno veniuut. Vix unus Ilelenor Et Lj'cus elapsi. Write down tho first four lines of (c) and {d). Mark the feet and the quantity of each syllable.

2. (a) Parse these words—alacres, ansa, conceperit, excubiis, latro, locujiletes, patefacta, referebat, secius, ORDINARY EXAMINATIONS, O.T. 1870. xlvii

(6) Note and explain anything peculiar in the form of admittier, crateras, Homolen, intrastis, Ru- tulum.

3. {a) Translate and explain fully the construction in (1) Eos hoc monco, (2) fluvio succedit opaco, (3) Laurus erat tecti medio, sacra comam, (4) Qui sese in bella sequantur, praestantes virtute legit. (6) State and explain by an example of each what is meant by a Genitive of Price, a Dativus Corn- modi, an Accusative of Space.

4. {a) Give the meaning and the full derivation of alipedes, decoctor, exsilium, fecundus, invidia, omen, patera, pulverulentus, seminarium, setiger. (J) What kinds of words arc formed in Latin by the following terminations -ulus, -osus, -io, -eus ? Give an example of each, and state the kind of word to which each termination may be appended.

5. {a) What are Heteroclite Substantives ? Name two. (6) What is the irregularity of conjugation of capio? Name two verbs similarly conjugated. » {c) What are Impersonal Verbs ? What are their two principal classes ? {d) Name, with their meanings, two Latin adverbs denoting motion from a place and two denoting manner. {e) Give the 1st person perfect of amicio, ferveo, premo, sedeo, soleo, tondeo. 0. {a) Name and define the two parts of Syntax. (6) Name two classes of verbs which take after them a Nominative Predicate. xlviii. EXAMINATION PAPERS,

(c) What singular use of the Dative is found in Virgil ? {d) What difference is observed in Latin between the uses of the two tenses of the Imperative Mood ?

7. Give the Latin for each of the follow'ing and state the Rule of which it is an example— (a) Ctesar came to Gomphi, which is a town of Bceotia. (6) The people made Ancus king. (c) What else was he to do ? {d) Though Phocion might have been rich, he was poor.

8. Translate and explain— (a) Quosque sccans infaustum interluit Allia nomen. (6) Hfec pater iEoliis properat dum Lemnius oris. {c) Qui vadimonia deserere quam exercitum malue- runt. {d) Quod mihi primum post banc urbem conditam togato contigit. {e) Legum rera liquefacta sunt.

9. (a) Who were the following—Anchisiades, Cisseis, Danae, Hannibal, Pompeius, Sulla ? (6) What and where were Anio, Ciminius, Helicon, Inarime, Scylla, Picenum ? ORDINARY EXAMINATIONS, O.T. 1870. xlix

ENGLISH AND LOGIC—PABT I. Professor Irving.

1. {a) Name with examples some of the orthographic expedients by which English spelling seeks to distinguish e and o long from e and o short. (6) Name with examples tho four diphthongs as­ signed by Latham to English.

2. {a) Give an example of an English derivative formed by each of the following suffixes -t -ing -y -ness. State what sort of words is formed by each. (6) To what other words does the pronoun it be­ long ? What has it lost, and what is the mean­ ing of the last letter in it ?

3. (fl) On what ground does Latham deny that He and She are personal pronouns ? (6) Give the Anglo-Saxon and the old English inflections of the Present Tense, I love.

4. («) Names are of two kinds variable and invaria­ ble. Explain this with examples. (6) What is the construction in (1) I have ridden the horse (2) Rob rne the Exchequer (3) He sleeps the sleep of the just.

5. {a) " There has been no chemical combination of English and Welsh only a mechanical admix­ ture." Explain this. (6) What causes other than the Norman Conquest can be assigned for the breaking- up of the old English system of inflexions ? c 1 EXAMINATION PAPERS,

6. {a) What is Mr. Guest's view of the relation be­ tween Angles and Saxons, or Engle and Sexe ? (6) Give the modern English for (1) Ice hafe wennd intill Englissh godspelles halg-he lare afftcrr thatt little witt thatt me min Drihhtcn hafethth lenedd. (2) Me befel a ferly of fairye me thoghte. To what date would you refer each and why ?

7. («) Thomson says that Pure Logic is a science rather than an art. Give briefly bis reasons. (6) Explain what is meant by calling Logic the Architectonic Art.

8. {a) State and explain the five steps involved ac­ cording to Thomson in the process of Abstraction. (6) Explain Thomson's division of nouns into uni vocal, equivocal and analogous.

9. {a) Thomson divides all prcdicables into substitute and attribute. Illustrate by examples and ex­ plain the principle of the division. (6) Into what Hypothetical Judgment can you ex­ pand the categorical proposition Every X is Y.

10. (rt) What does Whately mean by dividing all words into Categorematic, Syncategoreinatic and Mixed ? Give examples of each class. (6) When are terms called by Whately Opposite, and when Compatible ?

11. («•) Give Whately's division of Plane Superficial Figure and explain the principle, of it. ORDINARY EXAMINATIONS, O.T. 1870. Ii

(6) Give what Whately would call a Logical Defi­ nition of Knife and of Chair.

12. {a) Which of the four forms of proposition dis­ tribute the Subject ? Why cannot A be converted simply ? (6) State and prove the Law of Subcontrary Oppo­ sition. /

ENGLISH AND LOGIC—PART II. Professor Irving. CRAIK, English of Shakespeare. WHATELY, Rhetoric. WHATELY, Logic. THOMSON, Laws of Thought. 1. Give tho substance of Craik's notes on the follow­ ing— {a) If you shall find them decked with ceremonies. (6) That gentleness as I was \vont to have, (c) Let us not break with him. {d) Shall it not grieve thee, dearer than thy death.

2. In what senses now forgotten are the following wTords used in the Julius Ctesar ? brook flatterer knave liable passion success. Quote passages in which they occur.

3. Certain emendations have been proposed in the following. What and why ? {a) Ctesar doth bear me hard. (6) For if thou path, thy native semblance on. (c) I have neither writ, nor words, nor worth. r O Iii EXAMINATION PAPERS,

4. Give the Etymology of these words, bayed, cautelous, dress, herse, kerchief, worthy.

5. (a) What is Whately's opinion of Quinctilian as a writer on Rhetoric ? (6) Explain by an example the Argument from Progressive Approach.

6. {a) What is meant by the Exaggerating aud by the Extenuating method ? (6) Quote or construct examples of the mixed metaphor.

7. {a) What are the four functions under which the Logician must notice language ? (6) Explain and exemplify Thomson's view of Defi­ nition.

8. {a) How does Thomson distinguish Explicative from Ampliative judgments ? (6) Give an example of Immediate Inference by Added Determinants. Justify the mode of In­ ference. 9. {a) Construct an example of the Goclenian Sorites, and state its laws. (6) State and explain tho Principle of Contra­ diction.

10. (ft) Give Whately's definition of a Fallacy. Con­ struct examples of the Fallacy of Etymology, and of Ignoratio Elenchi. (6) What is meant by Reduction ? What is sup­ posed to be its advantage ? Construct a Syllogism iu Dimaris and reduce it. ORDINARY EXAMINATIONS, O.T. 1870. liii

11. {a) Quote and explain the four mnemonic Lines which embody the Rules of the Syllogism. (6) Assuming these only prove that the Major Pre­ mise in the First Figure must be universal.

12. Put into strict Logical form and examine these arguments (a) Some objects of great beauty answer no other perceptible purpose but to gratify the sight ; many flowers therefore answer no other purpose but the gratification of the sight. (6) Jupiter was the son of Saturn : therefore Sar- pedon was the grandson of Saturn.

SENIOR GREEK.

Professor Irving.

HERODOTUS, Clio. ARISTOPHANES, Clouds. TRANSLATION.

1. Translate into Greek (Herodotean or Attic)— But the flame was too strong: and when Croesus saw that the mind of Cyrus was changed, but that the men were not able to quench the fire, ho prayed to Apollo to come and save him, if over he had done aught to please him in the days that were passed. And suddenly the' wind rose, and clouds gathered where none had been before and there burst from the heaven a great storm of rain which put out the blazing fire. Then Cyrus knew that Croesus was a good man, liv EXAMINATION PAPERS, and that the Gods loved him, and when he came down from the pile he said, 0 Croesus, who per­ suaded thee to march against my land ? and to become my enemy rather than my friend ? And Croesus answered, It is the God of the Greeks, 0 King, who urged me on, for no man is so senseless as to choose war rather than peace, in wliich the children bury their fathers while in war the fathers bury their children. But so it pleased the Gods that thus it should be. 2. Translate any three of the following— {a) ov yap TOT' EvOvg )(fii}y IT apdrrEaOai TE Kal TrarElcrdai, IfSeiy KEXEIIOI'6', do-TTfpEt rirnyag laTidvTa; ST. roiavra fiEvroi Kal TOT iXEyEV 'iyBov, oidirEp vvv, Kal rov ~2iifitx>yicriv iipacrK Eivai KUKOV Ttot7]T)]v. Kayo) /xi'iXig fitv, dXX' Sfiwg ^vEcr^Ofirfy TO Trptirov' ETTEtTa B' EKEXEVU avrbv dXXa fxvppiyr)y Xafloyra TU>V A\ir-)(yXov Xi^ai ri fioi' Kq.6' ovrng EvOvg EIWEI', lyw yap Alir^yXoy vofxit,io Trpwrov iv Troir]Ta~ig \poipov TrXf'wr, d^vnraTOy, crvfiipaKa, KpritivOTroiov; Kavravda Ttwg OIEITOE fiov rijv KapBlav bpE^Oelp; ofioig BE TOV Ovfiby BaKwv E07)r, OU C' AXXa TOVTLOV XEJo>' ri TCOV yEWTEpuiy, aVr' EITTI ra croipd ravra.

(6) (S GtwfiEtoi, Karepui rrpbg h/idg EXEUQE'P&IC TaXrjOij, )'j) roi' Aibyvirov rov EKdpEipayrd fXE. iwrot viKiiirai^d T Eyw Kal yofii£ol[ijjy troipog, wg h/idg ijyovfievog Eivai Qeardg CE^iovg cat ravrny croipuira-' E'XEH' TWV ifiuiy KwfiuBiSiv, TTpoirovg ))JidW di'ayfOo vfxdg, i) rrapiiry^E fxoi Epyoy TrXE'tiTToy' E'IT' dvE\wpovy vrr ai-Cpdv if>opTtKuiv ilTTvtidc, OXIK dlwg wy' TOUT ovv iijuv ixefi(j>Ofiai Tolg irofnlg, uiv U'VIEK kyio raur' Eirpay^iaTEVofiriv. dXX' oiiB'

(c) SapCtEC BE ijXwirav SICE. IITUCI) TEcrccpEcrKaiCEKaTn ORDINARY EXAMINATIONS, O.T. 1870. Iv

tyEyETO J(/UE'P>; noXiopKEOfitvu) Kpoiaa), Kvpog rij arpanr) rrj EIOVTOV Bia7TE/j.\pag iTnriag irpoEiTTE, rjl irpwrii) ETTifldrri rov rd^eog Bwpa BOXTEIV' fitra BE TOVTO, TreiprjirafiEyiig rijg arpanfjg u>g ov 7rpoey(u)pEE, eydav-a TUIV aXXdiv TTETrav/iEvwv, dyijp MdpJSoc ETTEI- pdro TTpoaflaitiov, TO) o'vvofia ffv "XpoidBng, Kara TOVTO TJjg uKpoTroXiog rij ovBEtg iriraKro ipvXaKog' ov yap i)v BEIVOV Kara TOVTO /D) dXai KOTE' diroTOfiog TE yap EITTI ravrr) tj dkpoTroXig Kal d/iay^og. rrj oiiBt M»;X7;e, o TrpoTEpov flaaiXEvg 2apBiu>v, fiovyrj ov nEpiyyEiKE Toy Xiovra rov ol }/ TraXXaKi) ETEKE, TEX- p.r\aaEo>v BiKaadvriay wg TEpiEyEi")(divTog rov Xiovrog TO TEi\og EaovTai SripSiEf dvdXwroi.

{d) EI fiev vvv TrpoeTTvdovTo y Efiadov ol BaflvXuvioi TO EK TOV KvpOV TTOlEVflEVOV, OvB aV TTEpliBoVTEg TOVg THpoag EOEXDE'LV ig T>)V irbXiy BtiipBEtpav KaKiara' KaTaKXn'iaavTEg yap dv irdaag rag kg rov TrorayuoV TrvXiBag i\ovirag, Kal avrol ETTI Tag al/tairidg dva- fldt'TEg rag rrapd ra %EIXEU TOO Trora/jov t'Xr/Xa^iE'rac, EXapoy ay cnpEag ihg iv Kvprn' vvv BE ii, dirpoaBoK))- rov aipi TTapia-r\aav ol Hipirai. OTTO BE fxEyi'i0Eog rijg TtoXiog, tug Xiyerai vwb TWV TavTr/ OLKJ)JIEVU>V, roiv •KEpl ra Eir^ara Trjg noXiog EUXMKOTIDV, rovg TO fiiaoy oiKEOvrag Ttov BaflvXwvlioy oh fiavddvEiv EaXujKorag' dXXa {TV^ETV yap aipi ioiiaav bprijv) y(OptvEiv TE TOVTOV TOV xpovov Kal iv EVTradEirjiTi Eivai, ig o oV) Kal rb Kapra iirvBovTO.

3. Point out any peculiar Ionic forms in (c) and give the corresponding Attic ones.

4. Translate explain and refer to the context— {a) aTtiKVEETai ig Tag XdpBig dvi)p crufiif>opfj i-^AfiEVOg Kal oh Kadapbg \E~tpag itov. Ivi EXAMINATION PAPERS,

(6) ippiirrai o' b floXog, TO BE BIKTVOV iWETrtVaorat • Oiiyvoi B' olfJ)lv VTTO ralg jjiopiaig diro- dpetiei. {J ) on Kal ere \vTpeovv ovra Oebv fiyr\ndjxr)v.

5. Give the meaning and discuss the derivation or formation of aKnKOwg, aTrXerog, d:roXtrapy(£7c, St<3- pv%, KarearpdipaTO, fiadriTiu), bvofia, ZvvwpiKevEirdai, aafjipopdg, rvOrjyai,

6. What and where are Araxes, Cos, Delphi, Is, Miletus, Sardis, Sunium, Tarentum ?

SENIOE LATIN.

Professor Irving.

LIVY, Books XXI. XXII. XXIII. LUCRETIUS, Books I. and VI. TRANSLATION.

1. Translate—. The Athenians now petitioned for the bodies of the slain. This petition was immediately granted. When the banquet was ended, which, according to the custom of those times, w^as given after a victory, Philip visited the field of battle. He and the companions of his victory were decked with ORDINARY EXAMINATIONS, O.T. 1870. Ivii

garlands, and in the excess of his exultation at the prosperous issue of a project, which it had taken him two-and-twenty years to execute, he made himself merry over the slaughtered Athe­ nians. The orator Demades, who was among the prisoners, had the spirit to rebuke this ignoble exultation. " Win' dost thou act the part of Thcrsites," said ho, " when fate has assigned thee that of Agamemnon ?" Philip felt the justice of the rebuke, and was so far from resenting the frankness of the Athenian, that he granted to him, and to all his fellow-citizens, freedom without ransom.

. Translate any three of the following— (a) Hoc etiam pacto volucri loca lumine tingunt Nubes, et treiuulo tempestas impete fulgit, Ventus ubi invasit nube-in, et vcrsatus ibidem Fecit, ut ante, cavam, docui, spissescere nubem ; Mobilitate sua fervoscit, ut omnia motu Percalefacta vides ardescere : plumbea vero Glans etiam longo cursu volvenda liqucscit. Ergo fervidus hie, nubem quum perscidit atram, Dissipat ardoris quasi per vim expressa repente Semina, qua) faciunt nif.-tantin fulgura (iniiimas : Inde sonus scquitur, qui tardius allicit aures, Quam quae pervoniunt oculorum ad limina nostra. Scilicet hoc densis fit nubibus, et simul alte Exstructis aliis alias super impete rniro. (6) Nee tarnon undique corporea stipata tenentur Omnia natura; namque est in rebus inane : Quod tibi cognosse in multis erit utile rebus: Nee sinet errantem dubitare et quairere semper De sumina rerurn, et no.stris diftidere dictis. Quapropter locus est intactu.-, inane, vacansque. cS Iviii EXAMINATION PAPERS,

Quod si non esset, nulla rationc moveri Respossent; namque, officiura quod corporisexstat, Officere atque obstare, id in omni tempore adesset Omnibus: hand ig-itur quidquam procedere posset, Principium quoniam cedendi nulla daret res. At nunc per maria ac terras sublirnaque coeli Multa modis multis varia ratione moveri Cernimus ante oculos ; qure, si non esset inane, Non tani sollicito motu privata carerent, Quam genita omnino nulla ratione fuissent: Undique materies quoniam stipata quiesset.

(c) Hoc statu renim in Ilispania P. Scipio in pro- vinciam venit, prorogate post consulatum impcrio ab senatu missus cum triginta longis navibus et oeto millions militum magnoquc commeatu ad- vecto. Ea classis ingens aginine oncrariarum procul visa cum magna hetitia civium socio- rumque portum Tarraconis ex alto tenuit. Ibi milite exposito profectus Scipio fratri se conjungit ac deindo cominuni animo consilioque gerebant helium. Occupatis igitur Cathaginicnsibus Cel- tiberico hello hand cunctanter Ilibeiuin transgre- diuntur nee ullo viso hoste Saguntum pergunt ire, quod ibi obsides totins Hispaniie traditos ab Ilan- nibale fama erat modico in arcc custodiri ]ira;sidio. Id unurn pignus inclinatos ad Roinanarn societatem omnium Hispanite populorum animos inorabatur, no sanguine liberum suorum culpa defectionis luoretur. Eo vinculo Hispaniam vir unus sollerti mag-is quam fideli consilio exsolvit: Abelux erat Sagunti nobilis Hispanus, fidus ante Pooni-, turn, qualia plerumque sunt barbarorum ingenia, cum fortuna mutaverat fidein.

{d) Unus nee doininoruni invitationc nee ipsius ORDINARY EXAMINATIONS, O.T. 1870. lix

interdum Hannibalis Calavii filius Perolla ad vi- num potandum impelli potuit, ipso valetudinem excusans, patre animi quoque ejus baud mirabilem interturbationem causante. Solis ferme occasu patrem Calavium ex convivio egressum sequutus filius : ubi in secretum—hortus erat posticis oedium partibus—pervcnerunt, " Consilium " inquit " ad- fero, pater, quo non veniam solum peccati, quod defecimus ad Hannibalem, impetraturi ab Romanis sed in multo majore dignitato et gratia simus Campani, quam umquam fuimus." Quum mira- bundus paler, quidnam id esset consilii, qutereret, toga rcjccta ab humero latus succinctum gladio nudat: " Jam ego " inquit " sanguine Hannibalis sanciam Ivornanum fcedus : te id prius scire volui, si forte abesse, dum facinus, patratur, malles."

3. Mention any archaic forms employed by Lucretius.

4. Translate explain and refer to the context— (a) Nunquam, donee sanguinis Barcini quisquam supersit, quietuia Romana fcedera. (6) Punicum abhorrens ab Latinorum nominum prolatione pro Casino Casilinum dux ut acciperet fecit, (c) Duobus ducibus unus resistas oportet. {d) Cohortati sunt, ut qui redempturis auxissent patrimonia, rcipublica; ex qua crevissentad tempus commodarent. (e) Sic et Averna loca alitibus summittere debent Mortiferam vim. {f) Nee clam durateus Trojanis Pergama partu Inflamasset equus nocturno Grajugenarum. 5. Give the meaning and discuss the etymology and Ix EXAMINATION PAPERS,

the formation of oestifer, alumnus, bruma, hospes, narro, pellicio, propalara, scintillo, sursum, vir- gnlta.

6. (a) Who were Empedocles, Iphianassa, Hammo, Hiero, Lutatius, Xenophanes ? (6) What and where were Charybdis, Cercina, Cremona, Iberus, Isara, Venusia ?

Professor fVilson.

GEOMETRY.

Four of these must be done. Algebraical symbols must not be used. 1. Parallelograms upon the same base and between the same parallels are equal to one another.

2. The straight line drawn at right angles to the diameter of a circle from the extremity of it falls without the circle; and no straight line can be drawn from the extremity of a diameter so as not to cut the circle unless it be at right angles to the diameter.

3. The angle in a semicircle is a right angle; the angle in a segment greater than a semicircle is less than a right angle; and the angle in a seg­ ment less than a semicircle is greater than a risht i n angle. ORDINARY EXAMINATIONS, O.T. 1870. Ixi

4. If a straight line touches a circle and from the point of contact a straight line is drawn cutting the circle the angles which this line makes with the tangent shall be equal to the angles in the alternate segments of the circle.

5. To inscribe a circle in a given triangle.

6. If the angle of a triangle is divided into two equal angles by a straight line which also cuts tho base the segments of the base shall have the same ratio which the other sides of the triangle have to one another.

7. Similar triangles are to one another in the dupli­ cate ratio of their homologous sides.

8. To draw a straight line perpendicular to a plane from a given point above it.

TRIGONOMETRY.

Four of these must be done> 9. Define the sine the versed sine and the cosine of of an angle and starting from the definition prove that sin2^4 -f- cos2^4 = 1

10. The sines and the cosines of A and B being given investigate a formula for cos {A + B)

11. Find the sines and the cosines of 30° and of 45° and deduce the cosine of 75°. Ixii EXAMINATION PAPERS,

12. Prove that sin A + sin B = 2 sin A + B cos A ~ B

62 + c2 — a2 13. Prove the formula cos A = 26c 14. From a given point a line is drawn due north 30 feet long and from the end of this line one is drawn eastwards 20 feet long thence northwards 10 feet thence eastwards 25 feet thence southwards 15 feet and thence westwards 20 feet: Find the length and direction of the line joining this to the start'ing point.

15. Find an expression for the area of a triangle in terms of two sides and the included angle.

10. Show how to solve a triangle when two sides and the included angle are given.

ALGEBRA.

Professor IVilson.

All results must be reduced to their simplest form.

1. Find the greatest common measure of Gal - lSasx + \Sa-x2 - 2Sax3 + 10a;* and. 6a* + wi3x - I8a"x- + 23(7x3 - 10a.-4 ORDINARY EXAMINATIONS, O.T. 1870. Ixiii

2. Find the square root of a* + b* + c* + d* -2a2(62 -M2)-262(c2 -d2) + 2c2{a2—d2)

3. Shew that 1 — -T—Z ^ r—••'•——' — ~ (a3 + 62 +c2){x2 +y2 + z2) _ {ay — bx)2 + (az — ex)2 + {bz — cy)2 ~ {a2 + 62 + c2) {x°- +y2 +z") 4. Substitute ax + by for x and 6a; — ay for y in x2 + Sxy + 5y'2 and find the relation between a and 6 that the result ma}' not contain a term xy

5. Substitute 1 + j-for x in - 1+- 1+- y x 6. Eliminate t from the equations x =. vt y — ut — \gt2

7. Find the value of c which makes the expression a2x2 + bx + c a perfect square.

8. Find the square root of 14 — 4V6

9. Solve the equation (' -^ = —- 1 \x + 6/ a; + a + 26 10. Solve the equations x + y + z = 12

a;-f 2# + 3z := 26 x + Sy + Qz = 45

11. Solve the equation 2 {x — 2)2 + {x — I)2 = 0

12. Solve the equations 4a;2 + dy2 =97 xy = (j Ixiv

13. The distance through which a body falls from rest in any time varies as the square of the time; a body falls 16 feet in one second; in what time will a body fall one mile ?

14. The sum of n terms of a series of numbers in arith­ metical progression is n + 2n2 ; find the first term and the common difference.

15. Find the Arithmetic the Geometric and the Har­ monic means between 2 and 18.

10. The number of combinations of n things 5 together is 19 times the number of combinations of n — 2 things 3 together; find n.

DIFFERENTIAL CALCULUS AND ANALYTICAL GEOMETRY.

Professor Wilson.

1. Differentiate

/ s i\3, x + Vq« - x*_ ^ . gin ,ax_a.t^

sin «/a + 6a;

2. Differentiate (a2 + x2)

3. Investigate the nth differential coefficient of d" cos mx ORDINARY EXAMINATIONS, O.T. 1870. IxV

A2 4. Shew that/(a + h) -f{a) -f{a)h -f"{a) j-g •,« h3 ~J"ia) -100 is intermediate between the greatest and least values of f"{z) for values of z between a and a -f- h.

5. Expand u in a scries of powers of x when «" log u = ax

6. Find the value of -1—-— v when a; = 0 2a-2 tan irx 7. Investigate an expression for the length of the perpendicular from a given point on a straight line whose equation is given.

8. Find the equation to the locus of a fixed point in a finite straight line which moves with its ends on two fixed lines at right angles to one another.

9. Find the general equation to the tangent to tho curve u = 0 at a given point.

10. Investigate the conditions which must be satisfied in order that the general equation of the second degree may represent a parabola.

11. Find the length of the perpendicular from the focus of an ellipse on the tangent in terms of the radius vector of the point of contact.

12. Find the greatest parabola that can be inscribed in a given isosceles triangle, the axis of the Ixvi EXAMINATION PAPERS,

parabola being perpendicular to the base of the triangle.

13. Integrate the following functions " 1 1 ! 1 + X + X2 ' 1 + X — x2 * «2 N/1 - X2 1 ~5 —r-i , (tan x) a -f- b cos a; 14. A circular'disk rotates about a line through its center, inclined at a given angle to its plane, find the volume of the least space in which it will turn.

NATURAL PHILOSOPHY.

Professor Wilson.

1. A force of twenty pounds is just sufficient to sup­ port a weight of one ton upon a smooth inclined plane; through what distance along the plane must the weight be moved that it may rise ver­ tically through fifty feet ?

2. A rough excentric circular pulley four inches in diameter is moveable without friction about a fixed axis one inch from its center; a weight of sixty pounds is supported by a cord which passes over it; find the greatest and the least forces which can pull at the cord consistently with equilibrium.

3. Supposing the pulley in the previous question to ORDINARY EXAMINATIONS, O.T. 1870. Ixvii

be moveable, the weight of 60 lbs. to hang from the axis and the cord to pass under the pulley one end being fixed; find the greatest and the least forces which can be applied to the other end con­ sistently with equilibrium ; the two portions of the cord not in contact with the pulley being parallel.

4. In a cubical block a pyramidal hollow is cut, the base of the pyramid coinciding with one face of the cube and its vertex with the center of the cube : find the center of gravity of the remainder.

5. Explain the terms "energy" "potential energy" and " kinetic energy." A body is projected ver­ tically upwards, find an expression for the velocity at any height and shew that the sum of the kinetic and potential energies is constant.

6. A body is projected with a given velocity in a direction making an angle a with a horizontal plane ; find the greatest height it will attain, the range on a horizontal plane and the value of tana in order that the greatest height attained may be equal to the range.

7. Define precisely the terms "angular velocity" "momentum" " moment of momentum" " mo­ ment of inertia" as applied to a rigid body. What is the " radius of gyration" ?

8. How is the kinetic enerp-v of a rigid bodv rotating: about a fixed axis expressed in terms of the angu­ lar velocity ? A wheel whose weight is 20 lbs. and radius of gyration with respect to its axis of motion one foot rotates so as to make two revolutions in one Ixviii EXAMINATION PAPERS,

second; with what velocity must a weight of 20 lbs. be moving without rotation so as to have the same kinetic energy ? 9. A body attached to a fixed point by a string 25 feet long swings round in a horizontal circle 30 feet in diameter : calculate the time of one revo­ lution ? 10. A conical vessel is filled with water, closed by a •watertight lid, and inverted; find the ratio of the pressure on the lid to the weight of the water and explain the result. 11. If the chemical combination of certain substances would produce 43 cubic feet of gas under the atmospheric pressure when the water barometer is at 34 feet, how many cubic feet would the com- , bination of the same substances produce at a depth of 50 feet in water ? 12. Describe the barometer and the mode of reading it. A barometer requires a correction + -01 when the reading is 29 inches in consequence of a small quantity of air in the upper part of the tube which extends three inches above the mercury, what.will be the correction when the reading is 30 inches ?

13. Explain what is meant by the mechanical equiva­ lent of heat. A weight of 193 lbs. is moving with a velocity of 32 feet a second : to how many thermal units is its kinetic energy equivalent ? 14. What is meant by the specific heat of a substance ? Explain how the specific heat of a substance may be determined : construct an example in the case of lead whose specific heat is '03. ORDINARY EXAMINATIONS, O.T. 1870. Ixix

15. Explain what is meant by the dew point. Describe the construction of a dew-point hygrometer.

10. State the amount of the resistance to a plane sur­ face moving perpendicularly to itself in a fluid. Find the terminal velocity of a leaden ball one inch in diameter sinking in water, the resistance to a sphere being one-half the resistance to a circle of" the same diameter and the specific gravity of lead being; 11445.

CHEMISTRY AND MINERALOGY.

Professor McCoy.

1. Define clearly the doctrines of chemical combi­ nation by weight and by volume respectively, giving examples with the proper numerical nota­ tion of the quantities.

2. Describe the methods of distinguishing acids from bases in difficult cases, and in those cases in which a given body is sometimes acid to a base and under other conditions basic to another acid.

3. State for Hydrogen.the approximate coefficient of absorption by water, index of refraction, specific gravity and other physical characteristics.

4. Give some examples of Catalysis, and discuss the theory of the action. Ixx

5. In what proportions will Hydrogen combine with Iodine, Bromine, and Chlorine, and what are the general characteristics of the compounds ?

6. What are the hardness, crystalline angles, specific gravit}', and other minernlogical characters by which you would identify Graphite.

7. Give a description of all the kinds of Pseudo­ morphism with examples of the minerals involved in the formation of each.

8. Shew how the Tetrahexahedron may be derived from the Cube, the Dodecahedron, and the Octa­ hedron respectively.

9. Explain the relation of each of the solid angles of the Rhombohedron to the crystalline axes and to the Hexagonal Prism on the suppositions of four and of three axes.

10. Define a crystallographic "zone " and shew bow- many faces could be referred to one in the Cube, the Octahedron, and the Tetrahedron.

BOTANY.

Professor McCoy.

1. What do you understand by a "superior radicle," and how does the micropyle assist to indicate it ? ORDINARY EXAMINATIONS, O.T. 1870. Ixxi

2. What are the chief relations between the embryo and the albumen in most seeds ?

3. Describe the different kinds of dehiscence of a capsule, mentioning some plants illustrative of each.

4. Many kinds of nut result from plurilocular ovaries. How do you explain this ? Refer to the changes in some plant proving your view.

5. Describe all the parts of a succulent fruit, as a Pepo.

6. How are the stamens inserted in different modifi­ cations of Perigynous and Epigyuous flowers ?

7. Describe all the parts with their relations of posi­ tion of a campylotropous ovule.

8. What are the true homologies of the parts of the so-called fruit of a strawberry ?

9. Give some common plants illustrating the distinc­ tion between dioscious and monoecious unisexual flowers.

10. Describe all the tissues successively appearing in the root of au Oak in progress of growth. Ixxii EXAMINATION PAPERS,

COMPARATIVE ANATOMY AND ZOOLOGY. (VEKTEBRATA.)

Professor Mc Coy.

1. Describe the parts, with their relative positions, of an ideal typical perfect vertebra, including diverg­ ing appendages, and distinguishing tho Autogenous from the Exogenous elements. 2. Describe all the parts of a complete vertebral seg­ ment of the thorax in a bird. 3. Describe and name all the elements in the first or most posterior vertebral segment of the skull of a crocodile. 4. Describe fully the chief osteological peculiarities distinguishing the hind limbs of birds from those of mammals, describing all the bones of each in full.

5. Describe the digestive canal in all its successive changes of name, character, and functions, from one end to the other in mammalia.

6. What are the systematic characters of each of the orders of fishes ?

7. How are the families of the Testudinata discri­ minated ? - 8. Define the families of the Diurnal Raptorial birds.

9 Characterise the orders of the mammalia.

10. Define the families of the Cctavca or whales. ORDINARY EXAMINATIONS, O.T. 1870. Ixxiii

GEOLOGY AND PALyEONTOLOGY.

Professor McCoy.

1. What are the most characteristic Trilobites of the Caradoc Sandstone formation and how do you distinguish them generically ? 2. What are the subdivisions of the Upper Silurian formations and what difficulties have arisen in classifying the Woolhope Limestone and the Mayhill Sandstone ?

3. Describe the Scotch section of the Old Red Sand­ stone or Devonian Series and compare it with that of the border counties of England and Wales.

4. What, fossils would enable you to discriminate a Carboniferous from a Permian Limestone ?

5. How would you distinguish between Leptwna, Orthis, Spirifcr, and 1'roducta, and what is the geological range of each ?

6. What are the main subdivisions of the Wealden system and what are the precise arguments used in favour of the alliance of the deposits with the Cretaceous and the Oolite respectively?

7. Describe the process by wliich coal basins have been formed and the peculiarities of the different parts.

8.vHow would you discriminate the upper chalk from the lower or chalk marl? d Ixxiv EXAMINATION PAPERS,

9. Describe clearly tho methods of forming and filling of mineral veins in the older rocks.

10. Describe tho methods of making a geological plan and a geological section of a district.

ANCIENT HISTORY.

Professor Ilearn.

1. What is the meaning of the word cliens, and what what were a client's rights and duties ? 2. State and explain the principal uses of the verb rogare and its compounds in reference to Roman legislation.

3. Explain the difference (if any) between the MYOX\\ and the Proletarii.

4. " Ut plebiscita omnes Quirites tenerent." When and by whom was this law passed ? Explain fully its meaning. Give some account of any other legislation upon the same subject.

5. How does Mr. Merivale connect the insurrection of the slaves with the growth of the tendency towards monarchy at Rome ?

6. What expedients did Sulla adopt to secure the permanency of his constitution ? ORDINARY EXAMINATIONS, O.T. 1870. IxXV

7. What provinces were established by Pompeius, and in what circumstances ?

8. Explain the nature of the Proconsular authority as held by Augustus.

9. State the propositions held by the great Roman jurists as to the origin and the extent of the Imperial authority.

10. Explain the system of taxation adopted by Con­ stantino and his successors.

11. What was the original seat of the French mo­ narch}' ? What were the circumstances and the date of its establishment ?

12. With what Emperors does Gibbon compare Charles V. and Philip II. respectively, and what in each case arc the grounds of the comparison ?

HISTORY OF THE BRITISH EMPIRE.

Professor Hcarn.

1. Describe the successive changes in the judicial polity of Western Europe.

2. Describe the successive changes in the system of public defence in Western Europe.

3. How do you account for the voluntary conversion of alodial into feudal property ? d2 Ixxvi EXAMINATION PAPERS,

4. State exactly the meaning of the following terms viz., Frankpledge: Homage: Rachimburgii: Sithcundmen: Weregild.

5. Explain the distinction between echevins, com­ purgators, and jurors.

6. " By such means a large proportion of the pea­ santry bad become hired labourers instead of villeins." (a) What were these moans ? (6) At what time did this change take place ? (c) On what occasion and in what circum­ stances do we first hear of these free labourers ?

7. What are the causes of.moral improvement which Mr. Hallam notices as operating during the latter part of the mediaeval period ?

8. Explain the difficulty of the Pope's position in" reference to Henry VIII.'s suit for a divorce from Queen Katharine.

9. What does Mr. Hallam describe as the peculiar iniquity of the condemnation of the Roman Catholic clergymen under Elizabeth ? What defence has been set up for these executions ?

10. What was the first occasion of dispute after the Reformation between the members of the Pro­ testant party ?

11. From wdiat sources do we derive our earliest know­ ledge of the debates of the House of Commons ? ORDINARY EXAMINATIONS, O.T. 1870. Ixxvii

12. Who was the last ecclesiastic that held high poli­ tical office in England and in what circumstances was he appointed ?

CONSTITUTIONAL AND LEGAL HISTORY.

Professor Hearn.

1. " Nemo potest exuere patriam." How has this maxim been affected by recent legislation ? 2. What is the date of the first English newspaper ? 3. What were the principal legal impediments to the development of journalism ? 4. What is the precise form in which the establish­ ment of Responsible Government in any colony is effected ? 5. The Electoral Franchise has often been described as a trust: State your reasons for agreeing with or for dissenting from this opinion. 6. In speaking of the so-called Royal Veto, or power of rejecting Bills, Lord Palmerston observed that this power is at the present day neither impaired nor abandoned. (a) What was" the last occasion on which this power was exercised ? (6) What is the objection to the term "veto" as descriptive of this power ? (c) What are the grounds of the assertion above cited ? Ixxviii EXAMINATION PAPERS,

7. State the different methods, whether obsolete or now in force, of creating Peers. 8. What was Lord Bacon's opinion of Cabinet Coun- ' cils, and how do you account for that opinion ? 9. In what circumstances (if any) may the Crown take notice of matters pending in Parliament ? 10. Mention some cases in which a Ministry has re­ signed because the Sovereign declined to follow its advice. 11. What was the occasion of the quarrel between LordThurlow and Mr. Pitt, and for what principle is that case an authority ? 12. How was the loss of revenue arising from the abolition of the Military tenures supplied ? State the arguments for and against this arrangement ?

POLITICAL ECONOMY.

Professor Heam. 1. State some of the definitions of Man that have been proposed from an economic point of view. How do you account for their diversity ? 2. To what circumstances is the great industrial progress of Scotland during the last century principally due ? 3. What are the peculiar advantages which England possesses as a manufacturing country ? ORDINARY EXAMINATIONS, O.T. 1870. Ixxix

4. Is a map describing the geological and chemical character of the soil of any country a sufficient guide for the practical settlement of that country ?

5. " The efficiency of natural agents is relative." (a) What do you mean by natural agents ? (6) What do you mean by their efficiency ? (c) What do you mean by their efficiency being relative ?

6. "Men always cultivate the best soils first." Is this proposition sufficiently accurate ? If it is not, in what form should it be expressed ?

7. Montesquieu says "Countries are not cultivated in proportion to their fertility but their liberty." Explain this statement.

8. What do you understand by the expression " the field of employment" in reference to capital ?

9. Gibbon observes that " the power of the sword is more felt in an extensive monarchy than in a small community." On what principle does this fact depend ?

10. In regard to labour what is the double function of exchange ?

11. Shew that the extent of co-operation is limited by the extent of the market.

12. What, according to Comte, is the function of co­ operation in the formation of society ? 1XXX EXAMINATION PAPERS,

SURFACE AND MINING SUEVEYING AND LEVELLING.

Mr. Kernot.

1. Describe Gunter's chain. What special advantages attend the adoption of the chain as a unit of length ? What are the usual sources of error in chaining, and what precautions should be adopted against them ?

2. It is required to range out a perfectly straight line ten miles long, bearing due east and west, through an undulating and thickly timbered country. How would you proceed to effect this object, and what precautions would you employ to neutralize in­ strumental errors ?

3. Explain fully the optical principle and mechanical details of the telescope of a theodolite or dumpy level.

4. Make a series of sketches showing all the mechanical details of any one of the following instruments— (a) The prismatic compass. (6) The transit theodolite. {c) The dumpy level. 5. Describe fully the adjustments of the plain theo­ dolite.

6. Plot tho following survey, test the accuracy of the work by calculation, compute the area, and deter­ mine the bearing and length of a line drawn ORDINARY EXAMINATIONS, O.T. 1870. Ixxxi through the point X, so as to divide the whole into two equal parts.

A 344 270° 10' D

463 188° 14' C

343 91° 3' B

462 400 X 370 340 310 359° 30' A ^3 Ixxxii

PEACTICAL MECHANICS.

Mr. Kernot.

1. Give a full account, accompanied by chemical for­ mula?, of the manufacture and use of lime, mortar, and Portland cement. 2. Describe fully the machinery employed in the manu­ facture of pressed bricks.

3. Give all the information you possess relative to the following materials— (a) Oregon pine. (6) Jarrah. {c) Bessemer steel. {d) Portland cement concrete. 4. Supply a sketch design for any one of the following works. The sketch should be fully dimensioned, and accompanied by a brief description containing the principal requirements of a specification, (a) A wooden lattice girder, 100 ft. span, to carry a distributed load of 100 tons. (6) A laminated arch of the same span to carry the same load, (c) A brick arch of 20 ft. span to carry a road ; the height of the road above the footing-course of the abutments to be 25 feet. {d) An iron girder bridge, 80 ft. span and 25 ft. wide, to carry ordinary road traffic. 5. What are the principal precautions required to ensure the durability and efficiency of a metalled road ? ORDINARY EXAMINATIONS, O.T. 1870. Ixxxiii

. Give a detailed description, accompanied with sketches, of any two of the following— (a) The Crumlin Viaduct. (6) The High-level Bridge at Newcastle, (c) The machinery of a modern screw steamer. (a7) A road traction-engine.

LAW.—PAET I.

Br Dobson.

1.- Define a consideration as sufficient to support a simple contract.

2. What is the rule as to the consideration for con­ tracts entered into by deed ? Distinguish between an illegal consideration, and no consideration, in cases where the contract is under seal.

3. Are there any cases in which contracts by deed must be founded upon a consideration ?

4. A agreed to print for B a work which was to con­ tain a dedication to be thereafter sent to A. A printed the work, but finding the dedication to be libellous refused to print it. B refused to accept what had been printed or to pay for it. Can A recover the price of his work and labour ? Give your reasons.

5. Write out the fourth section of the Statute of Frauds. Ixxxiv

6. What are the essential ingredients in a memo­ randum of an agreement under the fourth section ?

7. A goes into a shop and says (i.) " Let B have what goods he pleases to order, and if be do not pay you, I will" ; (ii.) " Let B have goods on my account." In which of these cases, if in either, is it necessary to have a writing within the fourth section ? Give reasons for your answer.

8. Where a consideration is executed, in what cases is a preceding request implied ?

9. Give an instance in which a promise is implied as well as a request.

10. Explain and illustrate the maxim " delegatus non potest delegare."

11. Who may be an agent to bind another by contract ? Are there any instances in which an agent can bind his principal although he could not bind himself?

12. Give the usual form of a bill of exchange, and a promissory note.

13. State what is the contract entered into by the drawer, the acceptor, and the indorser respec­ tively, with tho bona fide holder for value when the bill becomes due.

14. How far are an infant's contracts valid as against himself? Suppose, in a case in which he can bind himself, he gives his acceptance for the price of the goods; what is the effect ? ORDINARY EXAMINATIONS, O.T. 1870. IxXXV

15. Into what two classes are contracts of affreight­ ment divided ? State the usual contents of a bill of lading.

16. Suppose a bill of lading expressing that freight has been paid, contrary to the fact, gets into the hands of an assignee of the consignee, without knowledge that the freight has not been paid; has the ship a lien on the goods for the freight ? Give reasons for your answer.

17. What is the difference between municipal and trading corporations with respect to their power of binding themselves by simple contract ?

18. Describe the mode of forming a company under " The Companies Statute."

19. Both the fourth and the seventeenth sections of the Statute of Frauds permit the signature of the contract, or of the note or memorandum of the contract, to be made by an agent; must this agent be appointed by writing ?

20. Where man and wife are living together, upon what principle does it depend that he is bound by certain contracts which she makes in his name ?

L A W—PAST II. Mr. Billing. 1. Is a landlord deprived of his right to distrain by taking a bill or note for his rent ? Ixxxvi EXAMINATION PAPERS,

2. If a tenant from year to year underlets from year to 3'ear, has he such a reversion as entitles him to distrain for rent reserved on the underlease ? 3. What is the law now as to persons not parties to deeds taking the benefit of covenants in them ? 4. If a purchaser has notice at the time of his pur­ chase of a prior unregistered conveyance of the propertj-, will registration of his purchase deed give it priority over the unregistered conveyance ? 5. Suppose a tenant for life to make a lease for life generally, bow will it be construed ? Give the answer fully. 6. In what way does a donatio mortis causa differ from a legacy ? 7. What is the law as to how far writing is neces­ sary to declare a trust ? Give the answer fully. 8. If a person has power to charge an estate with a portion for his daughter, can he limit it to her separate use with a restraint on anticipation ? 9. What kind of covenant is a covenant to settle particular lands, and what is its effect ? 10. Where the Habendum of a deed gives a larger estate than the Premises, what does the grantee take? 11. How would it be if the Premises should give a larger estate than the Habendum ? 12. Where no time is limited for the performance of a covenant, within what time must it be performed ? ORDINARY EXAMINATIONS, O.T. 1870. IxXXvii

LAW. —PART III.

Mr. Billing.

1. Will parol evidence be admitted to show that an agreement not under seal was in the nature of an escrow ? 2. Will parol evidence be admitted in any case to show the consideration for a deed f Give the answer fully.

3. Suppose an infant sues by his guardian, are the ad­ missions of his guardian evidence against him ?

4. If a vendor who has contracted to sell real pro­ perty is unable to make title, can the purchaser recover the expenses of investigating the title under a count for money paid ?

5. In an action for "use and occupation, under what plea can evidence of eviction be given ?

6. Would a plea in an action by drawer v. acceptor of a bill of exchange traversing the drawing be good ?

7. Is leave of the Court or a Judge necessary in any cases to pay inonev into court? Give the answer fully.

8. Enumerate the cases in which by statute special particulars may accompany the writ.

9. In what instances is power given to the Court or a Judge to decide summarily in interpleader cases ? Ixxxviii EXAMINATION PAPERS,

10. Suppose a person were informed against for ob­ taining property under false pretences, and it should be proved that he obtained the property in question in such a manner as to amount to larceny, is he entitled to be acquitted ?

11. In proceedings before justices is it allowable to include several matters in one complaint?

12. What power have justices to adjudicate on claims to goods seized under their warrant ?

L A W.—PAST IV. Dr. Dobson. 1. Distinguish between express, implied, and con­ structive trusts.

2. What is the nature of the trust in the following case : Executors by mistake, but bona, fide, have paid legatees before a due discharge of all the debts ? 3. A person buys freehold lands and pays the pur­ chase-money for thern, but takes the conveyance in his own name and that of another person; what is the effect of this in Equity ?

4. Illustrate and explain the maxim " Vigilantibus non dormientibus equitas subvenit."

5. Explain what is meant by tacking, as applied to incumbrances. O.T. 1870. Ixxxix

6. Explain tho term "constructive fraud "as a ground for the interference of the Court in its Equity jurisdiction.

7. Under what circumstances, and on what grounds will Courts of Equity relieve persons who have entered into contracts whilst in a state of intoxi­ cation ?

8. State what is meant by " specific performance." What is the general rule on which Courts of Equity act when they decline to decree specific performance of contracts clearly proved ?

9. An agreement is entered into by a party incom­ petent to contract {e.g. an infant, married woman); will the Court enforce it cither for or against such party ? Give your reasons.

10. Does the Court in its Equity jurisdiction enforce the specific performance of any thing else but contracts?

11. Illustrate and explain the maxim " Equitas sequi- tur legem."

12. Mention some of the facts upon which the Court will fix, in order to hold that a given transaction is a mortgage only and not a sale.

13. Explain what is meant by the expression "Time being of the essence of the contract." What difference exists between Courts of Law and of Equity on this subject ?

14. Explain the terms " Equity of redemption " and " Foreclosure." XC EXAMINATION PAPERS,

15. Explain how it is that Courts of Equity have almost an exclusive jurisdiction in matters of account. 16. What is the law as to the appropriation of pay­ ments ? 17. What is meant by a "donatio mortis causa"? From whence does the English law derive the doctrine ? 18. In what way does a lien at Law differ from a lien in Equity ? 19. Explain what is meant by the term "marshalling assets." 20. What is the distinction between legal and equita ble assets ?

JDNIOE DESCEIPTIVE AND SURGICAL ANATOMY.

Professor Halford.

ORAL.

1. Describe the following bones :— The Occipital, the Radius, the Unciform, the Ischium, the Tibia and Internal Cuneiform.

2. Give the steps of a dissection of the muscles of the fore-arm and back of the leg, describing minutely the origin, insertion, and relations of any that may be required. ORDINARY EXAMINATIONS, O.T. 1870. xci

WRITTEN.

1. Mention the various kinds of joints.

2. Describe the ligaments connecting the following bones together—Lower Jaw and Skull, Axis Atlas and Occipital, Clavicle and Scapula, Ver­ tebra and Rib, Tibia and Astragalus.

3. Describe the muscles which are inserted into the great trochanter, into the phalanges of the hand, and into the spine, symphysis and crest of the pubes.

SENIOE DESCEIPTIVE AND SURGICAL ANATOMY.

Professor Halford.

ORAL.

1. Mention the necessary steps of a dissection of the axillary space and the sole of the foot. 2. How w-ould you proceed to expose the following parts of the Encophalon :—The fifth Ventricle, the Pineal gland, the island of Reil, the Hippo­ campus major, Corpora quadrigemmina, and the anterior Commissure.

3. Having removed the spinal cord contained in its membranes how would you proceed to dissect it ? EXAMINATION PAPERS,

WRITTEN.

1. Describe the process of development for the follow­ ing bones :—The Occipital, Frontal, Sphenoid, Humerus, Sternum and Femur.

2. Give the origin, course, and relations of the follow­ ing blood vessels:—Arteries: Phrenic, Spermatic, Axillary, Internal Carotid and Posterior Tibial. Veins: Internal and External Jugular, Radial, Internal and External Saphena, the Portal and Umbilical.

3. Describe the formation and situation of the Cer­ vical, Lumbar and Sacral plexuses of Nerves, enumerating the branches in the order in which they are found by dissection.

GENEEAL ANATOMY, PHYSIOLOGY, AND PATHOLOGY.

TIIIUD AND FOUUTH VEAKS. Professor Halford. 1. Describe the minute structure of Cartilage, Bone, and Muscular Fibre.

2. Describe the various forms of Epithelium, and the sources from which they are obtained.

3. Give a sketch of the Lymphatic and Lacteal Vessels and Glands, and in addition the source, nature, destination, and uses of their contents. ORDINARY EXAMINATIONS, O.T. 1870. XClil

4. Describe the mechanism of the Heart's action.

5. What arc the microscopic appearances of Cancerous and Recurring fibroid tumours.

CHEMISTEY (MEDICAL).

John Drummond Kirkland.

\_N.B.—In describing the preparation of substances give the equations used to explain the various re­ actions. Either the old or the new notation may be used.]

1. Give the principal properties, physical and chemical, of the "Nitrogen Group" of elements.

2. Select any one of the above and describe the pre­ paration and properties of its principal compounds with oxygen and with hydrogen.

3. Describe Marsh's method for the detection of arsenic and antimony—mentioning the sources of error to be avoided.

4. Describe the preparation of the following, viz.:— i. Oil of Vitriol ii. Sulphide of Ammonium in. Glucose iv. Urea (artificial) v. Hydrocyanic Acid. XC1V EXAMINATION PAPERS,

5. Describe the appearance under the microscope of the following starches, viz.:—Rice, Potato, Sago, with sketches showing their relative size and re­ spective shape.

6. Define the following terms, and give one or two examples in each case, viz.:— i. Element. Organic Radicle. II. Allotropic. Isomorphous. in. Atom. Molecule. 7. Give the generally received theory of Etherifi- cation.

8. Briefly describe the method adopted for the ulti­ mate analysis of an organic compound containing Carbon, Oxygen, and Hydrogen.

PEACTICAL CHEMISTEY.

John Drummond Kirkland.

\N.B.—The means by which the several results were obtained must be written out. Any calculations necessary to be given in full.]

1. Qualitatively analyze any one of the substances marked respectively A, B, C.(o)

2. A substance weighs (75) seventy-five grains in air,

(a) LeuJ Acetate. Calcic Sulphate. Sodium-Carbonate. ORDINARY EXAMINATIONS, O.T. 1870. XCV

and (67) sixty-seven grains in water, what is its specific gravity ? A cubic foot of water weighing (1000) one thousand ounces, what would be the weight of a cubic foot of the above substance ?

3. Examine the sample of Urine marked D for albumen and glucose. Mention any sources of error in the processes ordinarily used.

4. Qualitatively analyze the solution marked E.(6)

MATEEIA MEDICA, THEEAPEUTICS, AND MEDICAL BOTANY.

S. D. Bird, M.D.

1. What are the properties, officinal preparations, and general therapeutics of Acetate of Lead ; what morbid states are induced when it is taken for a long time ?

2. Whence is Bromine obtained ? Give its proper­ ties, state in what form it is usually prescribed, and for what diseases, in what doses.

3. Give the principal Alkaloids in Opium, and the formula for Liq : Morph : Mur : and Liq: Morph : Acet: Contrast the action of Morphia and Opium as narcotics.

4. From what natural orders of plants do we obtain

(6) Mercuric Chloride. XCV1 EXAMINATION PAPERS,

Olive Oil, Strychnia, Scamraony, Rhubarb, Quina, Matico, Cannabis Indica, Scilla, Veratrum Viride, and Copaiba ?

5. Describe Kino, its appearance, properties and composition. Whence is it derived ? Give its officinal preparations and its therapeutics.

6. Give the doses of Pulv: Jalap: Co, Pulv: Ipecac Co, Tinct: Chlorof: Co: Liq: Potassoj, Liq Atropine, Tinct : Verat: Virid : Digitalis, Ex Cannab: Indie: Creasotum, Extract: BeltB liquid Morphia (hypodermically).

7. Whence is Podophylline obtained ? Describe its action as a purgative and au alterative, with doses, combinations, and precautions.

8. Define and enumerate Emraenagogues.

9. Write in full a prescription for Chlorosis, Spasmo­ dic Asthma, Cardiac Dropsy, and the Vomiting of Pregnancy.

10. Which drugs dilate the pupil and which contract it?

SUEGEEY.

Edwd. Barker, M.D., F.R.C.S.Eng.

1. Describe the different forms of mortification affect­ ing an external part, the causes, the condition of the structures involved, and the treatment. ORDINARY EXAMINATIONS, O.T. 1870. XCvil

2. What is the usual cause of fracture of tho lower end of the Radius 1 Describe the deformity pro­ duced by the fracture, and the proper method of treatment. What is the common result of un­ skilful management ? 3. Enumerate briefly the characters and the appro­ priate treatment of burns of different degrees of severitv. 4. State the circumstances that would render the operation of trephining the skull advisable or necessary, including those which are the direct result of the accident and those occurring soon after the accident or at a later period. 5. Mention the different kinds of wounds wliich may arise from accident or any other violent causes. Give the treatment local and general of each variety. 6. Give the various dislocations of the shoulder joint, the symptoms, the appearances of the limb under each variety, with the means of reduction; and when unable at the first attempt to replace the head of the bone, describe the steps which are to be taken to ensure a more successful issue.

THEORY AND PRACTICE OF MEDICINE.

FOURTH YEAH. James Robertson, M.A.^ M.D. 1. Define the terms diathesis and cachexia, distinguish­ ing them from each other. XCV111 EXAMINATION PAPERS,

2. What is understood by congenital and hereditary, as applied to disease ? Illustrate by examples and explain what is meant by atavism.

3. Explain the meaning of the terms symptoms and signs; also of pathognomonic, diagnostic, objective, subjective, positive, negative, idiopathic, and sym­ pathetic, as applied to symptoms or signs.

4. State the rules or laws that distinguish the diseases included under the term Exanthemata.

5. Describe the different stages of variola, its varieties and most common complications. Distinguish between variola, varioloid, and varicella.

6. Diagnose between Scarlatina and Measles.

7. Give the classification of continued Fever now ge­ nerally adopted.

8. Describe the form of Fever prevalent in this Colony, noticing its characteristic symptoms, duration, and most common complications, and indicating the general treatment, medicinal, dietetic, and re­ giminal.

9. Describe the pathology of Diphtheria, and notice its symptoms, sources of danger, and sequela. Indicate the appropriate treatment. ORDINARY EXAMINATIONS, O.T. 1870. XC1X

MIDWIFERY AND DISEASES OF WOMEN AND CHILDREN.

Richard T. Tracy, M.D., L.R.C.S.I.

1. Describe the axes and measurements of the brim, cavity, and outlet of the standard female pelvis.

2. What changes occur in the ovaries and uterus during each period of healthy menstruation?

3. Enumerate the various symptoms that usually occur during the first three months of pregnancy.

4. What is Abortion ? Describe how you would treat such a case, both during the early symptoms and through all its stages, if unable to prevent it.

5. Describe the different cases of labour in which it is advisable to apply the forceps—the mode of ap­ plication and use from first to last.

6. How would you treat a case of chronic uterin leucorrhcea ? HONOUR EXAMINATIONS.

JUNIOE GEEEK. Professor Irving.

Translate into Greek— This was the end of the famous Trojan war. The Greeks had attained their object; their ven­ geance was satiated; the treasures of Troy had become their booty; Menelaus was restored to the possession of his wife; but most of the leaders of the army had been slain; part of tho survivors, whose lot it was, after long absence, to return to the land of their fathers, found their dominions in the hands of hostile tribes; Agamemnon himself fell, through the domestic treason of his wife, Clytemnestra; they had become strangers in their own homes, and within a" few ages most of the reigning houses that had brought the Trojan kingdom to ruins, were extinct. Princes and people (says the Scythian Anacharsis) should learn from this, that even victory is to be feared.

Translate— Et ovv fiE, oirep EITTOV, iirl rovToig dtj>loiTE, u-iroifi av V/MV, on 'Eyw v/idg, ]va7ot, daird- (ofj.ai fiiy Kal iptXti, TrEtcro/jai BE /idXXov r£ OEW t) vfxiv, Kal eweyirep av E/.nrviu> Kal oiog TE <3, oi fir) iravirii)ftai ipiXoaofoiy Kal vfj.lv TrapaKEXEvofievog TE Kal ivBeiKvvixevog OTW CIV del tiruy^aj'to vfxiov, Xiywv HONOUR EXAMINATIONS, O.T. 1870. Cl

oiatrEp E'iioBa, on, il apioTE dvBpwv, 'Adt]va~ing &v, trSXEwg Tfjg fiEylarrfg Kal evBoKt/xwrdrvg Eig aoipiav Kal \try/vv, xpijfiaTMV fxev OVK aiirxyvei iiriHEXoifiEvog, owiog aoi Etrrai ihg TrXstora, Kal Bo^g Kal Tijxiig, ippo- VJ/OEWC <5E ical dXrjfWac Kal Tijg \pvyrijg, Smog i>g osXr/oTij carat, OVK ETTI^EXET. OUSE cppovn^eig; Kal idv Tig v/xwv dfiipigflnriiirr) Kal cpij ivi/ieXeludai, OVK ehdvg d, Tron'iiTU), Kal £,EVU> Kal dirrH, fidXXov BE rolg dtrroig, oaif ftov iyyvripix) ierrE yevEt. ravra yap KEXEVEI b 0Eog,

3. Translate— "Ou»: 'iBov, ov TrvQofxrjv, dXXo crrbvov oTov aKOvaa KTEivo/iEywv' fifielg Be /ifX'"-" OaXdfitov EviriiKrwv H/IE6' drv^ofXEvai, cravlSeg B' i\ov ev dpapv'iai, Hpiv y STE S)/ fie abg vlbg dirb fiEydpoio KOXEUUE Tt]Xifiay(og' rov ydp pa TTarijp irpOEr/KE KaXicrirat. JLvpov tTTEir' OoWjja fUTa KTajXEvomi VEKvaaiv Earaod ' ol BE fiiv dfiepl, KparalireSov ovBag Ey(OVTcg, Kfiar in dXXijXoicnv' iBovcrd KE dvfxbv idvdrjg, Alfxan Kal XvQpio ireTaXay/jiEvov &g TE Xiovra. "Svv B oi fiiy B>) Trdi'reg iir ahXEhjiri dvpTjaiv Adpooi, ahrdp b Bw/ja dEEiovrai irEpiKaXXig, Wvp fieya Krjd/.tei'og' ere BE fiE irpoirfKE KaXiaaai, - AXX ETZEV, otppa mpSiv iiitppoirvvng eTrifliJTOv A/iipoTepu) ipiXov i)rop, itrel KaKa iroXXd TVETtoade. NOv <5' ijBrj TOBE fiaKpbv ieXBwp EKTETEXearai' HX6"E fiev ahrbg £wog iecmog, evpE BE Kal ai Kal TralS' iv fieydpoiai' KaK&g B' ol nip jxiv Epei^ov MyrjcrrijpEg, rovg Trdvrag irlaaTO if ivl O'IKU)," Cll EXAMINATION PAPERS,

4. (a) Prove and explain this statement—" For the exact recognition of a word of the Consonant De­ clension not only the Nominative but the Genitive is necessary." (6) State and illustrate further the Euphonic prin­ ciples involved in the following forms—XvOrjn, Eijil, irdijy(iii, Terayjxai. (c) Construct examples to prove that Greek uses participles more freely than does English. {d) Discuss fully the various uses of the Optative Mood in Greek, and give examples.

5. "AXXny fiev apyjiv ohBefiiav i)p^a, ipovXevira BE. Ex­ plain the constitution of the Athenian senate, and give an account of the incident to which Socrates refers.

6. KvXXijnog. To what deity is this epithet given and why ? Mention any other local names of the Greek Gods, and give the reasons for their use.

7. Why is it supposed that certain words were pro­ nounced in Homer's day with an initial Digamma ? Name any such and quote lines in support of your view.

8. {a) Discuss and illustrate the formation of dTroTTE- tpEvyr], ETriarpoipdctfy, Epe/ivbg, icrriifii, KpEtcawv, aeXvvrj (6) and the meaning and derivation of dyavam-ETi', a£iog, EKWV, Xio'iov, irbaig, £vvwpig. HONOUR EXAMINATIONS, O.T. 1870. Clll

JUNIOE LATIN. Professor Irving. 1. Translate into Latin— This was the end of the famous Trojan war. The Greeks had attained their object; their ven­ geance was satiated; the treasures of Troy had become their booty; Menelaus was restored to the possession of his wife; but most of the leaders of the army had been slain; part of the survivors, whose lot it was, after long absence, to return to the land of their fathers, found their dominions in the hands of hostile tribes; Agamemnon himself fell, through the domestic treason of his wife, Clyteranestra; they had become strangers in their own homes, and within a few ages most of the reigning houses that had brougnt the Trojan kingdom to ruins, were extinct. Princes and people (says the Scythian Anacharsis) should learn from this, that even victory is to be feared.

2. Translate into idiomatic English— Stabat acuta silex, prtccisis undique saxis Speluncte dorso insurgcns, altissima visu, Dirarum nidis domus opportuna volucrum. Hanc, ut prona jugo kevum incumbebat ad amnem, Dexter in adversum nitens concussit, et imis Avolsam solvit radicibus; inde repente Inpulit; inpulsu quo maxumus intonat sether, Dissultant ripae refluitque exterritus amnis. At specus et Caci detecta adparuit ingens Regia, et umbrosse penitus patuere cavernae : Non secus, ac si qua penitus vi terra dehiscens Infernas reseret sedes et regna recludat CIV EXAMINATION PAPERS,

Pallida, dis invisa, superque inmane barathrum Cernatur, trepidentque inmisso lumine . Ergo insperata deprensum in luce repente Inclusumque cavo saxo atque insucta rudentem Desuper Alcides telis premit, omniaque arma Advocat, et ramis vastisque molaribus instat. Hie autem, neque enim fuga jam super ulla pericli, Faucibus ingentem fumum, mirabile dictu, Evomit involvitque doinuin caligine creca, Pros|)ectum eripiens oculis, glomeratque sub antro Furaiferain noctem commixtis igno tenebris. Non tulit Alcides animis, seque ipse per ignem Prrecipiti jecit saltu, qua plurimus undam Furaus agit nobulaque ingens specus asstuat atra. Hie Cacura in tenebris incendia vana vomentem Corripit in nodum conplexus, et angit inhasrens Elisos oculos et siccum sanguine guttur.

3. Translate into idiomatic English— Atque hfec omnia sic agentur, Quirites, ut res maxima? minimo motu, pericula summa nullo tu- multu, helium intestinum ac domesticum post hominum memoriam crudelissimum ac maximum, me uno togato duce et imperatore, sedetur: quod ego sic administrabo, Quirites, ut, si ullo modo fieri poterit, ne improbus quidem quisquam in hac urbe pcenam sui sccleris suft'erat. Sed si vis manifestre audaciae, si irnpendens patrire periculum me necessario de hac animi Icnitate deduxerit, illud profecto perficiam, quod in tanto et tarn insi- dioso hello vix optandum videtur, ut neque bonus quisquam intereat, paucorumque poena vos jam omnes salvi esse possitis. QUIB quidem ego neque mca prudentia, neque humanis consiliis fretus pol- liceor vobis, Quirites, sed multis, et non dubiis deorum immortalium significationibus, quibus ego HONOUR EXAMINATIONS, O.T. 1870. CV

ducibus in hanc spem sententiamque sum ingres- sus : qui-jam non procul, ut quondam solebant, ab externo hoste atque longinquo, sed hie prassentes suo numine atque auxilio sua templa atque Urbis tecta defendant: quos vos, Quirites, precari, ve- nerari atque implorare debetis, ut, quam urbem pulcherrimam, florentissimam potentissimamque esse voluerunt, hanc, omnibus hostium copiis terra marique superatis, a perditissimorum civium ne- fario scelere defendant.

4. Give a brief account, with dates, of the four Speeches against Catiline.

5. Name with approximate dates and a brief notice of their writings six of the Latin writers of the Golden Age other than Cicero and Virgil.

6. Sketch the general uses of the Latin Subjunctive mood, and the special ones of its tenses.

7. Draw a rough map of the country round Rome shewing the principal places named in the seventh and eighth books of the JEneid.

8. (a) Discuss and illustrate the formation of these words—collega, fortuna, fulmen, nefarius, prox- imus, popina, publicus, sublimis. (6) and the meaning and the derivation of requor, cerebrum, illustro, imago, lorica, machinor, palma, poena.

e3 CV1 EXAMINATION PAPERS,

ENGLISH AND LOGIC—PAET I.

Professor Irving.

1. Analyse fully the following sentence— I have no feelings but those of gratitude and reverence for the man who did what he did when he did: and a sort of shame, that any one should repine against taking a fair share of such treat­ ment as the world thought good enough for him.

2. Put into modern English and comment generally upon the following— Thil onsendon hi him micelne here t6 fultume, and s6na thiis the hi on this ealond c6mon, thii compedon hi vidh heora feondum and hi micel vill ongeslogon and of heora gemiirum Adrifon and aflymdon.

3. Also on the following— To prey ere and to penaunce Putten heom monye, For the love of UT Lord Liveden ful harde In hope for to have Ilevenriche blisse; As ancres and as hermytes That holden hem in heore celles Coveyte not in cuntre To carien aboute, For non likerous lyflode Heore licam to plese.

4. When two mutes of different character come to­ gether in a word the general law of Language is to assimilate them. English however presents one remarkable exception to this law. Illustrate fully both law and exception. O.T. 1870. CV11

6. Give some account with dates of these authors and their works Beda, Ormin, Jonson, Mandevil.

6. Derive fully and illustrate the formation of these English words—alms, country, double, down, gleam, lord, shear, should, swoon, tavern.

7. Explain clearly how it is that there are so very few terms which can strictly be called Univocal.

8. "Every judgment is an attempt to reduce to unity two cognitions, or an expression that two notions can or cannot be reconciled." Explain and criti­ cize this account. What definition of a judgment would you adopt, and why ?

9. Every Conjunctive hypothetical judgment which contains three terms only may be reduced to a Categorical containing two only out of the three terms. Prove this by examining every possible case.

10. Explain the views of Logic severally taken by Whately, Thomson, and Mill.

11. State the Lawrs of Opposition between Contradic­ tories and that of Subaltern Opposition. Assuming these as self-evident deduce the other two.

12. Construct carefully according to Mill's view of definition, definitions of the following terms— canal, machine, picture, wages. CV11] EXAMINATION PAPERS,

ENGLISH AND LOGIC—PABT II.

Professor Irving. SHAKESPERR, . MILTON, Paradise Lost, Book II. SPENSER, Faery Queen, Book I. RHETORIC. LOGIC. 1. Sketch the character of Brutus as delineated in the Julius Caesar. 2. What is the general history of the Text of Shakespere ? On what grounds are conjectural emendations admissible ? Mention a few which you consider necessary in the Text of the Julius Ccesar, and give your reasons. .3. Explain the etymology of these words—(a) errand, place, ransom (6) fiend, outrage, sorcery (c) gossip, housling, welkin. 4. In what peculiar senses do you find the following used ? Quote if you can passages in which they occur—(a) exorcist, proof, physical (6) alchemy, deform, wards (c) graine, recreaunt, sacred. 5. Explain fully— (a) All that he can do Is to himself, take thought, and die with Ccesar. (6) You know, it is the feast of Lupercal. {c) The wealth of Ormuz or of Ind. {d) Gorgons and Hydras and Chima;ras dire. (e) Help then, 0 holy virgin chiefe of nine, Thy weaker novice. {f) Theseus condemned to endless slouth by law, And fifty sisters water in leke vessels draw. HONOUR EXAMINATIONS, O.T. 1870. CIX

6. Discuss the following passage from a Rhetorical point of view— 0 progeny of heaven, empyreal thrones ! With reason hath deep silence and demur Seized us, though undismayed. Long is the way And hard, that out of hell leads up to light; Our prison strong; this huge convex of fire, Outrageous to devour, immures us round Ninefold ; and gates of burning adamant, Barred over us, prohibit all egress. These passed, if any pass, the void profound Of unessential night receives him next Wide-gaping, and with utter loss of being Threatens him, plunged in that abortive gulf. If thence he 'scape into whatever world, Or unknown region, what remains him less Than unknown dangers, and as hard escape ?

7. Discuss and exemplify the following—(a) Argu­ ment from Example (6) Metaphor (c) Antithesis.

8. "In the proposition all equilateral triangles are equiangular, the predicate is distributable not distributed: if I say all equilateral triangles and no other are equilateral, I am asserting not one proposition, but two." Discuss this dictum, and the opposed view.

9. Whatisrneantby the Fourth Figure ? On various grounds is it retained, and on what various grounds has it been condemned ?

10. What do you consider the Typical Form of an Inductive Argument ? Can it be brought under any Syllogistic Form ? CX EXAMINATION PAPERS,

11. Name and characterize briefly the various opera­ tions enumerated by Mill as Subsidiary to Induc­ tion.

12. What is'the ground of our belief in Axioms? State and criticize Mill's answer to this question.

GEOMETRY AND TEIGONOMETEY.

Professor Wilson.

1. Assuming the truth of the formulas for sin {A +B) and cos {A + B) when {A + B) is less than a right angle extend them to angles of any magni­ tude and explain carefully the general mathemati­ cal principle made use of.

2. Shew that -i-i -ii -ii -ii tan i + tan ~ + tan -i + tan i = 45° o o 7 o

3. In any triangle shew that sin {A - B) _a2 - b2 sin {A + B) c2

4. W rite down the expressions for the areas of poly­ gons inscribed in and circumscribed about a given circle and deduce tin expression fur the area of the circle. HONOUR EXAMINATIONS, O.T. 1870. Cxi

5. Investigate the method of expressing sin"0 in a series of sines of multiples of 6; and find the first three terms and the last term when n is of the form 4?n + 1.

6. Investigate the expressions for sin 6 and cos 6 in series of ascending powers of 6 and deduce the exponential expressions for sin 0 and cos 0.

7. Investigate the distinguishing property of the curve in which the surface of a cone is cut by a plane oblique to the axis.

8. Shew that in any conic section the portion of the tangent between the directrix and the point of contact subtends a right angle at the focus.

9. If from any point tangents are drawn to a parabola the diameters through the points of contact are equidistant from that point, and the diameter through that point bisects the chord of contact.

10. In the ellipse shew that FN2 : AN. NA' ::BC* :AC*

11. The perpendiculars from the foci on the tangent to an ellipse meet it in the circumference of the circle whose diameter is the axis major, and the rectangle contained by them is equal to the square on the semi-axis minor.

12. If any straight line cuts the hyperbola and its asymptotes the portions intercepted between the curve and the asymptotes are equal to one another. CXI1 EXAMINATION PAPERS,

ALGEBEA. Professor Wilson. 1. Find the relation which must exist amongst the coefficients of the equation Ax2 + Bxy + Cy2 + Dx + Eg + F= 0 in order that it may give two real values of y for real values of a; between certain limits, and impos­ sible values of y for all values of x outside those limits. 2. Eliminate m from the equations y = mx + •JmPa* + b2 and

x /ai y = -m + V^ + b2

3. Shew that if — is a fraction nearly equal to unity a the square root of 0 will be obtained approxi mately by halving the difference between a and 6. 4. Solve the equations x* + y* = 14:X2y'i x + y = a 5. Solve the equations 1 1 1 y + z = - z + x =*- x + y = —

int0 ifs artial 6. Resolve (cT _ ]) ix__ 2) ix - 3) P fractions. HONOUR EXAMINATIONS, O.T. 1870. CS111

7. Investigate the form of the continued product of n simple binomial factors and deduce the bino­ mial theorem for a positive integral index.

8. Find the middle term of the expansion of ( m n n n, m \imn 0~" . X "> — 0«" T I 9. Investigate the expansion of log (1 -f- a-). What is el

10. If Sn is the sum of n terms of a given geometrical progression find the sum of Sv -j- S2 + .. . -\- S„

11. There are seven boat clubs of which one has four boats three others have two boats each and the remaining three have only one each ; find an ex­ pression for the number of ways in which a list can be formed of the thirteen boats, observing that the second boat of a club cannot be before the first &c.

12. Shew that „ Cr + „ Cr _ , = „ + l Cr

NATUEAL PHILOSOPHY.

Professor Wilson.

1. Every particle of a thin spherical shell attracts according to tho natural law ; shew that a particle will be in equilibrium at any point in the interior. cxiv 2. A heavy uniform rod of length I and weight TFis placed in equilibrium across a rough horizontal cylinder of radius a; it is then loaded at one end with a weight W and assumes a new position of equilibrium; find what angle it will make with the horizontal If the length of the rod is equal to the circumference of the cylinder and its weight 10 lbs., find the weight with which it must be loaded that tho angle may be 30°.

3. Describe the common balance; and calculate the angle at wliich the beam is inclined to the hori­ zontal when the weights in the scale pans are slightly different.

4. State the second law of motion ; shew from this law that when a body moves in a circle with uniform velocity the resultant of the forces acting on it must pass through the center of the circle; and calculate its magnitude.

5. Find the resultant of two parallel forces acting in the same direction at different points of a rigid body;

6. What is meant by the center of oscillation of a compound pendulum ? and by the statement that the centers of oscillation and suspension arc recip­ rocal ? Explain the principle of Captain Hater's pendulum.

7. Supposing the specific heat of iron to be -114 and its melting point 3300° F.; calculate the velocity with which a mass of iron at 50° P. must be mov­ ing that the conversion of its motion into heat should be just sufficient to raise it to the melting point. HONOUR EXAMINATIONS, O.T. 1870. CXV

8. Explain the reason of the difference between the specific heat of air at a constant volume and under a constant pressure. 9. Describe the jet pump and explain its action. 10. Explain the essential points of Watt's invention for the improvement of the steam engine. 11. Describe the general characteristics of the daily range of the barometer; what cause has been assigned for this phenomenon ? Explain its opera­ tion. 12. Explain the principal causes of the lower tempera­ ture in the upper regions of the atmosphere.

CHEMISTRY AND MINERALOGY.

Professor McCoy.

1. Write down the metalloids in the order of their electro-chemical powers. 2. What is a Tetrad ? Write down all the elements to which the term may apply. 3. Explain Dulong and Petit's law of the relations between Specific Heat and Atomic Weight, giving au example in detail. 4. Describe all the prisms having two optic axes. 5. What are the interfacial angles of the Octahedron, Tetrahedron, Dodecahedron ? CXV1 EXAMINATION PAPERS,

BOTANY.

Professor McCoy.

1. Describe all the parts of a seed, with the chief va­

riations in each.

2. Describe all the modes of Cytogenesis.

3. Give examples of plants affording as maiw different non-nitrogenous cell-contents as vou can. 4. Describe the structure and point out the differences of each of the great divisions of Thallogens. 5. Describe as many of the recognized kinds of Ovule as you can.

ZOOLOGY.

(VEETEBBATA.)

Professor McCoy.

1. Describe each of the bones of the carpus and tarsus, with their articulations in the higher Mammalia.

2. Enumerate the bones of the Nasal segment of the skull of a bony fish, giving the relations and ho­ mologies of each.

3. Characterise the chief Families of Snakes. HONOUR EXAMINATIONS, O.T. 1870. CXV11

4. Characterise the chief families of Plagiostouiatous Fishes.

5. Describe as fully as you can the circulation in Fishes.

ANCIENT HISTORY.

Professor Heam.

1. What event does Dr. Mommsen regard as the turning point at which the Protectorate of Rome over its subject foreign communities degenerated into a tyranny ?

2. Describe the three leading law reforms effected by Sulla.

3. How does Mr. Merivale account for the apathy of the Romans during the Second Civil War? What was " the last spontaneous levy of the Roman People"?

4. What were the principal acts done by Augustus in his capacity as Pontifex Maximus 1

5. When and in what circumstances was a perma­ nent maritime force established at Rome? Where were the principal naval stations ?

6. Why was the assimilation to Rome of the Western part of the Empire more close than that of the Eastern part ? CXV1U EXAMINATION PAPERS,

7. Dr. Mommsen claims for Caisar the honour of having originated two great branches of modern legislation. What were they ?

8. What were the primitive political unities of early Italian history ? Give some account of their

9. What was meant by the Edictum Perpetuum ? By what other name was it known and what was the origin of that name ?

10. State and explain the Roman definition of an In­ heritance.

11. State and explain the Roman definition of an Ob­ ligation.

12. On what grounds does Dr. Mommsen contend that Roman Constitutional Law was essentially of native growth ?

HISTOEY OF THE BRITISH EMPIRE.

Professor Hearn.

1. On what grounds does Mr. Hallam contend that in questions concerning the ancient Germans the authority of Tacitus is not conclusive ?

2. " The Thegns supplant the old Eorls." Explain the nature and the process of the change indicated bv these words. HONOUR EXAMINATIONS, O.T. 1870. cxix

8. How far is the term " Anglo-Saxon " appropriate as descriptive of the period before the Norman Conquest?

4. What was meant by the " trinoda necessitas," and to what cases did it apply ?

5. What is said to have been the revenue of William the Conqueror ? What are the reasons for doubt­ ing the accuracy of this statement ?

6. What were the causes of the migration of the Normans into Scotland ?

7. What were the essential characteristics of com­ munes, and how did they differ from boroughs ?

8. Shew how the Ceorl of Alfred differed from the Villein of Glanville.

9. How does Mr. Hallam account for the assumption of the title "Basileus" by English Kings prior to the Conquest ?

10. What was the last unequivocal testimony to the existence of Villenage in England ?

11. What historical value does Mr. Hallam attach to the poetry of the Troubadours ?

12. What does Bolingbroke describe as " the era of hereditary right" in England? Why was that doctrine at that time put prominently forward ? CXX EXAMINATION PAPERS,

LAW.—PABT I.

Dr. Dobson.

Cite any cases which you may remember as authorities in support of your answer.

1. What is the law with reference to contracts in restraint of trade ?

2. Under what circumstances will courts of law enter into the question of adequacy of consideration ?

3. A declaration alleged that, in consideration that the plaintiff had bought a horse from the defend­ ant at a certain price, the defendant promised that it did not exceed five years old and was sound. Upon what principle did the court hold that the declaration disclosed no cause of action ? Roscorla v. Thomas, 3 Q.B. 234.

4. What is the difference between executed and exe­ cutor}7 contracts ? What is it necessary to prove before a contract founded upon an executed con­ sideration can be enforced ?

5. Give an instance of merger of a contract.

6. An action was brought upon an agreement between the executor of A B and the widow of the tes­ tator, which set out that the testator had orally declared his wish that his widow should enjoy certain premises for her life, and that it w;is agreed, in consideration of such desire, that the HONOUR EXAMINATIONS, O.T. 1870. CXxi

executor should convey them to the widow, pro­ vided she would pay £1 towards the ground-rent of those and certain other premises, and keep the premises conveyed in good repair. Was this action maintainable ? Give your reasons.

7. A writes to B "I will buy the horse you were driving to day for £40 if you will warrant him sound and quiet in harness." B answers, " I accept your offer, and I hereby warrant him sound in double harness." Is any binding con­ tract disclosed ? Give your reasons. 8. Distinguish between a general and a particular agent, and state the difference w7hich exists in their power respectively to bind their principals. 9. What is the difference between factors and brokers ?

10. As a general rule corporations can only bind them­ selves by instrument under their common seal; mention the principal exceptions to this rule. Does the nature of the corporation itself affect the rule? 11. What is the law as to the power of (i.) infants, (ii.) married women, (iii.) lunatics, and (iv.) in­ toxicated persons, to bind themselves by contract? How has class (ii.) been affected by recent legis­ lation ? 12. Define a bill of exchange and a promissory note. What is the effect of an indorsement " by procu­ ration " ? 13. Mention the usual stipulations contained in a charter-party. / CXXU EXAMINATION PAPERS,

14. Explain the terms average, general average, par­ ticular average, and demurrage.

15. Under what circumstances can the consignee of goods under a bill of hiding give to a third person greater rights than ho himself possesses against the ship ?

10. Upon what principle does it depend that one part­ ner can bind the firm by entering into contracts without the knowledg-e of the other members but within the scope of the partnership business ?

17. At common law what was the liability of a common carrier towards the owner of goods entrusted to his care to be carried ? To what extent has the legislature restricted this liability ?

18. If the drawer refuse to accept, what rights does the payer of a bill of exchange possess ? What is acceptance for honour ?

19. What is the law as to the ratification by persons of full age of contracts entered into by them durante miiiorc .rotate ?

20. Write out the fourth and the seventeenth sections of the Statute of Frauds ? HONOUR EXAMINATIONS, O.T. 1870. CXxill

LAW.-PAM II.

Mr. Billing.

1. Suppose a man by deed grant to A B certain lands therein mentioned, what estate does A B take ? 2. A lease is made for twelve months from the 1st January, 1870. Another lease is made for " a twelvemonth" from the 1st January, 1870. Is there any difference between them ?

3. A man makes a lease to another for so many years as A B shall name. Is it valid ?

4. Suppose it were for so many years as A B shall live, is it valid ?

5. A man who has the reversion of a property in fee marries a woman who is tenant for years of it, what is the law as to merger in the case ?

6. What is the rule in Shelley's case ?

7. Suppose a tenant for life commit legal waste by felling timber, what remedies has the remainder man ? Give the answer fully.

8. If a contract for the sale of land be rescinded through want of title in the vendor, and the pur­ chaser has paid part of the purchase money, has lie any lien on the land for it ?

9. What is the period of limitation for recovering monej' charged on land ? /2 CXX1V EXAMINATION PAPERS,

10. What is the period of limitation for recovering a legacy ?

11. What power has the Court to order the sale of timber growing on settled estates ? Is there any exception ?

12. Is property in the hands of a trustee liable to an execution against the cestui que trust ? Give the answer fully.

L A W.—PART III.

Mr. Billing.

1. When a plea containing a defence arising after the commencement of an action is pleaded with a plea of a defence arising before its commence­ ment, what course may the plaintiff take ?

2. What is the law as to security for costs in an action against the Sheriff for any thing done by him in the intended execution of his duty ?

3. Under what plea is a privileged communication given in evidence in an action for Libel ?

4. Is it a good plea to au action that the plaintiff entered into a covenant with the defendant not to sue for a limited time ?

5. Suppose a man executed a deed in a wrong name, how should he be sued ? HONOUR EXAMINATIONS, O.T. 1670. CXXV

6. If an infant is joint executor with others, is he at liberty to join with them in suing by attorney ?

7. If a declaration contain a count against A B as executor, and a count against him in his own right, what course may the defendant take ?

8. If Executors who should be joined as defendants are not joined, how is the nonjoinder to be taken advantage of?

9. Under what plea is the defence of lien raised in . an action of conversion ?

10. In an action for having knowingly kept a mis­ chievous animal w-hich injured the plaintiff, what is put in issue under not g"uilty ?

11. What is the law as to materiality of evidence in cases of perjury ?

12. Suppose a person is tried as an accessory, can he be afterwards prosecuted for the same offence ?

JUNIOR DESCRIPTIVE AND SUEGICAL ANATOMY.

Professor Halford.

1. Omitting the atlas and axis, what are the characters of the typical and peculiar Vertebra? and Ribs ?

2. Describe minutely the origin, insertion, and relations CXXV1 EXAMINATION PAPERS,

of the following muscles:—Scalaeni, Quadratus lumborum, Circumfloxus palati, Internal Ptery­ goid, Levator palati, Stylo-Pharyngeus, Middle Constrictor of the Pharynx, Adductor Brevis, Popliteus, Complexus and Splenius.

3. Describe minutely the articulations of the Tarsus, Hip and Elbow.

SENIOR DESCRIPTIVE AND SURGICAL ANATOMY.

Professor Halford.

1. Describe minutely the Pelvic, Perinatal, Trans- versalis, and Inguinal Fascite, showing how an exact knowledge of them is necessary to the surgeon.

2. The anterior walls of the Thorax having been re­ moved, and afterwards the Heart and Lungs (the roots of the lungs remaining), describe the relative position of the structures (organs, vessels and nerves) seen from the apex to the base of the Thorax.

3. Describe minutely the origin, course, distribution and connections of the following nerves:—Nasal, Vidian, Chorda tympani, Superior Laryngeal, Cervical, Sympathetic and Pudic.

4. Describe the origin, course and relations of the Iliac Veins. HONOUR EXAMINATIONS, O.T. 1870. CXXvii

GENERAL ANATOMY, PHYSIOLOGY, AND PATHOLOGY.

Professor Halford.

•1. What are the various modes by which external im­ pressions are converted into muscular action ?

2. W?hat are the electrical phenomena manifested in living bodies; what are their sources, and what are the best means for exhibiting them ?

3. Show how far the physical forces explain the pheno­ mena of life; where they seem to fail and another force (Vital) to be necessary.

CHEMISTEY (MEDICAL).

John Drummond Kirkland. 1. How would you formulate the following substances, according to the " Unitary System"? viz.:— i. HO, NO, II. HO, SO, in. C„ HC13 IV. C4' H CI, 02

2. Give a list of the " non metallic " elements, with their atomic weights, distinguishing those which are "Artiads" from those which are "Perissads." CXXV111 EXAMINATION PAPERS, 3. Describe the general process for the detection of poison, both mineral and vegetable, when con­ taminated by foreign organic matter. Mention the object of each step in the process.

4. In the " Methyl" series, give the preparation and properties of as many of the compounds used in medicine as you can.

PEACTICAL CHEMISTEY. John Drummond Kirkland. 1. Qualitatively analyze the powders marked respect­ ively A, B, C. («» 2. Examine the calculi marked respectively D, E. W

3. A substance weighs in air 18-24 grains, in alcohol 11-02 grains. The specific gravity of the alcohol being = 0*792, what is the specific gravity of the substance ? -

(a) Acid Potassium Oxalate. Barium Carbonate. Alumina. (6) Phosphate of Lime. Uric Acid + Urate of Ammonia. HONOUR EXAMINATIONS, O.T. 1870. CXxix

MATEEIA MEDICA, THEEAPEUTICS, AND MEDICAL BOTANY.

S. D. Bird, M.D.

1. Describe the Calabar Bean, its properties, compo­ sition, and therapeutics.

2. What is the influence of Opium on the Digestive, Nervous, Vascular, Respiratory, Secreting, Cuta­ neous, and Sexual Systems? What circumstances influence the operation of Opium ?

3. Describe Sulphur, its properties, officinal prepara­ tions and compounds, and their several thera­ peutics.

4. Give a sketch of the general therapeutics of Mercu­ rials.

5. Describe Carbolic Acid, its preparation, properties, and therapeutics.

6. Give general rules for the administration of Alco­ holic Stimulants in disease.

7. What plants and what parts of them are officinal in the families Umbelliiersc, Convolvulaceae, So- lanacece, Euphorbiaceoe, and Piperaceae, what is their medicinal action, and what is the active principle to which such action may be or has been ascribed ?

8. What are the Spinal Sedatives ? To what morbid states are they applicable ? /3 CXXX EXAMINATION PAPERS,

SURGERY.

Edwd. Barker, M.D., F.R.C.S. Eng.

1. Define Erysipelas, its causes, describing especially a case of the Phlegmonous Variety of the leg, with the local and general treatment, and the reasons for its adoption.

2. Which are the most frequent situations of fractures of the femur; give the age at which each may be expected to occur, their symptoms, diagnosis, and treatment.

MIDWIFERY AND DISEASES OF WOMEN AND CHILDREN.

Richard T. Tracy, M.D., L.R.C.S.I.

1. What is contained in the umbilical cord of the foetus ?

2. Describe the manner in which vascular connection is first established between the foetus and uterus in the early stage of conception.

3. Describe the exact position and size of the unim- pregnated uterus in a healthy woman, also its exact relations to all adjacent parts. HONOUR EXAMINATIONS, O.T. 1870. CXXXl

4. Describe the position, shape, size, and structure of the ovaries in a healthy woman.

5. Enumerate the different deviations from the natural process of the third stage of labour that may cause retention of the placenta, and also describe fully how you would treat each case.

6. How would you treat a case of severe post partum hcemorrhage which did not yield to ordinary treat­ ment ?

7. From what'different causes, both constitutional and local, may dysmennorhea arise ? Give the treat­ ment you would adopt in each case. EXAMINATIONS IN THE FEBRUARY TERM, 1871.

MATRICULATION EXAMINATION.

GREEE. Professor Hcarn.

SMITH, Smaller Greek Grammar. XENOPHON, Anabasis, Books IV. V. VI.

\_N.B.—The principal parts of a verb are the first persons singular indicative of its present future and perfect active, of its perfect passive and second aorist active; or if these le not in use, then of the present future perfect and aorist employed. In parsing a verb give its tense mood voice and principal parts. In parsing a substan­ tive or an adjective- give its gender number case and nominative and genitive singular. No ab­ breviation of any Greek word is to be used. You need not accentuate.]

1. Translate literally— 'AKOvaai; ravra b SEVOCJ>U>V, irpocayayijv irpbg rfjv \apaCpav, roiig fiev btrXlrag EKEXEVE Oecrdai ra SirXa' avrbg Be Btafldg avv TOUQ Xoy(ayo~ig, eaKOirE'iTO Tzorepov in] Kpelrrov dtzdyeiy Kal rovg Biaj3Eftr]Korag, {) Kal CXXX1V EXAMINATION PAPERS,

Tovg btrXiTag Biafliflatreiv cog aXovrog av rov y/wplov. 'EOOITEI yap rb fjiv dirdyetv OVK eivai dvEV TTOXXWV VEKp&v, IXEIV C' av IfOVTO Kal bi Xoy(ayol rb y^wptov. Kal b Sevorpuiy crviEy^wpriTE ro'tg iepolg triorEvaag' bi yap fidvreig dwoBeBeiyfiEt'Oi i]crai on fiayy) JXEV EIT] TO BE riXog KaXor rijg eS,6Bov. Kai Tovg fiiv Xoy/ayovg TTE/xTi-Ei Biafliftdiroyrag Tovg iWXlrag dvrog BE JXEVEI dva\ioptcrag Cnravrag Tovg TEXraaTag Kal ovBiva Eta aKpofloXitTEirOai. E?m B' I)KOV bi birXIrai, EKEXEVOE TOV Xoy(i>y EKaarov itoir\aai TOIV Xoy^aywv dig av Kpdnara oivrai dy)g yap i\v {] iraparal-ig Bid rb y^copiov. 2. In the foregoing passage point out (a) an infinitive used for a noun, (J) a genitive absolute, (c) a neuter plural subject with a singular verb, {d) an accusative of time. 3. Decline throughout the following words:—avrog, yXvKvg, Bvvafiig, fiei^wv, ovBEig, oBovg, iraiprjg, crrpaTio)TT]g. 4. Give the principal parts of the following verbs:— dipiii), flaivw, yiytcouKU), idw, "icrTj]fxi, Ttday^o), Triirni), Tvy\avco, cpaivw. MATRICULATION, F.T. 1871. CXXXV

5. Write down throughout the following tenses :— 1st aor. ind. act. of ndrjfii, 1st aor. ind. pass, of o-w£w, 1st aor. ind. mid. of TTUVLO, perf. ind. act. of dyyEXXco, 1st aor. opt. pass, of SiBwfii, imp. ind. act. of bpaw.

6. Parse the following words:—ayioviE'iadat, aXovrog, BiriyKvXicrftit'Ovg, TrapecrKEvaaro, Trepippayfjvai, ar\- fiyy>], irvvEiopwv, y/Eipovg.

7. Give the meaning and the derivation of the fol­ lowing words:—dirpoaBoK^Tog, BopKaBEg, tKaTEpwSet', iirei,i)Bia, vavTrr/yticri/ia, yeoiKopog, bX6rpoy(og, irapa- KClTaQI]KT).

8. Translate the following passages:— (a) tn B' avro'ig rpirog fiacrrbg Xottrog rjv TTOXV 6pdii>- TCITOC, b v~tp TT}C iirl no trvpl KaraXi]ipdEicT7ig ipvXaKijr rijg vvKrbg VTTO TUIV irpoeXOovTUiv. {b) KaraKEifiEvcoy Be iXEEtvbv rjv \iuiv iirnrETmoKvIa orij) fir] irapappvEit]. {c) EK BE rbvrov trdv bfiov eyivero TO EXXnyiKov, Kal iaKi]vi)crav dvTOv iv woXXalg Kal KaXdlg oiKiaig Kal i7rirt]BEloig Baxj/iXicn Kal yap oivog woXvg »;»', &UTE iv XaKKoig Koviaroig Eiy^ov.

0. What is meant by the Generic Article ? Give an example.

10. What is meant by the Predicative Participle,? Give an example.

11. Explain the meaning of the expressions, "The Present Stem," "The Verbal Stem," "The Root," and give examples of each. cxxxvi EXAMINATION PAPERS, 12. Distinguish between the following expressions :— (a) b dvrjp ECTTI KaXog, dvrjp ECTTI b KaXog, {b) avrog b SEvoipuiv, b avrog Sevoch&v. (c) i] dyopd flier], 7/ fiicrr] dyopd.

LATIN.

Professor Heam.

SMITH, Smaller Latin Grammar. CESAR, Gallic War, III. IV.

[N.B.—In parsing a verb give tense, mood, voice, and principal parts, viz., the first person singular present and perfect indicative (active if in use), the supine in um, and the present infinitive {active if in use). In parsing a substantive or an adjec­ tive give gender, number, case, and nominative and genitive singular. No abbreviation of any Latin word is to be used.] 1. Translate— Genus hoc est ex cssedis pugnre: prime per omnes partes perequitant et tela conjiciunt, atque ipso ten-ore equorum et strepitu rotarum ordines plerumque perturbant et, quum se inter equitum turmas insinuaverint, ex essedis desiliunt et pedi- bus prcelia'ntur. Aurigtc interim paullatim ex proslio exccdunt atque ita currus collocant, ut, si illi a multitudine hostium premantur, expeditum MATRICULATION, F.T. 1871. cxxxvii ad suos receptum habeant. Ita mobilitatem equitum, stabilitatem peditum in prceliis praestant, ac tantum usu quotidiano et exercitatione efficiunt, uti in declivi ac praecipiti loco incitatos equos sustinere et brcvi moderari ac flectere et per temonem pcrcurrere et in jug-o insistere et inde se in currus citissime recipere consuerint. Quibus rebus, perturbatis nostris novitate pugnae, tempore opportunissimo Caesar auxilium tulit: namque ejus adventu hostes constiterunt, nostri se ex timore receperunt. Quo facto, ad lacessendum etadcommittcndum prceliumalienum esse tempus arbitratus, suo se loco continuit et, brevi tempore intermisso, in castra legiones reduxit. Dum hiec geriintur, nostris omnibus occupatis, qui erant in agris reliqui, discesserunt. Secutoa sunt continues complures dies tempestates, quae et nostros in castris continercnt, et hostem a pugna prohiberent. Interim barbari nuncios in omnes partes dimiserunt paucitatemque nos- trorum militum suis prtedicaverunt et, quanta praedce facienda; atque in perpetuum sui liberandi facultas daretur, si Romanes castris expulissent, demonstraverunt. His rebus celeriter magna multitudine peditatus equitatusque coacta, ad castra venerunt. 2. Who wrere the enemy to whom allusion is made in this passage, and what circumstances led to the hostilities between them and Caesar ? 3. In the foregoing passage point out— (a) a subjunctive of purpose, (J) a subjunctive of result, (c) an accusative of time, (d) an ablative of time. cxxxvm EXAMINATION PAPERS,

4. Decline throughout the following words :—Celer, dies, dives, pulvis, vis, unusquisque.

5. Give the principal parts of the following verbs :— Consuesco, haereo, lacesso, pango, praesto, veto, vincio.

6. Write down throughout the following tenses: — Pres. Subj. Pass, of capio, Past-Perf. Ind. Act. of malo, Fut. Ind. Act. of eo, Imp. Pass, of loquor, Past-Imp, Ind. Act. of venio, Past-Perf. Subj. Pass, of fero.

7. Parse the following words:—Adisset, constituisse, continentem, hiemi, nactus, peditum, projcipiti.

8. Give the meaning and the derivation of each of tho following words: — Ephippiis, meridiano, postridie, septemtriones, vicos, vectigales.

9. Give with their respective meanings examples of the following—adverbs in -itus, verbs in -urio, adjectives in -ax, substantives in -men.

10. Translate the following sentences and state the rule for every other case than the nominative found in them— (a) Caesar ubi intellexit frustra tantum laborem sumi. neque hostium fugam captis oppidis reprimi, neque his noceri posse, statuit ex- pectandam classem. (J) Obsides imperavit, quorum illi partem statim dederunt, partem ex longinquioribus locis arcessitam paucis diebus sese daturas dixerunt. MATRICULATION, F.T. 1871. CXXX1X

11. Construct a sentence in Latin shewing the difference between a direct and an indirect question.

12. Translate into Latin the following sentences— (a) Let us always so live as to believe that we must render an account. (J) It is not lawful for any one to lead an army against his native country, (c) We ought to grieve at a fault, to rejoice at its correction. {d) There are many who take from one to bestow on another.

ENGLISH.

Professor Ilearn.

CROMBIE. MORELL. LYCIDAS.

Copy in correct spelling the following passage— The isttorryeun nine aplawd the himppawrfunz and vearriatty of his subjak, but wheil he is konshus of his hone himppuffakshuns, he must often akoos the diffishshensey of his mattareyalls. Itt was emnmng tho rewens of the Gappy tall that I fust konseeved the hideer of a wurk wich has hammoozed and egsursized nere twinnty yeres of mey leif, and wich, houevir innaddykit to my hone wysshes, I finealey dillevar to the kurossety and kanddar of the publique.—GIBBON. Write a letter to a friend in England giving an account of the principal events, whether public cxl EXAMINATION PAPERS,

or personal, since the beginning of the year that appear to you interesting.

3. "Ye valleys low, where the mild whispers use Of shades and wanton winds and gushing brooks On whose fresh lap the swart star sparely looks, Throw hither all j^our quaint enamell'd eyes That on the green turf suck the honey'd showers And purple all the ground with vernal flowers." Analyze this passage.

4. Write out in prose the substance of the above passage, giving its full meaning but avoiding all poetical words and expressions.

5. Note all the words of Latin origin in the above passage and give their derivations.

6. Quote the verses descriptive of the flowers men­ tioned in the above passage. 7. Quote the verses that relate to Orpheus and shew their connection with-the rest of the poem. 8. Explain the following expressions: glistering foil: laureate hearse : monstrous world : swart star : unexpressive song. 9. Give the derivations of the following wrords: eclipse: excuse: melodious: promontory: rathe: wizard. 10. Explain and correct the errors in the following sentences— (a) The child was overlaid. {b) The river has overflown its banks. MATRICULATION, F.T. 1871. cxli

(c) You thought me not a worse man than a poet. {d) By greatness I do not only mean the bulk of any single object but the largeness of a whole view. {e) From hence arises naturally a secret satisfaction and sense of security and implicit hope of somewhat further. 11. Give examples of each of "the three general species of error against grammatical purity."

12. What is Mr. Morell's classification of conjunctions ? Give an example of each class.

FRENCH. Professor Halford.

{Hie first five questions must be answered to the satisfaction of the Examiner.)

1. Write down throughout the present subjunctive of s'asseoir, the future interrogative of avancer (shall I advance, &c), the present subjunctive of avoir, the preterite subjunctive of resoudrc (that I may have resolved, &c), the imperative of se taire, the future of s'abstenir (I will abstain, cfec). {Four of these must be done without mistake.)

2. Write down the 1st singular present indicative and subjunctive, and the present and the past par­ ticiple of 6tre, louer, pleuvoir, agreer, acquerir, Cxlii EXAMINATION PAPERS,

. fuir, hair, savoir, moudre, peindre; the 3rd plural future of valoir, the 1st singular preterite definite of plaire, the 2nd plural imperative of s'eu aller, the past participle feminine singular of confire, the 3rd singular imperative of Jeter, the 3rd singular preterite indefinite of y avoir. {Five ofthe former and three of the latter must be done without mistake.)

3. Translate literally— Ceux qui ne voyaicnt en lui qu'un sujet revoke contre son roi disaient qu'il avait morite la mort; ceux qui le rcgardaicnt comme un Livonien, ne dans une province laquelle avait des privileges a defendre, et qui se souvenaient qu'il n'etait sorti de la Livonie que pour en avoir soutenu les droits, 1'appelaient le martyr de la liberie de son pays : tons convenaient d'ailleurs que le titre d'ambassadeur du czar devait rendre sa personne sacree. Le seul roi de Suede, elevc dans les prin- cipes du despotisme, crut n'avoir fait qu'un acte de justice, tandis que toute 1'Europe coiidamnait sa cruaute.

4. Put into French— (1.) It was a'Beckett and Johnson w-ho saved the laurels of the Victorians. (2.) Which do you prefer, the former or the latter ? (3.) Some one who saw7 them play says that neither the one nor the other is worth much. (4.) Then he knows nothing about it. (5.) This is the only pen that I can find. (0.) Ask the porter to give you another. MATRICULATION, F.T. 1871. Cxliii

5. Give the French for a street, a rain-bow, a slipper, • some towels, dirty, ugly, heavy, to translate, to swim, to lie down; and the English of so souve­ nir, benit, aveugle, marais, etang, dedommager, souhaiter, soupconner, se souler, priutemps.

0. Translate— Ce fut le 8 juillet de fanned 1709 que se donna cette bataille decisive de Pultava, entro les deux plus singuliors rnonarques qui fussent alors dans le monde: Charles XII. illustre par neuf annees de victoires, Pierre Alexiowitz par neuf annees de peines prises pour former des troupes egales aux troupes suedoiscs; Pun glorieux d'avoir donne des etats, I'autre d'avoir civilise les siens; Charles aimant les dangers, et no combattant que pour la gloire, Alexiowitz ne fuyant point le peril, et ne faisant la guerre que pour ses interets; le monarque su^dois liberal par grandeur d'ume, le moscovite ne donnant jamais que par quelque vue; celui-lu, d'uno sobriete et d'une continence sans exemplc, d'un naturel magnanime, et qui n'avait etc barbare qu'une fois; celui-ci n'ayant pas depouille la rudesso de son education et de son pays, aussi terrible il ses sujets qu'admirable aux etrangers, et trop adonne tt des execs qui ont meme abregc ses jours.

7. Correct the following— Enfin, apres douzes jours d'une marcho si pe- nible, pendant lequel les suedois avait consommer le pen de biscuit qui leur reste, cet armee, ex- tenue de lagitude et de fairn, arivo sur les bords de la Desna, dans la endroit oil Mazeppa ave marquer le rende-vous; mais, au lieu d'y trouve cxliv

cet prince, on trouvat un corps de moscovites qui advangaient vert I'autre bord du riviere.

Put into French— (1.) They crossed the river in their accustomed manner, some upon rafts made in haste, others by swimming. (2.) Mazeppa secretly leagued with the King of Sweden in order to hasten the downfall of the czar and to profit thereby. (3.) Wherever there is a crowd of writers there is a crowd of libels.

ARITHMETIC.

Professor Wilson.

All working required in obtaining a result must be sent in as part of the answer. Every result must be reduced to its simplest form. When an answer cannot be expressed in whole numbers it must be worked to three places of decimals.

1. Write down in words the quotient and the re­ mainder obtained by dividing Five thousand nine hundred and sixty-nine billions five hundred thousand millions ten thousand by One hundred and ninety-seven thousand and nine millions nine thousand nine hundred. In this case an integral quotient only is required.

2. Two clocks are set together at 3 a.m. on the 1st MATRICULATION, F.T. 1871. Cxlv

January 1871; one of them gains uniformly 1 minute 37 seconds in 24 hours and the other loses uniformly 2 minutes 33 seconds in 24 hours ;, when will they next shew the same time ?

3. A clock is 27 seconds slow at 2 hours 51 minutes a.m. on the 22nd April and 21 seconds fast at 7 hours 7 minutes p.m. on 8th May; supposing it to have gained uniformly when did it shew correct time ? 4. Messengers travelling ten miles an hour are sent out every six minutes to meet a person approach­ ing at five miles an hour : at what intervals will they meet him ?

5. Supposing £1 to be worth 25-6 francs; express the value of a twenty franc piece in shillings.

6. A gramme is 16"434 grains: express 1000 tons in grammes. 7. Supposing the food of one man for one day costs Is. 8Jd. what will it cost to feed half a million men from 1st September to the 10th of the fol­ lowing January including both days ?

2 a "R i (V -*) + 3(V-*) f , . , a. Reduce M , t . .. ;01 ., j-. to a decimal. ~'° U ~r s) — •i'-'1 (r — s) -0807-(-27--1902) 9. Reduce TTS rm~r; ,1/.- ,, to a vulgar frac- •>•/ — o (rj - •ubi' I) ° tion in its lowest terms. 10. How many yards of fencing will be required to enclose a square paddock 250 acres ? 'J Cxlvi EXAMINATION PAPERS,

ALGEBRA.

Professor Wilson.

All work required in answering a question must be sent in as part of the ansiver. Every result must be reduced to its simplest form.

1. The number a is divided by the sum of the num­ bers a and x and the number b is divided by the sum of the numbers b and x and the two quotients are added together ; write down tho algebraical expression for a number which when multiplied by itself will give the sum of these two quotients.

2. Four times the product of a* and x2 is multiplied twice in succession by four times the product of a and x; write down the square root of the * result.

3. A gambler first wins (2a—b) pounds he then loses {a+2b) pounds he then wins (3a—2b) pounds and again loses five times (a—b) pounds, after which he has nothing left: how much had he to start with ?

4. Reduce to its simplest form _1 h-a a ab + 1 " 1 b-a 1 + a * ab + 1 MATRICULATION, F.T. 1871. cxlvii

18 12 5; Multiply together — + -r and

ab (-y - -g-) - a'-b* (2a - 3b)

6. Substitute {x — a) for x in a3 + 3asa; + Sax2 + x3

2x 7. Substitute rr- for y in Qa3y3 — Sa2y2 + 2ax — 1

2a 2b n a b . 8. Substitute -r- — — for x and fr. + K for y in a-8 -£ +2xy + Ay2

9. A person walks 20 miles at a miles an hour and 10 miles at b miles an hour; at what rate must he walk that he may do the whole distance uniformly in the same time.

10. Solve the equation x - 1 2x - 1 1 1 — 2 2x — 2~ x — 1

11. Solve the equation (p — qx) {p -f q) — {p + qx) {p - q) x = iv — 4) ip +

EUCLID.

Professor Wilson.

Algebraical Symbols must not be used. Book I. Three of these must he written out. 1. From a given point to draw a straight line which

shall be equal to a given straight line.

2. To bisect a given rectilineal angle.

3. If two straight lines cut one another the vertical or opposite angles shall be equal. 4. Any two sides of a triangle are together greater than the third side. 5. In any right angled triangle the square described on the side subtending the right angle is equal to the squares described on the sides containing that ang-le.

Book II. Two of these must be written out. If a straight line is divided into two equal parts an also into two unequal parts the rectangle contained by the unequal parts together with the square on the line between the points of section is equal to the square on half the line. MATRICULATION, F.T. 1871. Cxlix

7. If a straight line is divided into two equal parts and also into two unequal parts the squares on the unequal parts are together double of the square on half the line and the square on the line between the points of section.

8. To describe a. square that sliall be equal to a given rectilineal figure.

HISTORY.

Professor Hcarn.

Candidates are to select any two, but not more, of the subdivisions of each question.

I.—Describe the geographical position of each of the followring places; and state, with their respective dates, the principal historical events connected with each place :— 1. Lamia : Thurii: Thermopyhe. 2. Arpinum : Pergamus : Soli. 3. Acre : Corunna : Stamford Bridge.

II.—Give, with the proper dates, some account of each of the following persons :— 1. Phoebidas : Plato : Polycrates. 2. M. Licinius Crassus: M. Marcellus: Sextus Pompeius. 3. Egbert: Robert Devcreux, Earl of Essex : Captain Porteous, III.—Give, with the proper dates, some account of each of the following events :— 1. The Battle of Delium: The Battle of Ipsus : The Siege of Tyre. 2. The Battle of the Colline Gate : The Intro­ duction of Gladiatorial Exhibitions: The First Invasion of Britain. 3. The Mission of Augustine: The Divorce of Henry VIIL from Queen Katharine: The Impeachment of Warren Hastings.

IV.—State the principal events in each of the following years :— 1. B.C. 546: 425: 336. 2. B.C. 216: 133: 44. 3. A.D. 1283: 1483: 1783.

GEOGRAPHY.

Professor McCoy.

1. Describe the directions, times, and tracks of the Hurricanes of both hemispheres. 2. Describe the boundaries of the north and south zones of constant rain.

3. What are the north and south boundaries of the area in the northern hemisphere in which the vine and orange flourish ? MATRICULATION, FT. 1871. cli

4. Draw a rough outline map of America.

5. Mark out and name on the above map the chief political divisions.

6. Mark on the above map the chief rivers, naming them on a separate list, and the main mountain chains.

7. Mark with numbers on the map, and name on a

corresponding list, the principal islands.

8. Mark on the above map each of the seas.

9. Give any general and political information in your text-book relative to the United States and the West Indies. 10. Trace the northern limit of the geographical dis­ tribution of Reptiles. dii EXAMINATION PAPERS,

EXAMINATION FOR EXHIBITIONS AT MATRICULATION.

GREEK:.

HOMER, Iliad I. XENOPHON, Anab. IV. V. VI.

Professor Hcarn.

1. Translate the following passages:— (a) ArpEiBi], vvv dfifiE T7CtXiinrXayy(6E)'Tag o'ito tvli diruvooTijiTEiv, tt K£v Odvaro!' ye ifwyoifiEV, el Bi] bfiov —oXefiog re Biifiij Kal Xoifibg Ay^awvg. dXX' dyE 017 rtra fidvnv epeioftev, j) lepija, 77 ml oi'EipoTrbXov—Kal yap r' bvap EC Awe icrnv— «S K' enrol 'ii n roacrov iy/Mcraro $>oij3og 'ATTOXXWV, in dp 6y Evy^ioXrig iirifiifiiperai e'ld' EKarofi'prig' ai KEV Trwg dpv&v Kviarjg alyaiv re rtXeiiov flovXerai dvndirag fiftiv d—6 Xoiybv dfivvai. "Ilrot oy wg cliruiv KUT' dp' i^ero. TIHCTI B' dvearr] YJaXyag OeirroplBrjg, O'IIVI'OTTOXWV 6% apicrroij' bg l)Bi] ~d T eovra -a r eacrofieva irpo r Eovra, Kal I'yecrcr j;y/;irar Ay(aiiov ' IXwv eicrw, i)v Bin fiavTotrvviji; ri]v bi Trope

{b) 'E-EI ?' EipEvyev b £evoipw>; KciToiKOvvrog ijBt) avrov iv SKIXXOVVTI Jin) TUIV AaKiBatfiovloiv oiKtcrOivrog trapd ri]V OXvfmiav, dipiKvelrai Rleyd/)v%og elg OXvfnrlav dtwpi'icnov, Kill d~oBiBiocri TI)V 7ra»ai.ara- 0>]Krji' avrb). Sevoipwr BE XiiftH)v y^oipiov lovelrai rjj dtio, ItTTOv dve'tXev b Hebg. "Rrvy^e Be Bid fieirov peo>v rov y^wplov trOTiiftbg Sf.Xi roue. Kal iv 'Eipicrii) BE Trapa -or rf]g 'Aprefiicog vtwr HeXivovg Troraftbg wapappe't, Kai l\0veg CE iv cifiipoTepmg Eveiai nal MATRICULATION—EXHIBITIONS, F.T. 1871. cliii

Kuyy^ai' iv Be no iv IZKIXXOUVTI y^wpiio Kal dijpai rrdv-wv, birbaa icrrlv dypevbfieva tir/pla. 'Ej-o/ijirt ci Kal {nwfiov Kal vaov d~o TOV tepov dpyvpiov, Kal TO XOITTOV BE del BCKCITEVWI' ra EK TOV dypov copala, Overlay i~olei rfj deo), Kal irdvreg bi TroXIrai Kal bi irpbiryriitpoi, bfiov dvBpeg Kal yvvajKeg, fiErely(Ov rfjg eoprijg. Hapely^e Be debg rang crKrjvwcnv dXcjira, dprnvg, olvov, Tpayt'ifiara, Kal TWV dvofiivcov dirb rfjg lEpdg vofiTjg Xdy^og, Kal TWV OrjpEVOfiivoiv Bi.

. Translate tho following sentences:— (fl) Kpe'iaainv yap J3acnXevg OTE y/jfoaETai dvCpl \epni inrep yap TE y^oXov y£ Kai avTtjfiap Karanfxpv dXXa re Kal fieroTioOev ey/ei KI'ITOV bippa TeXicrcn]. {b) lib o' dvrw fidprvpoi ECTLOV Trpbg TE OEUIV fiaKapwv irpog TE OVIJTWV dvOpwTrwv Kai Trpbg TOV flacriXijog drrrjiiog, EITTOTE B' UVTE \pEii) iftelo yevjjrai deiKea Xoiybv dfivvai ring dXXoig' i] yap by oXoijcri (ppecri Ovet, {c) Iirria ftev irreiXavTO Dinar B' iv rf]i fieXalvr]' larbv o' iim/BiiKr] ireXaaav, TspoTovoiirtv vipevrtg KapTraXifiwg' TI)V 0 eig opfiov —poepvoirav ipeTfiolg. iK B' ivt'dg EpaXov Kara BE Trpvfivncri ectjerav. {d) bi Be TroXifiioi, bpwvTeg ftiv rnvg ifujil XEiplccxpov EV7TETCOC TO vBwp TTEpWVTag OpWl'TEg CE TOVC dflljjl Sevoip&VTa Oeovrag To'vfi~aXiv, Beicrat'Teg ft>) diro- KXeicrtidijcrav ipevyovcnv diet K/idrog wg Trpbg TI)V d?ro rou TTOTiifxov eKflacriv dvw. {e) TWV Be TroXwpKovfierwv diroXofieviov, crvv rolg Xeipi- crocbov fibvoig KI'IKIOV ion BiaKirBweveiv, J] TWVBE CWBEVTWV irdvTag iig TCIVTOV iXOovrag KOIVI] rijg irwrt]piag 'iy^eirOai. (/) bvg yap napd Kvpov eXafle Tpicy^iXiovg ore rag oeKa yfiipag i]Xl}devcre OvbfiEvog Kvpio BieireirioKet. div EXAMINATION PAPERS,

3. Explain the following expressions— (a) tvrjpmv opEUKwoiaiv. {b) Kprjrijpag ETrEcrTeipavTO TTOTO'IO. {c) Bitrag dfiipiKVTreXXov. {d) icnpayidi^ovTO iig rov Ttorafibv. {e) T))V bvpdv TOV Kepang Ttoirjaafuvoi.

4. Parse the following words :—avriaaag, a-rrovpag, ETTE\l/r]ipiKwg, EypnyopEvai, rfVEiy/ovro, traXifitrXay- Xdevrag, x(P'fi- 5. Give the meaning and the derivation of each of the following words :—afiflpoitiai,Eiracrcn>TEpoi, rjXSiwv, ovXoy(vTag, o^Or/o-ug, yEpvnfyavro.

6. Scan the following lines, and state their metrical peculiarities :— («) ovo£a TE CKioevTci ddXaaod re {]y(i]Ecraa. {b) ewg b ravd' wpfiatvE Kara ippiva Kal icard Qvfibv. {c) TO~ig a\\o(£ i] ydp by oXofjin ippeal Ovei.

7. Translate into Greek— Thereupon Xenophon rises equipped for war as handsomely as he could, since he thought that if the gods were to give victory the handsomest dress befits victory, and if he needs must die, that it was right that lie, as he had deemed himself worthy of the handsomest, should in them meet his end : and he commences his speech as follows : "The perjury and faithlessness of the barbarians Kleanor speaks of, and you, 1 think, know it too. If then we design again to be on friendly terms with them, we must have great discouragement. MATRICULATION—EXHIBITIONS, F.T. 1871. dv

as we see what treatment our generals have suf­ fered, who through their reliance upon them placed themselves in their hands. If however we are minded with our arms to punish them for what they have done, and for the future to be in utter hostility with them, with the help of the gods we have many and fair hopes of safety."

LATIN.

VIRGIL, /Eneid, X. CESAR, De Bell. Gall., III. IV.

Professor Hearn.

1. Translate the following passages— (a) Non ego te, Ligurum ductor fortissime hello, Transicrim, Cinvra, et paucis comitate Cupavo, Ciijus olorinae surgunt de vertice pcnnae, Crimen amor vostrum, formreque insigne patcrnse. Namque ferunt, luctu Cycnurn Phacthontis amati, Populeas inter frondes umbramque sororum Dum canit. et maestum Musa solatur amorem, Canentern molli pluma duxisse senectara, Linquentern terras, et sidera voce sequentem. Filius, sequales comitatus clas.se catervas, Ingentem remis Centaurnm promovet; ille Instat aqme saxumque undis immane minatur Arduus, et longa sulcat inaria alta carina.

{h) Venetireliquacque item civitates, cognito Caesaris adventu, simul quod, quantum in se facinus admisissent, intellege.bant, (legates, quod nomen clvi EXAMINATION PAPERS,

ad omnes nationessanctum inviolatumque semper fuisset, retontos ab se et in vincula conjectos,) pro magnitudine pcriculi helium parare et maxime ea, qute ad usum navium pertinent, providere instituunt; hoc rnajore spe, quod multum natura loci confidebant. Pedestria esse itinera concisa aestuariis, navigationem impeclitam propter in- scientiarn locorurn paucitateinque portuum scie- bant: neque nostros exercitus propter fiumenti inopiam diutius apud se murari posse, confidebant: ac jam, ut omnia contra opinionem acciderent, tamen se plurimum navibus posse: Romanes neque ullam facultatern habere nuvium, neque eoi-urn locorurn, ubi helium gesturi essent, vada, portus, insulasque novisse : ac longe aliam esse navigationem in concluso mari, atque in vastissirno atque apertissimo Oceano, perspiciebaut.

2. Translate and explain the following sentences— (a) Equidem credo mea vulnera restant Et tua progenies mortalia demoror arma. {b) Quae cuique est fortuna hodie quam quisquo secat spent Tros Rutulusvc fuat nullo discriinine habebo. {c) Gens illi triplex, popnli sub g-cnte quaterni : Ipsa caput pupnlis: Tusco de sanguine vires. {d) In quos eo gravius Caisar vimiicandum statuit quo diliguentiusin reliquum tem^ius a bnrbaris jus legatorum conservaretur. Itaque omni senatu necato, reliqnos sub corona vendidit. {(•) Op])ortunissiina res accidit, quod postridie ejus diei mane eadeni et perfidia et simulations usi Germani frequenter, omnibus principibus ma- jiiribusquc natu ndhibitis, ad enm in castra ven- erunt; simul, ut dicebatur, sui purgandi causa MATRICULATION—EXHIBITIONS, F.T. 1871. civil

quod contra atque esset dictum et ipsi petissent, proeliurn pridie commisissent; simul ut, si quid possent, de induciis fallendo impetrarent.

3. Explain the following expressions— {a) Oricia terebintho. (J) Tacitis Amyclis.. (c) Omnipotentis Olympi. {d) Primipili centurio. {e) Supplicatio a senatu decreta.

4. Give the meaning and the derivation of the following words :—eminus, exorsa, hodie, irrita, lacessere, malacia, provincia, sonteni.

5. Scan the following lines and state their metrical peculiarities— (a) Cujdit seniianimis Rutulorum calcibus arva. {b) Grains homo infectos linquens profugus by- menaaos. {c) Clamorc incendunt ccelum Troesque Latinique. 6. State the substance of the description given by Ciesar of the persons called "Soldurii." On what occasion does he mention them, and what is the corresponding name among the Teutonic nations ?

7. Translate into Latin the following— After his death the Ilelvetii nevertheless en­ deavour to carry into effect the resolution that the}' had adopted of leaving their country. When they supposed that their preparations for this object wvre complete, they set fire to all their towns to the number of twelve, to their villages dviii EXAMINATION PAPERS,

to the number of four hundred, to the rest of their private buildings ; they burn all their corn except that which they intended to take with them, that by taking away the hope of return home they might be more prepared to undergo all dangers; they order each man to take with him from home provisions for three months. They persuade their neighbours to burn in pursuance of the same policy their towns and villages and to join their expedition. There were on the whole two roads by wliich they could leave home. One through the Sequani, narrow and difficult, between Mount Jura and the Rhone, where wagons in single file could hardly be brought; moreover an exceedingly lofty mountain hung over it so that-n very few men could readily step the passage: the other through our province, much easier and less obstructed, because the Rhone flows between the territories of the Hel- vetii and the Allobroges, who had lately been re­ duced to peace.

ENGLISH. Professor Hcarn. 'Daughter to that good Earl, once President Of England's Council and her Treasury Who lived in both unstained by gold or fee And left them both, more in himself content, Till the sad breaking of that Parliament Broke him as that dishonest victory At Chajronea fatal to liberty Killed with report that old man eloquent." MATRICULATION—EXHIBITIONS, F.T. 1871. dix

(a) Who is the author of this passage and to w-hom is it addressed ? {b) Who was the good Earl, what were the exact titles of his offices, and do these offices or either of them or their equivalents now exist ? {c) What was the date of the breaking of the Par­ liament, what was the cause of that event, and how was that Parliament distinguished ? (a*) Where was Chaeronea and what is the date of the battle referred to ? (e) What other celebrated battle was fought in the same place and when ? if) Who was the old man eloquent and what is the allusion ?

2. Explain the allusions in each of the following passages, and mention the poems in which they respectively occur— (a) Where I may oft outwatch the Bear With thrice great Hermes. {b) Where grateful science still adores Her Henry's holy shade. (c) He saw, but blasted with excess of light Closed his eyes in endless night.

3. Refer to their several contexts the following pas­ sages— (a) They also serve that only stand and wait. {b) He raised a mortal to the skies, She drew an angel down. (c) Where ignorance is bliss Tis follv to he wise. clx

4. Explain the meaning of the following passages and notice any peculiarity in their construction— (a) Then muse not Nymphs that I bemoan The absence of fair Rosaline; Since for a fair there's fairer none Nor for her virtues so divine. {b) But thy eternal summer shall not fade Nor lose possession of the fair thou owest. (c) These twain that did excel The rest so far as Cynthia doth shend The lesser stars.

5. Explain the following expressions :—Variable flowers : The buskinea stage : Jonson's learned sock.

6. Give the meaning and the derivation of each of the following words:—Adamantine, Buxom, Curfew, Cynosure, Forlorn, Guile.

7. Quote the passages from the poems of Milton and of Gray in which reference is made to Pindar.

8. State the references to his personal history con­ tained in Spenser's " Prothalamion."

9. What is the difference between a period and a loose sentence ? Cite or construct an example of each.

10. Distinguish between thejrses of the following words:—each, every, either, both. MATRICULATION —EXHIBITIONS, F.T. 1871. clxi

FRENCH.

Professor Halford.

. Translate into idiomatic English— («) MAD. J.—Ah ! ah ! voici une nouvclle histoire ! Qu'est-ce que c'est done, mon mari, que cet equi- page-lil 1 Vous moquez-vouz du monde, de vous £tre fait enharnacher de la sorte ? et avez-vous envie qu'on so raille partout de vous? MONS. J.—.11 n'y a. que des sots et des sottes, ma femme, qui se railleront de moi. MAD. J.—Vraiment, on n'a pas attendu jusqu'a, cetteheure; et il y a longtemps que vos fagons de fa ire donncnt a rire a, tout le monde. MONS. J.—Qui est done tout ce monde-la, s'il vous plait ? MAD. J.—Tout ce monde-la est un monde qui a raison, et qui est plus sage que vous. Pour moi, je suis scandalisec de la vie que vous menez. Je ne sais plus ce que c'est que notre maison. On dirait qu'il est ceans careme-prenant tous les jours; et des le matin, de peur d'y manquer, on 3r entend des vacarmes do violons et de chanteurs dont tout le voisinage se trouve incommode. NICOLE.—Madame parle bien. Je no saurois plus voir mon menage propre avec cet attirail de gens que vous faites venir chez vous. Us ont des pieds qui vont chercher de la boue dans tous les quartiers de la ville, pour 1'apporter ici; et la pauvre Francoise est presque sur les dents, il frotter les planchers que vos biaux maitres viennent crotter regulicromont tous les jours. MONS. J.—Ouais ! notre servante Nicole, vous avez le caquet bien affile pour une paysanne ! clxii

MAD. J.—Nicole a raison; et son sens est meilleur que le vfitre. Je voudrois bien savoir ce que vous pensez faire d'un maitre il danser, a, 1'age que vous avez. NICOLE.—Et d'un grand maitre tireur d'armes, qui vient, avec ses battements de pied, cbranler toute la maison, et nous doraciner tous les car- riaux de la salle. MONS. J.—Taisez-vous, ma scrvante et ma femme.

(5) LE MAITRE D'ARMES, apros avoir pris les deux fle.urets de la main du laquis, et en avoir presente un a monsieur Jourdain. Allons, monsieur, la reverence. Votre corps droit. Un pen penchc sur la cuisse gauche. Les jambes point tant ecartees. Vos pieds sur une meme ligne. Votre poignet a, 1'opposite de votre hanche. La point de votre epee vis-a-vis de votre epaule. Le bras pas tout h fait si tendu. La main gauche a la" hauteur de 1'ceil. L'6paule gauche plus quartet. La t

LE M. D'ARMES.—Je vous 1'ai dejil dit, tout le secret des amies ne consiste qu'en deux choses, il donner et a. ne point recevoir; et, comme je vous fis voir I'autre jour par raison demonstrative, il est impossible que vous receviez si vous savez d^tourner Tepee de votre ennemi de la ligne de votre corps ; ce qui ne depend sculement que d'un petit mouvement du poignet, ou en dedans, ou en dehors. MONS. J.—De cette facon, done, un horame, sans avoir du cceur, est sur de tuer son homme, et de n'tStre point tuc ? LE M. D'ARMES.—Sansdoute; n'en vltes-vous pas la demonstration ? MONS. J.—Oui. LE M. D'A.—Et c'est en quoi Ton voit de quelle consideration nous autres nous devons etre dans un Etat; et combien la science des amies Temporte hautement sur toutes les autre sciences inutiles, comme la danse, la musique, la . . . , Put into French—• (1.) No country surpasses Victoria, its climate is charming, whatever people sty about it. (2.) Do you think he loves me as much as he tells me he does ? (3.) If he will not consent to it, I will force him to it. (4.) Is it true that you are going to England ? (5.) I am delighted that it is you wrho come here. (6.) Let them know at least that to m3r last mo­ ment I have thought of them.

. Correct the following— Tels etait la fierte et ignorance de cette peuple dxiv EXAMINATION PAPERS,

qu'ils crurent avoir etes vaincu par une pouvoir d'avantage que humaine, et que les Suudois etait de vrais magiciennes. Cet opinion fut si general, qu'on ordonna il ce sujet des publics prieres a, Saint Nicholas, patron de Moscovie. Ce prifire et trop singulier pour n'etre pas rapporlo; le voici : " 0 toi, qui est notre consolateur per- peruolle dans tous nos adversit,6s, grande St. Nicholas, infiniemcnt puisant, par quelle peche vous avez-nous offense dans notres sacrifices, genuflexions, reverences, et actions do graces, pour que tu nous ait ainsi abondonnaient ? Nous avions emplore votre assistance contres cos ter- ribles, insolentes, enrage, eponvantable et in- domptable destroj-eurs, lorsque, commes des lions ou des our qui a perdus leur petits, ils nous a attaquer, eff'raier, blesscr, tuer par millers, nous qui sont votre peuple. Comme il est imposible quo cola est arrive sans enchantment nous toi supplions, o grande St. Nicholas d'etre le cham­ pion et le port-etendard de nous, et do delivrez nous do cc foule des sorcies, et de cliassoz eux bien loins do nos frontiers aveeque hi recompance qui les est du."

4. Translate into French— Among the priests busied in the revolt figured a vicar named Bender, a man of engaging man- ners, persuasive eloquence, active without bounds, superior far in mind to those who surrounded him, bur hiding behind so man}- advantages a parching thirst for power, a genius for intrigue, dissolute habits, and a soul capable of the darkest crimes. Little known, too, at the epoch of which we speak, for what he really was, this skilful hypocrite en- jo3red, in the army of La Vendee, an ascendancy MATRICULATION—EXHIBITIONS, F.T. 1871. dxv

which soon became decisive. It was he who, in concert with Bernard de Marigny and D'Elbee, suggested the formation of a body charged with deciding, under the name of Conseil Superieur, judicial and administrative questions.

ARITHMETIC AND ALGEBRA.

Professor Wilson.

1. State and prove the rule for finding the greatest common measure of two numbers; State also fully what modifications are permissible, desirable or necessary when the highest common factor of two algebraical expressions is required. Find the highest common factor of a3 + 3a2 + 7a + 5 and 5a2 + 5a also the greatest common measure of the numbers represented b3r these expressions when a — 10.

2. Explain the reasons for the several steps of the rule for extracting the square root of a whole number. Extract the square root of 5329 Ivy the ordinary process, and also b3' first expressing it as an algebraical expression which is a perfect square as such.

3. Define the symbol a ; From 3rour definition prove that a . a ~ a ; How is the meaning of the symbol a arrived at ? dxvi EXAMINATION PAPERS,

4. Find the relation which must exist between x and a + x . a y in order that . may be equal to -i •

5. A clock was p seconds slow at noon on the 1st January and q seconds fast at noon on the 2nd; find its error at 3 a.m. on the 3rd supposing it to gain uniformly.

6. Divide - — - by aT* — v~4 x y J J 7. Find the square root

12. Solve the equations x3 + y3 = 152 *2y + xy2 — 120 MATRICULATION—EXHIBITIONS, F.T. 1871. dxvii

EUCLID.

Professor Wilson,

1. If an angle of a triangle and the adjacent exterior angle be bisected, the lines which bisect them are at right angles to one another.

2. If the diagonals of a parallelogram are equal to one another the parallelogram is a rectangle.

3. If from the vertex of a triangle a straight line is drawn to the middle point of the base twice the square on this line together with half the square on the base is equal to the squares on the two sides of the triangle.

4. Shew how to find the center of a given circle.

5. If two circles touch each other internally the • straight line which joins their centers being produced will pass through the point of contact.

6. The opposite angles of any quadrilateral figure inscribed in a circle are together equal to two right angles.

7. If two chords of a circle cut one another the rect­ angle contained by^ the segments of one of them is equal to the rectangle contained by the seg­ ments of the other.

8. To describe an isosceles triangle having each of the angles at the base double the third angle. clxviii EXAMINATION PAPERS,

9. If two circles cut one another the line which joins their centers will bisect their common chord. 10. To describe a circle which shall pass through a given point and touch a given straight line in a given point. 11. Shew that the area of an equilateral triangle cir­ cumscribed about a circle is four times the area of the equilateral triangle inscribed in the same circle. 12. If a circle is described on the radius of another circle as diameter, an3r chord of the larger circle passing through the point of contact is bisected by the circumference of the smaller circle.

HISTORY.

Professor Hcarn. I.—1. What was the earliest form of Hellenic Govern­ ment, and what changes did it undergo ?

2. State the distribution of the duties of Govern­ ment among the Archons and the special designations of these officers.

3. What was the first instance of discontent among the members of the Confederacy of'Delos, and what was the date of this event ?

4. Enumerate the principal temples aud statues in the Acropolis of Athens. MATRICULATION—EXHIBITIONS, F.T. 1871. dxix

5. What were the three stages of Attic Comedy and what were their respective characteristics ? 6. What "were the four schools of Athenian philo­ sophy and who were their respective founders ?

II.—1. Into what classes may the early inhabitants of Italy bo divided, and what is the ground of the classification ? 2. What were the principal institutions ascribed by the Romans to ? 3. How under the Republic were members of the Senate appointed ? 4. What were the circumstances that led to the commencement of the ? What was the date of the beginning and of the end of this contest ? 5. What was the origin of the Roman Satire ? What is the meaning of the name and who was its first distinguished writer? 6. Give with their respective dates some account of the principal Latin Comic Dramatists.

III.—1. Draw a rough map of Britain as it was left bj' the Romans, showing its northern boundary, its principal towns and harbours, and its main roads. 2. What claim of right did Henry II. set up to the Crown of Ireland ? 3. What claim of right did Edward I. set up to the Crown of Scotland ? h dXX EXAMINATION PAPERS,

4. What claim of right did Edward III. set up to the Crown of France ? 5. Make a list of the Queens Consort since the accession of James I. 6. Explain the allusions in the following passages— (a) " When the British Warrior Queen, Bleeding from the Roman rods." {b) "And that fierce Knight of Ellerslie." (c) " On the deck of fame that died With the gallant good Riou." (a*) " And Worcester's laureate wreath."

GEOGRAPHY. Professor McCoy. 1. Make a rough outline map of New South Wales and mark on it the chief baj-s, capes, rivers, and mountain chains. 2. Draw a rough map of England and mark -out the boundaries of the counties, and mark the place of the chief rivers, capes, islands, and chief towns b3r numbers or letters referring to separate lists. 3. Explain the S3'stem of government in England. 4. Explain the system of government in New South Wales shewing the main differences from that of England. 5. Mention the seats of the chief manufactures of England. ORDINARY EXAMINATIONS, F.T- 1871. clxxi

ORDINARY" EXAMINATIONS FOR THE DE­ GREES OF B.A., LL.B., AND M.B., AND FOR THE CERTIFICATE OF C.E.

JUNIOR GREEK.

HOMER, Odyssey, Books XXI.-XXIV. PLATO, Apology. SMITH, Student's Greek Grammar. 1. Translate literally either (a) or (b) nnd either (c) or (d)— (a) " KF.KXVTE ftfVy fivtj(TTi}ne(j flynjcXfiri/C fia ra fie Ovfwg evi rrrt'iOedtn KEXEUEL' YJ)QVfia\ov de /mXiora Kal 'AVTIVOOV OeotiCsa AiWo/t', iirel Kal TOVTO ETTOQ Kara /.toipap ULTTEP, JS'TJI' fjtv vavGin ToZoVyt.—irpi-d"11 <5t Otdtmv 'WOJOEP St OtiiQ oto(Tti Knnroij $ K' WiXyaiv. 'AXX' ay' I {.to i OnTt TO£OI> l.vtiooi', btypa /.aO' vfilv Keipatv Kal aV'tPtog 7rttp))(70/.iai, ?} fun tr' tartv "If, oh] TrapOQ txjKiv Ivi ypaf-iTrrotai fisXtao'iv, "H ?/07/ fid yXfcfffj' dXrj T dKOftiarirj re."

(/») IMtjn/p 5' aiTTfaaaa Beovg TrfpiKaXXs' dtBXa i)t}KE fttaqj Iv uydvi dpLOTi'jsaaiP 'Axatutv. *}\0}) fttp TroXtwi' rdtP'o dvduwv di'Tt.fSnXijfTaQ 'Uinotvi', ore KIP TTOT ('nroipOifiii'Ov /3a(7iX//of '/Mvi'vvrai TE veot Kal k-mvTVPOvrai dtOXa'

'AXXa KE Kelva ficiXtara uUr Qr]i](jao Ovf.upt OV trri

(c) TTfpi ftep ovv Cov oi TrpioToi fiov Kan'iyopoi Kanjyopovv avTij ttTTio tKaPi) AiroXoyia irpbtj vfidg' irpug fie ^IEXIJTOV TOV dyaOov TE Kal ^LXOTTOXIV, UKJ tyrjviy Kal TOVQ varioovg utrd h2 Clxxil EXAMINATION PATERS,

ravra TnipaGOfiat d7roXoyt7a9at. avBig yap St), toQTrep tTEpiop TOVTIOV ovTtov Kartjynpiov, Xdpiof.tEP av rt)v TOVTOJP dpT(of.iorriap. tx£L ()£ TTWQ (OSE. StoKpdrij tp^rrlv dciKtiv TOVQ TE V'IOVQ SiatpBtinopra Kai BEOVQ OVQ ?) TTOXIQ vofiiZet ov voftiCopra, ErEpa ck SaifJih'ta Katvd. TO fiev St) tyxXtifta TOIOVTOP tcrrf TOVTOV SE TVV tyKXi'iftarog tV EKaorop ^trduiu- fup. tyijai yap St) TOVQ PEOVQ dSiKtlp f.ie SiatpBEtpovra. £yw SE ye, w dvopEg 'AOtjvalot, dotKiiv

(rf) 'E/xol oe TOVTO,

TTpea-flvrtnoL yEPoj.tt.POt t.yvioGavy on P'EOIQ OVGIV aiirolij tyto KOKOP TnoTTork n ZvvtPovXf.vtra, vuvl avrovg dvaJDaivovrat; ffiov Kart]yope7p Kal rtfiiopElaBac et Se /.n) avrol T/OtXov, TLOP oiKf.'uop Tti'dg TWP IKE'IPIOP. irar'tpag Kal dSt.Xfyovg KUI dXXouQ TOVQ TTpOCl'lKOl'TOQ, E'ITTEO WTT' tpOV Tl KOKUV tTTETTOPBtGaV aurujp OL oiKtiot, vvv f.itfiPtj(rOaL, Write down the fourth and the eighth lines of (a) and of (b), marking the feet and the quantity of eveiy syllable.

2. Parse these verbs—aKouGeade, "iayjqKe, X/tfyoi, voel, irpd^at, (TKECdaavrtQj EIXETO, ijveiKE, 7jh)fTaro, TTIPE, TrvOivBaij TaOeiQ* [Sir, must be done correctly, this: Give tense mood voice, and principal parts, viz., first per­ son singular present J'uture perfect and second aorist active and perfect passive if in use, if not, of the present j'uture perfect and aorist employed."]

3. (a) Decline avroc, fleXriwy, pyriop. {b) Point out any irregularity of form in Ityayot, fxaXXop, TtQvaiqVj w0e\Y/K7/. ORDINARY EXAMINATIONS, F.T. 1871. clxxiii

4. (a) Name two classes of verbs which in Greek govern the genitive. Give with their meanings two verbs of each class. {b) Name two uses of the dative case in Greek, and give an example of each use.

5. (a) Quote or construct an example of the use of npuc with each of the three cases it governs, and give the English. (J) How does the meaning of rat vaiy with the position of the article ?

6. (a) What are the three orders and the three kinds of time which a Greek verb distinguishes 03- its tenses ; and 03' what tenses are the3r severalty denoted ? (J) What are verbals in Greek ? How are the3r formed and with what meaning ?

7. (a) Give the meaning of the following Greek ter­ minations forming nouns, and an example of each, -toy, -f.we, -rijptor, -T>IQ (feminine). (J) What is the meaning of the following verbal roots in Greek ? and what the form of the present in which they are found ? Give one or two English words embod3ing each. 6s-, ffpay-, rep-, tpav-.

8. Translate and explain fully the construction in— (rt) TapajjLib'ari jiot TOaovroy ^povov. {b) tKavov olfia.i Trapiyojiat TOV fiaprvpa. (c) E-yw F i]Ttov at cuairra, {d) KaKocpyit]<: evcpyc.ai>] fiiy aficirwv. clxxiv EXAMINATION PAPERS,

9. {a) Where and what were Ithaca, Parnassus, Crete, Charybd is, {b) Who were Hades, Orpheus, Pelcides, Triptole- nms. 10. Derive, explain fulby the formation, and give the meaning of lupofiaTtiy, yeupytKos, kktraaiQ, w\i]fi- HeXtia, tTKtafta^cii; vrrrepoc, a/KpoTepuiBey, aiiroSi- d«A.TOf, tTTUtTTUip, CTTTTOpOrOC, TroXutllTTOf, TTToXtTTOpOoC.

JUNIOE LATIN.

CICERO, Speeches against, Catiline. VIRGIL, /Eneid VII. VIIL IX. SMITH, Student's Latin Grammar.

1. Translate literally cither (a) or {b), and either {c) or (a1)— (a) Quum a ine quoque id responsum tulisses, me nullo modo posse iisdem parietibus tuto esse tecum qui magno in periculo essem, quod iisdem mcanibus continereinur, ad Q. Metellnm pnetoiem venisti. A quo repudiatus, ad sodalem tuum, viruin optimum, AL Marcellum demigrasti: quern tu videlicet et ad custodiendum te diligentissi- mum, et ad suspicanduin sagacissimum, et ad vindicandum fortissimura fore putasti. Sed quam longe videtur a carcere atque a vinculis abesse debere, qui se ipse iam dignum custodia judicarit? Qiuc quum ita sint, Catilina, dubitas, si emori tequo animo non potes, abire in aliquas terras,.et vitam istam, multis suppliciis justis debitisque ereptam, fuga? solitudinique mandare ? ORDINARY EXAMINATIONS, F.T. 1871. clxxv

(b) Ilabetis ducem, memorem vestri, oblitum sui: quae non semper facultas datur: habetis omnes ordines, omnes homines, universum populum Ro- manum, id quod in civili causa hodierno die priinum videmus, nnum atque idem sentientera. Cogitate, qnantis laboribus fundatuni imperium, quanta viitute stabilitam lihertatem, quanta deo- rum benignitate aiictas exaggeratasque fortunas una nox pteno delerit. Id ne unquani posthac non modo confici, sed ne cogitari quidem possit a civibus, hodierno die providendum est. Atque htec non, ut vos, qui mihi studio pojne praicurritis, excitarem, locutus sum : sed ut mea vox, quas debet esse in republica princeps, officio functa consular! videretur.

(c) Hie velut pelagi rupes inmota resistit, Ut pelagi rupes magno veniente fragore, QUOJ sese, multis circum latiantibus undis, ATole tenet; scopuli nequiquametspumeacircum Saxa fremunt, laterique inlisa refunditur alga. Verum ubi nulla datur caecum exsuperarepotestas Consilium, et sa3V03 nutu Junonis eunt res, Multa deos aurasque pater testatus inanis, Frangimur lieu fatis,inquit,ferimuiquepi-ocella! Ipsi has sacrilog-o pendetis sanguine jicenas, 0 miseri. Te, Turne, nefas, te triste manebit Supplicium, votisque deos venerabere seris. Nam mihi paitaquies, omnisque in limine portus; Funere felici spolior. Nee plura locutus Saspsit se tectis, rerumque rcliquit habenas.

{it) Vix ea fatus erat: defixique era tenebant /Eneas Anchisiades et fidus Achates; Multaque dura suo tristi cum corde putabant, Ni signum ca;lo C3'therea dedisset aperto. dxxvi EXAMINATION PAPERS,

Namque inproviso vibratus ab rethere fulgor Cuin.sonitu venit, et mere omnia visa repontc, Tyii-lienusque tubre mugire per ojthera clangor. Siispiciunt; iterum atque iterum frag-or increpat ingcns. Anna inter nubem eojli in rcgione serona Per sudum rutilare vidont et pulsa tonare. Obstipuere animis alii; sed Troius heros Adgnovit soiiitum et divas promissa parentis. Turn inemorat: Ne vcro, hospes, ne quaere pro- fecto, Queincasumpoi-tentaferant: ego poseur Olympo. Hoc signmn cecinit ruissurain diva crcati-ix, Si bclliim ingiueret, Vulcaniaquearma per auras Laturani auxilio. Write down the first three lines of (c) and (a*). Mark the feet and the quality of each syllable. 2. (a) Parse those words—colubris, libidinum,miscent, orsa, propagarit, quiorunt, rere, stabula, sustu- lissein, venturum. {b) Note and explain anything peculiar in the form of sethera, capitoli, dcum, doiiiinarier, faxo. 3. (ft) Translate and explain fulhy the construction in (1) Huic urbi feno minifantem ejecimus, (2) cli- peum excipiam sorr.i, (3) Hano sine me spem f'ene tui, (4) Pnustanti corjiore Turnus toto verticc supra est. {b) State and explain b3' an example of each what is meant by a Partitive Genitive, an Ablative of Separation, an Accusative of closer Definition.

4. (ft) Give the meaning and the full derivation of egregius, gestamen, lanigcr, incassum, infitiator, parricida, pranexta, subjector, vinculum, vitisator. ORDINARY EXAMINATIONS, F.T. 1871. clxxvii

{b) What kinds of words are derived in Latin by the following terminations, -tura, -etum, -ax, -icius ? Give an example of each and stale the kind of word to which each termination is ap­ pended. 5. (a) Name two Latin substantives that have no plural, and two that have no singular. {b) What is the peculiarity of declension of acer ? Name two adjectives similar!}7 declined. (c) What are Neuter-Passive Verbs ? Why are they so called ? By what other name are they known ? (a*) Name with their meanings two Latin adversa­ tive and two concessive conjunctions. {e) Give the 1st person perfect of allicio, domo, premo, plico, parco, sto, tergo.

6. (a) Name and define the elementary parts of a sentence. {b) Name two classes of verbs which in Latin take a double accusative. What are the accusatives ? (c) By what particles and with what differences of meaning are single direct questions asked ? {d) Name two of the changes of mood necessary for passing from oratio recta to oratio obliqua. 7. Give the Latin for each of the following, and state the rule of which it is an example— (a) Justice towards the gods is called religion. {b) I am vexed at 103' own folly, (c) It is asked whether the3r are gods or no. Id) They sent ambassadors to ask help from the Romans. hS clxxviii EXAMINATION PAPERS,

8. Translate and explain— (a) Mceonise delecta juvenilis, Etrusca acies. {b) Talis in Euboico Cumarum litore quondam Saxea pila cadit. (e) Accelerant acta testudine Volsci. (a7) Tertia fere vigilia exacta. (e) Signum ostendimus, cognovit: linum incidimus, legimus.

9. (a) Who were the following—Acrisioneis, Cinna, Circe, C3 therea, Perses, Pompeius ? {b) What and where were Ardea, Aurelia, Massilia, Mulvius, Proclyta, Syrtes ?

ENGLISH AND LOGIC.-PART I.

1. (a) "The Reduplication of the Consonant after a vowel is in English usually not real but only an orthographic expedient." Explain this, and point out by examples what the doubled consonant denotes. {b) Name with examples of their use the fourteen mutes in English.

2. (a) Give an example of an English derivative formed b}' each of the following suffixes -th, -ly, -ish, -kin. State what sort of words is formed hj each. {b) " Which is not the neuter of who." Explain this statement, and give the origin of the word. ORDINARY EXAMINATIONS, F.T. 1871. clxxix

3. (a) What are the two views of the formation of such a word as foremost ? Which is according to Latham correct ? (Z>) " Thou sangest is a form to the evolution of which two irregularities have contributed." Ex­ plain what these are.

4. (a) What are Indeterminate Propositions ? Give examples of the different wa3'S in which different nations express them. {b) What is the construction in (1)1 am to blame, (2) Myself is weak, (3) The brothers killed each other.

5. (a) With what single foreign element does Craik, admit that the Gothic basis of our language has combined freely ? {b) What time is called the Period of Middle English and what are its leading characteristics ?

6. (ft) What are according to Craik the subdivisions of the Gothic branch of Language ? {h) Give the modern English for— (1) lie lidh inne unforbaerned mid his magum. (2) Ich wene ther be ne man in world contrey^s none That ne holdeth to her kunde speche bote Engelond one. To what date would you refer each and why ?

7. (a) State and explain the distinction drawn by Thomson between pure and applied Logic. {b) Explain what is meant b3r calling Logic the " Orjranon." dxXX EXAMINATION PAPERS,

8 (a) " Summum genus and Tnfima species are fixed somewhat arbitrarily." Explain what Thomson means by this. {b) Explain Thomson's division of Nouns into Proper, Singular, and Common.

9. (a) What are the four relations as stated by Aristotle in one of which every predicate must stand to its subject ? Define each and construct an example. {b) Give examples of the judgments called by Thomson U and Y. Wherein do they respec­ tively differ from the old A and I ?

10. (a) What are the three principal defects incident to reasoning ? Against which does Logic guard directly ? {b) Wherein are Privative an3 Negative terms alike and different according to Whately ?

11. (a) Give Whately's several divisions of Propo­ sition, and the principle of each division. {b) Give what Whately would call a logical de­ finition of Fork and of Chair.

12. (a) Which of the four forms of proposition dis­ tribute the Predicate? Why cannot 0 be simply converted ? {b) State and prove the Law of Contrary Oppo­ sition. ORDINARY EXAMINATIONS, F.T. 1871. dxxxi

ENGLISH AND LOGIC—PART. II.

CRAIK, English of Shakespeare. WHATELY, Rhetoric. WHATELY, Logic. THOMSON, Laws of Thought.

1. Give the substance of Craik's notes on (a) Merely upon myself. {b) You ought not walk. (c) That eye did lose his lustre. {d) Brutus is ascended.

2. In what senses now forgotten are the following words used in the Julius Cassar—addressed, con­ ceit, favour, instance, resolved, respect ? Quote passages in which they occur.

3. Certain emendations have been proposed in the following, what and why ? (a) That her wide walks encompassed but one man. (6) What touches ourself shall be last served. {c) A curse shall light upon the loins of men.

4. Give the etymology of these words according to Craik—charm, clever, errand, nice, orchard, which.

5. (a) Wliat is Whately's estimate of Cicero as a writer on Rhetoric ? {b) Distinguish the wider from the narrower use of the term conviction. clxxxii EXAMINATION PAPERS,

C. (a) Give a general sketch of Whately's views as to the arrangement of arguments. (4) Quote or construct examples of the Personify­ ing Metaphor.

7. (a) What are the three views as to the origin of Language given by Thomson ? {/>) " For all Logical Purposes the Hypothetical Judgment may be treated as a Categorical." Explain and prove this dictum of Thomson's.

8. (a) How does Thomson distinguish Symbolical from Notative Conceptions ? (5) Give an example of Immediate Inference by Complex Conceptions. Justify the process.

9. (a) What is analogy ? How may it be employed as an argument ? State the canon given for its use by Thomson. {b) State and explain the principle of the Middle being excluded.

10. (a) Give Whately's definition of a Fallacy. Con­ struct examples of the Fallacia Accidentis, and of Petitio Principii. {b) What is meant by Reduction ? What is sup­ posed to be the use of the process ? Construct a Syllogism in Felapton and reduce it.

11. (a) State Whately's six; laws of Syllogism. {b) Prove that in every figure the mood IEO is invalid. ORDINARY EXAMINATIONS, F.T. 1871. dxxxiii

12. Put into strict logical form and examine these arguments— (a) All those must disapprove of punishing this woman w-ho consider her innocent: as 3-011 dis­ approve of punishing her we may fairly infer that 3TOII consider her innocent. {b) He who calls you a man speaks truly : he who calls 3rou a fool calls you a man, therefore he who calls you a fool speaks truly.

Professor Wilson. GEOMETRY. Four of these must be done. Algebraical Symbols must not be used. 1. If the square described upon one side of a triangle is equal to the squares described upon the other two sides the angle contained b3r these two sides is a right" angle. 2. If a straight line drawn through the center of a circle bisects a straight, line in it which does not pass through the center it shall cut it at right angles; and if it cuts it at right angles it shall bisect it. 3. From a given point outside a given circle to draw a straight line which shall touch the circle. 4. The angle at the center of a circle is double the angle at the circumference upon the same base. clxxxiv EXAMINATION PAPERS,

5. To describe a circle about a given triangle.

6. If the sides of two triangles about each of their angles be proportionals the triangles shall be equiangular and the equal angles shall be those which are opposite to the homologous sides.

7. Similar triangles are to one another in the duplicate ratio of their homologous sides.

8. If two straight lines be cut by parallel planes they shall be cut in the same ratio.

TRIGONOMETRY. Four of these must be done. 9. Express the secant of an angle in terms of the sine of the same angle.

10. The sines and the cosines of A and B being given investigate a formula for sin {A + B). A 11. Investigate a formula expressing sin -~ in terms of sin A.

12. Find the numerical values of the sines and the cosines of 45° and of 60° and deduce the value of sin 15°.

13. Shew that in any^ triangle A — B a — b A + B tan —o— = • •——T tan —~ 2 a + b 2 ORDINARY EXAMINATIONS, F.T. 1871. dxxXV

14. Explain precisely in what case there will be two different triangles having tho same parts a, b, Ii.

15. Find the area of a regular polygon of 12 sides inscribed in a circle whose radius is 12 feet.

16. Find au expression for the radius of a circle in­ scribed in a given triangle.

ALGEBRA.

Professor Wilson.

Eecry result must be reduced to its simplest form.

1. Find the Lowest Common Multiple of x2 — 4a2, x3 + 2ax- + \a"x + Sa3 and w3 — 2ax" + iaaw — 8a3.

2. Find the square root of x {x* + 4) - (*•» + 2) + J + £..

3. Shew that

=«+(7+i)(7+-:)(i+-i) dxxxvi EXAMINATION PAPERS,

4. Substitute ^—;—; for x and -; 5 for u in the 1 + ab 1 — ab a x + y , ® — y expressions -, and -\—; i 1 — *y 1 + a.y 5. Eliminate a; from the equations ax + by + c = 0 and ^.r + qy + r — 0.

6. Find the square root of 10 — 4 VlG.

?. Shew that if the roots of the equation x2 + px + ij — 0 are impossible, tho expression .r- + px + q can­ not be negative for any real value of*.

8. Solve the equation a-4 — 14a;- +40 — 0.

9. Solve the equation 25.r- — 16 _ Six2 -4) "TO x + 8 - ~2x'"—±

10. Solve the equations x — 2y + Sz = G 2x + Sy — 4* = 20 Sx - 2y + 5.- = 20

11. Solve the equations a-2 + Sxy = 54 xy + iy2 = 115

12. When a mass moves with any velocity its energy varies as the weight and the square of the velocity jointly, and the velocity acquired in falling ORDINARY EXAMINATIONS, F.T. 1871. dxxxvii

through an\' height varies as the square root of the height: Supposing a btxh' to acquire a velocity of 32 feet a second in falling 10 feet, through what height must a weight of one ton fall that it ma3r acquire the same energy as 1 lb. moving with a velocit}' of 1120 feet a second.

13. The sum of n terms of a series of quantities in 3w8 Un arithmetica• , • l ,progressio n is. r+j ' fin. d, th, e first term and the common difference.

14. Investigate an expression for the number of per­ mutations of n things r together.

15. The number of permutations of n things r together is 20 times the number of permutations r — 2 together; the number of combinations r together is the same as the number *• — 2 together : Find n and r.

DIFFEEENTIAL CALCULUS AND ANALYTICAL GEOMETRY.

Professor Wilson. 1. Differentiate (a + bx + ex2)* c / . x\s / x \5i \ Lr+^+7.-'.{(Bini) + (cosir)} log tan {a + b cos x)

2. Differentiate x" + y', x and y being independent. Clxxxviii EXAMINATION PAPERS,

3. State and prove Leibnitz' theorem.

4. Find the n'h differential coefficient of tan x.

TC0S 5. Expand E ". Cos {x sin a) iu a series of powers of a.-.

0. Investigate the method of determining the maxi mum and minimum values of a function of one independent variable.

7. Explain fully what is represented by the equation obtained by eliminating a from the equations to two curves f{x, y, a) = 0

8. Find the locus of the intersections of two straight lines which pass through two fixed points and cut at a constant angle.

9. Find the locus of a point the difference of whose distances from two fixed points is constant. From the equation deduce ihe general form of the curve.

10. Find expressions for the siibtangent and subnormal of the curve y =f{x).

11. Find an expression for the tangent of the angle between the radius vector and tangent of the curve r =if (fl).

12. Inscribe the greatest parabola in a given isosceles triangle, the axis of the parabola bisecting the vertical angle of the triangle. ORDINARY EXAMINATIONS, F.T. 1871. dxxxix

7 13. Integrate T~^~ "v ^Va- — a--, sin x

a cos x — b sin x

14. Find the volume of the solid generated by the revolution of an ellipse round a tangent at the extremity of the axis major.

NATURAL PHILOSOPHY.

Professor Wilson.

Write down the general expression for the magni­ tude of the resultant of two forces P and Q which act at one point in directions inclined at an angle 0 to one another. Find the magnitude of the resultant of two forces one of 36 lbs. acting horizontal!}' and the other of 15 lbs. acting vertically upwards, and the tangent of the angle which its direction makes with a vertical line.

O Explain what is meant by the coefficient of friction. A heavy particle is placed on a rough sphere at such a distance from the highest point that it is on the point of slipping down; what angle does that distance subtend at the center of the sphere, the coefficient of friction being 1 ?

3. Investigate the magnitude and the line of action of the resultant of two parallel forces acting in CXC EXAMINATION PAPERS,

opposite directions at different points of a rigid body : and shew from your result that if the forces are equal the}' have no single resultant.

4. What is meant by the moment of a force about a point? State the necessary and sufficient condi­ tions of equilibrium of any number offerees acting- in one plane on a free rigid body. If the body instead of being free can turn about one point which is fixed how are the conditions altered ?

5, Investigate the position of the center of gravity of a pyramid.

6. Explain the terms "work" "energy" "potential energy" "kinetic energy." A body is projected vertically upward, with a given velocity, find expressions for its potential energy and its kinetic energy in any position and shew that their sum remains constant.

7. A fly-wheel whose weight is one ton, and whose whole mass may be considered as collected in its circumference which is 32 feet makes one revolu­ tion in a second : through what height would the energy existing in it suffice to raise a weight of one hundredweight ?

8. Investigate the relation between the force and the velocit3' when a body moves in a circle with uni­ form velocity.

9. How is the pressure at a point of a fluid expressed ? Explain the meaning of the term density. Find an expiession for the pressure at a depth z below the free surface of a liquid of given density. ORDINARY EXAMINATIONS, F.T. 1871. CXC1

10. Shew that if two liquids which do not mix meet in a bent tube the heights of their free surfaces above their common surface will be inversely as their densities.

11. A portion of a solid is immersed in a liquid : find the resultant fluid pressure on the immersed portion 1st neglecting the atmospheric pressure; 2nd taking account of it. Deduce the conditions that must be satisfied when the solid floats in equilibrium.

12. Explain the method of finding the specific gravity of a solid b}' weighing it in air and in water, and construct a numerical example.

13. Write down the formula connecting together the pressure the density and the temperature of a gas : which of the two constants depends on the nature of the gas ? What is the numerical value of the other constant using the centigrade scale ?

14. Explain carefully in what particulars the vapour of a liquid differs from a so-called permanent gas.

15. Enumerate the principal parts in a stationary con­ densing engine, explain their use, and describe the action of the engine.

16. Give a general description of the circulation of the atmosphere, and explain the origin of the Trade Winds. CXCI1 EXAMINATION PAPERS,

CHEMISTRY AND MINERALOGY.

Professor McCoy.

1. Explain clearly the doctrine of combination by multiple proportion, giving examples with the characteristic quantities of the substances. 2. Explain the relations and differences between chemical and the various physical forces bringing bodies together. 3. What elements are Diads and WI13' are they so called ? 4. What are the Hydracids ? 5. What arc the chief products of the distillation of Coal at a red heat ? 6. Give the chemical and physical characters of Graphite and the usual modes of occurrence of the mineral. 7. Suppose a crystal of garnet to present the faces belonging to the Rhombic Dodecahedron, the cube, and the octohedron, how would 3-011 re­ cognize each of them ? 8. How are Diclinic distinguished from Triclinic Crystals ? 9. What systems of crystals are biaxial nnd how would you determine the fact ? 10. What is the relation and the difference between the two minerals Arragonite and Calc Spar ? ORDINARY' EXAMINATIONS, F.T. 1871. CXCiii

STRUCTURAL AND PHYSIOLOGICAL BOTANY.

Professor Mc Coy.

1. State the nature, force, and functions of Endosmose and Exosmose in Plants. 2. What is the structure called glandular woody tissue, and in what plants may it be found ?

3. State the characters and functions of the different kinds of starch, and the particular parts of the plants in which each may be found.

4. Describe the three main ways in which roots originate, and give some examples of the plants in which each ma}' be found. 5. Describe the mode of formation of each of the tissues in an Acrogenous stem.

6. Shew the difference between a Corymb, simple and compound Umbel, and a Cyme, mentioning some examples of each. 7. Describe the parts of a complete symmetrical flower. 8. What do you understand by Diclinous, Monoecious, and Dioecious Flowers ? 9. Describe the different methods of opening in An­ thers, giving examples of plants showing each.

10. Describe the structure of ovule found in the Mis­ tletoe. i CXC1V EXAMINATION PAPERS,

COMPARATIVE ANATOMY AND ZOOLOGY.

VERTEBRATA.

Professor McCoy. -

1. In most mammalia the toes of the hind foot have a fixed relation to certain bones of the Tarsus. From which of those does the third toe originate, which supports the second, and how are the fourth and fifth supported ? 2. Describe the relations of the Neural spines of the four cephalic vertebne to the adjoining bones of the skull. 3. Describe the earlier stag-es of the formation of the Vertebrate skeleton in the Ovum. 4. Describe the respiratory system in Fish, with the more nearly related bones and blood vessels. 5. Describe the digestive system in the higher mam­ malia. 6. Define the Orders of Birds.

7. Define the Families of the Lophobranchiate Fishes. 8. Define the Families of Chelonian Reptiles. 9. Contrast the anatomical and external characters of the Cetacea and Carnivora. 10. Describe the numbers, mode of succession, func­ tions, external characters, and internal structure of all the kinds of teeth found in the Primates. ORDINARY EXAMINATIONS, F.T. 1871. CXCV

GEOLOGY AND PALEONTOLOGY.

Professor McCoy.

1. Give the English subdivisions of the Cretaceous formations, with their true relative place, and the palaeontological characters of each.

2. Write down in correct order of superposition all the formations in which species of Ammonites occur.

3. What formations are indicated by (a) Trinucleus, {b) Cr toreros, (c) Zeuglodtm, (a7) Ceratites, {e) Dupednim ?

4. What is the range in time of (a) Orthis, {b) Tri- gonia, {c) Clymene, {d) Nautilus?

5. By what fossils could yon distinguish (a) Aymestry limestone, {b) Mountain limestone, {c) Purbeck limestone from each other ? fi. What are the chief characteristics of Granites in position, mineral character, ami relation to other rocks ?

7. How do yon account for the newer igneous rocks often bavins' a liijj-her density than the older ones ?

8. How do you proceed to make a geological section from a good geological map ? i2 CXCV1 EXAMINATION PAPERS,

9. What signs are used on geological maps to indi­ cate all tho varieties of position of tho formations and their relations to each other ?

10. Make a rough geological section showing the place of all the European Mesozoic formations.

ANCIENT HISTORY.

Professor Hcarn.

1. State the duties of the several Praetors, and the dates of the creation of the office and of the in­ crease in the number of persons holding it. 2. Explain the origin of the expression "The Classic Authors."

3. Explain the origin of the expression " Conscript Fathers."

4. What was the difference between Senatus-Con- sultum and Senatus-Auctoritas ?

5. Explain the terms Vasarium and Salarium, and trace the history of the latter.

6. How was Julius Caesar personally connected with the popular party ?

7. What in Air. Merivale's opinion would have been the probable consequences of the success of Antony at Actium ? ORDINARY EXAMINATIONS, F.T. 1871. CXCvii

8. When and in what circumstances did the Gothic Kingdom of Italy commence ?

9. When and in what circumstances was the Gothic Kingdom of Italy overthrown ?

10. When and in what circumstances did the Lombard Kingdom of Italy commence ?

11. When and in what circumstances was the Lombard Kingdom of Italy overthrown ?

12. State the dates, the places, and the circumstances of the following battles :—Vercellne, Coracesium, Winfeld (The " Herrmanschladt"), Chrysopolis, Pollentia.

HISTORY OF THE BRITISH EMPIRE.

Professor Hearn.

1. What were the causes that led to the increase of the Royal authority in Western Europe ? 2. What were the circumstances that led to the decay of the system of the Feudal Militia ? 3. State exactly the meaning of the following terms, . viz.: — Alodial, Commendation, Eorlcundman, Fief, Lcudes. 4. What does Air. Hallam consider to have been the principal advantages due to the Norman Con­ quest ? CXCV111 EXAMINATION PAPERS,

5. At what time and from what causes did the dis­ tinction between the Normans and the Saxons disappear ?

6. What was the peculiar character of the contest of the Barons with the Crown in England as com­ pared with the corresponding contest in France and in Germany ?

7. Who is described as "The Father of English Commerce," and what are his claims to that title ?

8. What were the essential checks upon the authority of the Crown at the accession of the Tudors ?

9. What distinction does Air. Hallam notice between the historical value of tho chronicles that preceded and those that followed the Reformation ?

10. Mr. Hallam observes that the principal statutes of the Lung Parliament suggest two remarks of no small importance. What are these remarks?

11. What was the immediate occasion of the Civil War, and what wore the circumstances that at that time excited suspicions of the King's sin­ cerity ?

12. What changes in the character of the polity of England occurred during the administration of Oliver Cromwell ? ORDINARY EXAMINATIONS, F.T. 1871. cxcix

CONSTITUTIONAL AND LEGAL HISTORY.

Professor Hearn.

1. How was prerogative strained for the punishment of Wilkes ? 2. How was privilege perverted for the punishment of Wilkes ?

3. Explain the nature of ex-officio informations, and their unpopularity.

4. Sir Erskine May observes that in the reign of George HI. two doctrines were maintained b3r the Courts which excepted libels from the general principles of our criminal law. What were their principles, and when were the3r altered ?

5. From what period may the use of political carica­ tures bo dated, and who was tho first celebrated artist of this class ?

6. Trace the history of fiscal legislation as it affected tho Press.

7. What parallels, if any, does our law present to the doctrine of the immunity of the Sovereign from legal process ?

8. An Act of Parliament directed that certain offend­ ers should be fined " at the will of the King." How in such circumstances must the fine be im­ posed, and why ? CC EXAMINATION PAPERS,

9. What in was meant by Prevarication ? In what circumstances did a similar danger pre­ sent itself in English law, and how was it met ?

10. What is the principle as to "open questions" in a ministry ?

11. State the circumstances of what is known as "The Bedchamber Question," and explain the principle that it involves.

12. Explain the origin of the legal title "Queen's Counsel." Who was the first person who ob­ tained this distinction in its modern sense ?

POLITICAL ECONOMY.

Professor Hearn.

1. What do you understand b3r the term labour ?

2. What physical explanation may be given of the superiority of voluntary over compulsory labour ? 3. Trace the influence of irregularity of employment upon (a) the remuneration of the workman; {b) his moral nature; {c) the quality of his labour. 4. Trace the influence of general education upon the progress of invention. ORDINARY EXAMINATIONS, F.T. 1871. Cci

6. What physical conditions in an unimproved country are most favourable to the growth of trade ?

6. Is every real invention necessarily beneficial ?

7. In what circumstances is the tendency to a separa­ tion of occupations strongest ?

8. Is the permanent increase in the cost of any article a good or an evil to the community ?

9. Trace the influence of co-operation upon capital.

10. State some of the circumstances that usually de­ termine the direction of national industry. 11. What is meant by the expression "real wages" ? What place, if any, does this subject occupy in the theory of Political Economy ?

12. Trace and account for the analogy between roads and money.

L A W.—PABT I.

Dr. Dobson.

Cite any cases which you may remember as authorities in support of your answers.

1. Who may bo parties to contracts ? What classes of persons labour under disability to bind them­ selves by contract ? iS CC11 EXAMINATION PAPERS,

2. How far may a general agent, bind his principal ? How far may a. particular agent do so ? Give a reason for the difference in their authority.

3 What is a contract, of insurance ? What arc the three kinds of insurance of most frequent occur­ rence ?

4. Explain the terms general average, particular average, demurrage. What different meanings has the word freight ?

5. What is meant by a wager policy ?

6. How far will the Courts entertain the question of adequacy of consideration for contracts ?

7. What distinction is there in the effect of contracts tending to a future separation between man and wife, and a contract contemplating immediate separation ?

8. " I do hereby promise Airs. Catherine Lowe that I will not marry any person besides herself. If I do 1 agree to pay her £1,000 within three months after I shall marry anyone else." Upon what ground did the Court hold this contract to be void ?

9. On what grounds is a contract illeaal at common law ?

10. What is the distinction between remunerated and unreniuncratod agents ?

11. What is an escrow ? ORDINARY EXAMINATIONS, F.T. 1871. CC111

12. Explain the difference between specialty and sim­ ple contract debts ?

13. A is indebted to B in a certain sum, and Cis his surety. If C be compelled to pay, what is the legal effect upon A ?

14. "Exnudo pacto non oritur actio." Explain and illustrate this maxim.

15. What is the doctrine of estoppel ?

16. How far may an infant bind himself by a contract ? 17. What were the objects of the Statute of Frauds ? Write out the fourth section of the Statute, and state the effect of the introduction of the word "agreement" into that section.

18. "Ex turpi causa, non oritur actio." Illustrate and explain this maxim.

19. Supposing- on a sale of goods under the seventeenth section of the Statute of Frauds there is no writing sufficient to-satisfy the Statute, in what other manner may the contract be proved ?

20. Define a consideration. For the validity of what contracts is the existence of a consideration un­ necessary ? CC1V EXAMINATION PAPERS,

LAW.—PAET II.

Mr. Billing.

1. Is the liability of an assignee different in the case of covenants in g-ross and covenants running with the land ? Give the answer fully. 2. Suppose a man demise land generally at a rent without saying how long the demise shall last, what estate has the lessee ?

3. In what instances has the common law rule re- regarding the assignment of choses in action been changed bv statute ?

4. Suppose a tenant were to attorn to a stranger claiming title to the lands, would the landlord's rights be affected ? State the statute law on the -subject.

5. In what case does a tenant holding over pay double value and in what double rent ?

6. What is an estate in reversion ? How does it arise ?

7. An estate is devised to A to the use of Ii to the use of C. Who has tho legal estate ? 8. What is the position of executors under the Statute of Wills in relation to the residue not expressly disposed of? '

9. What is considered as included in the premises of a deed ? ORDINARY EXAMINATIONS, F.T. 1871. CCV

10. Is it necessary to have declarations of trust of money in writing ?

11. What is the rule as to bargains with reversioners or remaindermen for the property to which they are thus entitled ?

12. State fully the law as to notice as regards the rea-istration of deeds.

LAW.—PAET III.

Mr. Billing.

1. What defence may be set up under a plea dem'ing the invalidity of au award ?

2. What does the plea of never indebted to an action on an attorney's bill put in issue ?

3. In what form of action is money deposited with a banker recoverable ?

4. What is it necessary for a plaintiff to prove in an action by vendor v. vendee of real property, if non assumpsit should be pleaded ?

5. Are there an3' degrees in secondary evidence ?

6. If a document is not produced after notice to pro­ duce served on the opposite party, what is the course to be adopted by the side who has served the notice ? CCV1 EXAMINATION PAPERS,

7. Is there any and what difference if a document is not produced by a person who has been served with subpoena duces tecum ?

8. Give examples of cases in which the usage of trade is admissible in evidence and where it is not.

9. Is an admission made by a person under compul­ sion, e.g., an answer in Equity, binding on him?

10. What way is a record proved on an issue of nul tiel record ?

11. In what way are foreign laws to be proved ?

12. What is the present law as to proof of deeds by attesting witnesses ? State the answer fully.

LAW.—PAKT IV.

Dr. Dobson.

Cite any cases which you remember as authorities in support of your answers.

1. Who may be made trustees ?

2. A is named executor of a will, and also a. trustee by the terms of the will. If he proves the will, what liabilities docs he incur ?

3. If a man appoints a trustee of real or personal ORDINARY EXAMINATIONS, F.T. 1871. CCVli

estate, without naming the heir or personal re­ presentative of such trustee, does the heir or personal representative become a trustee ?

4. When once a trustee has taken upon himself that character, how can he denude himself of it ?

.5. If the will contain no provision for the remuneration of the trustees appointed b3' it, can they claim any remuneration for the performance of their duties ?

C. A child recently after attaining majority makes over property to his father, either without con­ sideration, or for an inadequate one. What must the father prove before the Court will uphold the transaction ?

7. An agent sells his own property to his principal as the property of another without disclosing the fact. What rights has the principal on dis- • covering- the true nature of the transaction ?

8. What is the general rule as to a trustee obtaining profit or advantage to himself in managing the affairs of his cestui-quc-trust ?

9. If a creditor stipulates with his debtor in a binding manner, and upon a sufficient consideration to give further time for payment, what will be the effect upon the liability of a person who has pre­ viously become surety for the debt ?

10. A clear mistake appears on the face of a will. Will equity rectify it ? Is parol evidence generally admissible in such cases <' CCV111 EXAMINATION PAPERS,

11. An instrument is so general in its terms as to release the rights of a party to property to which he was wholly ignorant that ho had aii3' title, and which was not within the contemplation of the bargain. Will equity relieve in this case ?

12.. A preliminary contract or articles differs from the instrument of conveyance or settlement as finally executed—(i.) the final instrument contains a recital that it is made in pursuance of the original contract—(ii.) there is no such recital. On what grounds will the Court refuse to interfere with tho final instrument in the second case ?

13. Define a mistake as remediable in equit3r. What circumstances will induce the Court to grant relief where the mistake is that of the person seeking its aid ?

14. When will the Court grant relief on the ground of accident against—(i.) the defective execution, and (ii.) the non-execution of a power ?

15. Illustrate and explain tho maxim, Equity follows the law.

16. There is a contract between A and B—(i.) to enter into partnership simply, (ii.) to enter into partnership for three 3-ears. Will the Court decree specific performance of either or both contracts ? Give 3-0111- reasons.

17. Upon what ground is the jurisdiction of the Court of Equity practically exclusive of that of the Court of Common Law in questions of account ? ORDINARY EXAMINATIONS, F.T. 1871. CC1X

18. Explain the terms legal assets, equitable assets, marshalling of assets. 19. Explain what is meant by an equitable mortgage. How does it differ from an ordinary mortgage of realty ? 20. In what respect do liens in equity differ from liens at law ? What is the usual mode of enforcing a lien in equity ?

JUNIOR DESCRIPTIVE AND SURGICAL ANATOMY. Professor Halford. WRITTEN. Describe the following ligaments with their functions and relations:— The internal lateral of the lower jaw, the conoid, the trapezoid, the coraco-acromial, the orbicular, the external lateral of the wrist, the greater and the lesser sacro-sciatic, the crucial, the internal lateral of the ankle joint, and the occipito-axoid. Describe the following muscles, their actions, and relations : — The sterno-cleido-mastoid, the trapezius, the digastric, the platysma, the sterno-hyoid, the sterno-thyroid, the deltoid, the biceps cubiti, the supinator longus, the extensor communis digit- orum, the flexor longus pollicis manus, the lum- bricales, the transversalis abdominis, the cremaster, the adductor magnus, the peroneus longus, and the levator ani. CCX EXAMINATION PAPERS,

ORAL. 1. Describe the following bones :— The Frontal, the Occipital, the Sphenoid, the Lower ALixillary, the Ischium, the Tibia, and the Internal Cuneiform. 2. Give the steps of a dissection of the muscles of the axillary space and of the walls of the abdomen, describing minutely the origin, insertion and relations of any that may be required.

SENIOR DESCRIPTIVE AND SURGICAL ANATOMY.

Professor Halford.

WRITTEN. 1. Describe the process of development of the following bones :—The Occipital, the Sphenoid, the Typical lumbar Vertebra, and tho Femur.

2. Give tho origin, course and relations of the following blood-vessels:—Arteries: The Phrenic,the Renal, the Subclavian, the External Carotid, and the Peroneal. Veins: The Internal and External Jugular, the Femoral, the External, Internal and Common Iliacs, and the Portal.

3. Mention the trunks whence are derived the cutaneous nerves of the Head, Face, Walls of the Chest and Abdomen. ORDINARY EXAMINATIONS, F.T. 187!. CCxi

ORAL.

Give the steps of a dissection of the following regions, answering an}' question that may be put to 3-011 relating to 0113- of the structures met with :— The anterior triangle of the Neck, The upper third of the Thigh, The Perineum, and The Orbit.

GENERAL ANATOMY, PHYSIOLOGY, AND PATHOLOGY.

THIRD AND FOURTH TEARS. Professor Halford.

WRITTEN. 1. Describe the minute structure of Cartilage, Bone, and Muscular Fibre.

2. Describe the various forms of Epithelium, and the sources from which they are obtained.

3. Give a sketch of the Lymphatic and Lacteal Vessels and Glands, and in addition the source, nature, destination, and uses of their contents.

4. Describe the mechanism of the Heart's action.

5. What are the microscopic appearances of Cancerous and Recurring fibroid tumours. CCX11 EXAMINATION PAPERS,

ORAL.

This examination is upon the written papers of the Candidates in these subjects.

CHEMISTRY (MEDICAL).

John Dntmmond Kirhland.

[N.B.—In describing tlie preparation of substances, gice the equations used to explain the various reactions.]

1. Give the principal properties, physical and chemical, of the following, viz.:— I. Carbon. II. Chlorine. in. Sulphur. iv. Phosphorus. 2. Select any one of the above and describe the prepa­ ration and properties, physical and chemical, of its more important combinations with Oxygen and with Hydrogen.

3. Define the following terms, viz.:— i. Atom, Alolecule. ii. Homologous, Heterologous. in. Acid, Base, Salt. iv. Mixture, Compound. v. Aletameric, Pol3Tmeric. ORDINARY EXAMINATIONS, F.T. 1871. CCxiii

4. Give some proofs to show that "atmospheric air" is a mixture and not a compound. 5. How is sugar converted into alcohol ? Represent the reaction in an equation. 6. Describe the preparation of the following, viz.:— i. Glucose. II. Urea (artificial), in. Ether, iv. Bromine. v. Iodic acid.

PRACTICAL CHEMISTRY.

John Drummond Kirhland.

\N.B.— The means by which the several results were obtained must be written out. Symbols may be used. Any calculations necessary to be given in full] 1. Qualitatively analyze any one of the substances marked respectively A, B, C.(a) 2. Examine the samples of Urine marked D and E respectively for albumen and glucose. Alention any sources of error in the processes ordinarily used. 3. Qualitatively analyze the liquid marked E. (»

(a) As03. I'bO.Ac. K0,2T". (6) Hydrochloric solution of Arsculc. CCXIV EXAMINATION PAPERS,

4. 'Determine the specific gravity of wood from the following data:— Weight of wood in air ... 25-35 grs. „ a copper sinker in air - - 11 '00 ,, „ wood and sinker under water 5-10 ,, ,, copper sinker under water - 977 „

SURGERY. Edmd. Barker, M.D., F.R C.S. Eng. 1. Describe the local condition and" general symptoms of a case of Phlegmonous erysipelas of the fore arm, and state the local and general treatment of such a case, giving the reasons for its adoption. 2. How are fractures at the base of the cranium caused? What symptoms usually accompany those inju­ ries ? State the treatment to be adopted, and the probable or possible result. 3. What is Pyiemia? its causes, symptoms, and treat­ ment? 4. When the femur is broken a little below the lesser trochanter, what is the position of the upper end of the lower fragment'! To the action of what muscles is the distortion of each of them arising ? 5. What are the pathological conditions which give rise to senile gangrene? In what respect and for what reasons would the treatment of a case of traumatic gangrene of the leg- differ from that of senile jjamrrene? 6. Define an acute abscess of the arm, giving its symptoms, also the local and general treatment. DEGREE OF M.A., F.T. 1871. CCXV

EXAMINATION FOR THE DEGREE OF MASTER OF ARTS.

SCHOOL OF LANGUAGE AND LOGIC.

PLATO, Republic. , Books VI.-X. 1. Translate— till S' av ?/ iHfivala, yv Xtyui, rotaot jxdXtara, tt cti/roH' yivotTO, o'iav nori (pant ovvufiiv rj) Tvyrj rij) TOV AvOov irpoyovoi ytviaSai. ttvat fiiv yap avrov

irot/itva ^rfrtOovra irapa roi rort AvSiac u.p\ovTi} ofiftpov oi TTOXXOV yerofiivov Kal attirfiov payijvai n ri]Q yijc Kal ytviaBat j(aiT/ia Kara, TOV TUKOV, 7; fvtfity. tSoi'Ta it Kal Savfiatravra KUTCifiijyai Kal Ictiv aXXa Tt 01) fiv$oXuyoiicri $av/iaaTa Kal 'ITTTVOV ^(IXKOVI' KO'IXOV, BvpiCatj t\0VTa, Ka^ ac iyKv'ipavra. ictiy tvovra vtKpdv, (is (jiatvtrrBat, f*ti£iii y Kar' av- Spwitov' rovTov Si aXXo fiiy o'vciy t\uv, irtpl Si rij \ttpl y_pvaouv SaKTvXtov, ov trtpitX6j.ityov fV/3»7''Ct(. trvXXoyov Si ytvofxivov TOIIJ Trot^iirty tiuiBoTuc, tv' i^ayyiXXottv Kara /jf]va rip j3airtXti ra Trtpl ra noifi- vta, a(f>iKta$'nt Kal tKilyov t^oi'Ta TUV CaKTvXwv. Ka&t'ifitvot' ovv fjtTa TWV aXXuiv rv^tiv TI)V tT(ptvS6vi]y TOV caKTvXiov Trtptayayvvra Trpbc tavrov tic TO t'itrw rijc \ttp6c' TOVTOV oi ytvofiivov CKpavy avrov ytria- Sai rote 7rapaKa$7]fiiyot(:, Kal StaXeytoSai u>c irtpl ot\Ofiirov. 2. Translate— O'VTU) Si) TraptaKtvairfiivot orav TrapapdXXwmy aXXi)Xotc o't Tt apxoi'Tt'j Kal oi dp^ofitvot 1) iv bSwv CCXV1 EXAMINATION PAPERS,

troptlaiQ y tv dXXatg Ttrrl Koivwyiatc, y Kara Btwplas y KUTO. rtTpartlai', y iji^tTrXoi ytyvofitvot y avaTpa- Ttwrat, y Kal iv avro'tc rtut' Ktiouvotc dXXyXovc Sti)- fxtvot fjti]Ca[xy TUVTIJ Ka-a(l>poi'wvTat ol tvivyrtt; mo TWV trXovaiwy, dXXa iro\\aKi£ iV^rof un)p wivyc, y\iii)[iivo<:, —apara^tlt; iv fax?' ^Xoutr/w iaKtarpo- (jiyKOTt, TTOXXUC 'i-^oi'Ti adpKaq dXXoTpiag, 'ISy ao-Siia- TOQ re Kal cnroplac ijfurur, ap o'iet avTov oh% yyt'ttrBat KaKia. rij arptTtpij. TtXovTtiv Tovt; rotovTOvt;, Kal aXXoc &XXo) TtapayyiXXtty, OTav icda Zvyyiyvwvrat, ort iivSptt; iif-ttTipot t'urlv ovSiv ; Eu olSa fiiv ovv, i(f>ri, iywyt, on OVTW •Kotovatv. QVKOVV tictrtp awfia yo- trwStc [itKpas poniis i^wBtv Sural TtpocXafiia&ai node rb Kafxvtty, ivtort St Kal tlvtv TWV fijoj nramd^tt ahrb airy, o'vrw Si) Kal y Kara ravra tKiivw otaKtifxivy irbXir dnb aftiKpac ivpotpatrtwc, t^wBtv iirayofiivwy y TWV tripwv it, SXtyap^ovfjivyQ rroXtwc £vfifia-%tav y TWV tripwv IK SyfioKpaTOvfiivyc, voait Tt Kal avrij aiiry fxd^tTat, ivivrt ot Kal dvtv TWV 'i\w trraaid^tt ;

3. Sketch the doctrine of \Siai so far as developed by Plato in the Republic. 4. Translate— Doini majus certamen consulibus cum plebe ac tribunis erat: fidei jam sure non solum virtutis ducebant esse, ut accepissent duo patricii consu- latum, ita ambobus patriciis mandare : quin out toto cedendum esse si plebeius jam magistrates consulatis fiat, aut totum possidendum, quam possessionem integram a patribus accepissent. Plebes contra freinit: quid se vivere quid in parte civiuin censeri, si, quod duorum hoininum virtute L. Sextii ac C. Licinii partum sit, id ohtincre uni- versi non possint, ? vcl reges vol decernviros vel— si quod tristius sit imperii nomen—patiendum esse DEGREE OF M.A., F.T. 1871. CCXV11

potiu's quam arnbos patricios consules videant nee in vicemparentur atque imperetur, sed pars altera in aeterno imperio locata plebem nusquam alio natam quam ad serviendum putet. Non desunt tribuni auctores turbarum sed inter concitatos per se omnes vix duces eminent. Aliquotiens frustra in campum descensum quum esset miiltique per seditiones acti comitiales dies, postremo victas perseverantia consulum plebis eo dolor erupit ut tribunos, actum esse de libertate vooiferantes re- linquendumqiie non campum jamsolnm sed etiam urbem captain atque oppressam regno patriciorum, moesta plebs sequeretur.

Translate— Numquamne causa defiet cur victi pacto non stctis ? obsides Porsinre dedistis : furto eos sub- duxistis. Auro civitatem a Gallis redemistis: inter accipiendum aururn coasi sunt. Pacem no- biscum pepig-istis ut legiones vobis captas resti- tueremus: earn pacem iriitam facitis. Et semper aliquam fraudi speciern juris imponitis : non pro- bat populus Romanus ignominiosa pace legiones servatas ? pacem sibi habeat, legiones captas victori restituat: hoc fide hoc fboderibus hoc fe- cialibus caerimoniis dignum erat. Ut quidem tu quod petisti per pactionein habeas tot cives inco- lumes, ego pacem quam hos tibi remittendo pactus sum non habeam, hoc tu, A. Cornelt, hoc vos, feciales, juris gentibus dicitis? ego vero istos, quos dedi simulatis, nee accipio nee dedi arbitror, nee moror quo minus in civitatem obligatam sponsione commissa iratis omnibus diis, quorum eluditur , redeant. Gerite helium, quando Sp. Postumius modo legatum fecialern genu per- culit: ita dii credent Samnitem civem Postumium h CCXVlll EXAMINATION PAPERS,

non civem Romanum esse et a Samnite legatum Romanum violatum, eo vobis justum in nos factum esse helium. Haec hulibria religionum non pudere in lucem proferre, et vix pueris dignas ambages senes ac consulares fallendfe fidei exquirere!

6. Give Livy's account of the formation of the Roman Legion at the time of the Latin War.

7. Translate explain and refer to the context— (a) rrpoaifiXt^a avrov irportpog wart avrif owe T iyty6).iyv arroKpivaoSai. {b) diu> TrttrvvrrBat, ral St o-Ktal aiatrovai (c) dXX' OVK AXKIVOV yt utroXoyov ipw, dXX aXo/iov

{el) Adjurat in quoa adactus est verba. {e) Caput jecinoris a familiaria parti ca;sum dicitur Decio ostendisse. (_/') In pensiones aequas triennii, itautquartaprajsens esset, solutio aeris alieni dispensata est. 8. Give the meaning and the derivation of ovtraptaTo- TOKtia, rrapatjipdynara, ^aXarTrpalov, (f>opTiK('ie, con­ ditio or condicio, novensiles, lectisterniura, caeri- monia.

SCHOOL OF LANGUAGE AND LOGIC.

THUCYDIDES, I.—IV. TACITUS, Annals. 1. Translate— ' Kal rtivSt vfitlt; alrtoi TO T£ npwTOV kaaavrtQ avroiiQ ryv iroXtv jtira Ta M/jo'iicd Kparvvai Kai vaupov ra fiaicpa arrjaat DEGREE OF M.A., F.T. 1871. CCXix

Tlixfi: tQ T6SE TE del dTTO(TTepOVPTEQ OV pOVOV TOVQ VIZ EKEtVCOV 6ESOVXOJ/.IEPOVQ tXEvOepiag, dXXd Kal TOVQ vperkpovg yfoj Kvfifidxoug. ov ydp 6 SovXwadfiEPOQ, dXX' b Svvdptvog fikv TravGUL, TTtpiopwp Se, dXT^QkoTtpov avrb . p£, e'nrep Kal ri)v dtftoaiv TI)Q dpETtiQ 10g EXEV9EP&V Tt)v EXXdca (ptperat. fioXtg SE VVV TE *VP>)X9OLIEV Kal ovSk vvv tirl (pai'Epolg. Xptiv ydp oi.'K ti d6iKOVf.iE0a eri GKoire'iv, dXXd KaO1 o rt d/ivpovpeOa. ot. ydp SptopTeg fie[iovXEV{XEvoLTrpb g ov SteyviDKurag, yStj Kal on /iiAXovrft;, ETrkpxoprat. Kal ETTto'Td- /.uOa o'ia oSy oi *A9r)va'iot Kal OTI KUT' oXiyop x^povmv eirl TOVQ TrkXag Kai XavOdveiv pev oibftevOL Sid TO dvaicrOyTOv vpwp tjtJGov OappuvfTf yvoVTEQ Se. eiSoTag Trtptopdv {GX*>P&Q iyKei(7uv~at. yavxd'CEre ydp \IOVOL 'liXXi/Piop. w AaKEdat- fiovioi, oi) ry SvpdpEL rivd dXXd Ty fitXXijfftt d-pwdptvoi, Kal \iovoi OVK dpxoitsvffv rt)v av^rjo'LV TWV EX^piov, SnrXa- movpEvtjV Se KaraXvovreg,

2, Translate— Kal yv 7/ p-dxn KanTEpd Kal iv Xty^l Ttdfra-. Kal T6 pev Se%ibv KEpag Ttop AOr/vaitov jcrti KapvaTiiOVt ovrot ydp TrapaTeraypEvoL fjaav eVr^arot, tck^avTO Tt TOVQ KoptvOiovg Kai tioaavTO fibXig' ot SE vTroxiopyaaVTEQ Trpbg at/iacrtdv, yv ydp ro x^piov TrpovavrEQ ndv, fidXXovreg TOIQ XiOoig KaOuTTEpOtV OVTEQ Kal TTaifjOVltTaVTEQ IrryEffav av9ig, $E%a- \xkvtov SE Ttov 'AOijvaiLOp ev \Epalv yv irdXtv t) f*dxV- Xoxog Se TIQ TLOV K.opiv9iujv i.in(iori9iiGaQ TO} Evtovvpto Ktpo: tavTtov erpfipE TLUP 'AOtjvauov TO St£ibv KEpag Kal tTTESiiokev EQ ryv OaXaaaav TTUXIV St dirb TWV ve&v dviaTpeipav oi re 'A9rj~ vaiot Kal ol KapvanoL. rb SkdXXo arpaTOTrtSov dfjL(/iorEpo9EV ifidxfTO £vvtxwQ. pdXtara SE TO SEKLOV KEpag rtov Kopiv9iwv, c0* y 6 AvKotypwv tov Kara TO evu/wpov TIOV 'Atiyvaiiov yftvvETO' yXiri^ov ydp aurovg ETTI Tt)v XoXvyEtav Ktofirjv TTEipdffEtv. X9®vov hhv °vp TTOXVV avreixov 0VK ivciSovreg dXXyXoiQ' EiniTa, {jaav ydp rotg 'AOqvaioLQ oi 'nnryQ bjipeXipot Zvptiaxoptpoiy TCJV ireptov OVK EXOPTLOV 'iTnrovg^ ETpdirovTO ot Kopiv9ioi Kal VTrExvpijaav Trpbg TOV X6$ov, Kal e9tPTO rd onXa Kal OVK'ETI KaTEpaivov, dXX' t)avxa%ov, ev SE Ty rpotry ravry Kara rb Se^ibv KEpag oi irXtiaroi re avTaip dTTi9avov Kal AvKoypLov b CTparriyvg.

3. Translate— Stabat inter alias navis ornatior, tanquam id quoque houori inatris daretur : quippe sueverat triremi et A2 EXAMINATION PAPERS,

cl.issiarioruni remigio vchi. Ac turn invitata ad opulas erat, ut occultando facinori nox adbiberetur. Satis constitit extitisse proditorem, et Agrippinam auditis insidiis, an crederet iiinbiguam, gestamine sella? Baias pervcctam. Ibi blandiinentum sublevavit metum : co- miter exceptasuperque ipsum collocata. Nam pluribus sernioiiibus, modo fiiniiliaritiite juvcnili Nero ct rursus adductus, quasi serin consociaret, tractu in longum convictu, prosequitur abeuntein ; arctius oculis et pec- tori hasrens, sive explenda simulatione, seu perituroe raatris supremus aspectus quamvis ferum animum retiuebat. Nocteni sideribus illiistrem et plncido mari quietam, quasi convincendum ad scelus, clii pnebucre. Ncc multum erat progressa navis, duobus e numero familiarium Agrippinam comitantibus : ex quis Crepe- reius Gallus baud procul gubermiculis astabat, Accr- ronia, super pedes cubitantis rcclinis, painitcntiam filii et recuperatam mairis gratiam per gaudium memora- bat: cum dato signo ruerc tectum loci ranlto plumbo grave ; pressusque Crepereius et statim exanimatus est. Agrippina et Acerronia cminentibus lecti parieti- bus, ac forte validioiibus quam ut oneri cedercnt, pro- tecta: sunt.

• 4. Translate- Ingentia illi bella, cxpugnationes urbium, fusos cap- tosque reges, aut si quando ad interna prasverterent, discordias consilium adversum tribuitos, agrarias fru- raentariasque leges, plebis et optimatium certamina, libero egressu memorabant. Nobis in arcto et inglorius labor : immota'qiiippe aut modice laccssita pax, micstic urbis res, ef princeps proferendi imperii incuriosus erat. Non tamen sine usu fuerit introspicere ilia, primo aspectu levia, ex quis magnarum sajpe rerum motus oriuntur. Nam ennctas nationcs et urbes populus aut primores aut singuli regunt: dolecta ex iis et consociata rei publico: forma laudari facilius quam evenire, vel, si evenit, baud diuturna esse potest. Tgitur, ut olini plebe valida, TCI cum patres pollerent, noscenda vulgi natura et quibus modis temperanter liaberetur, senatusque ct optimatium ingenia qui maximc perdidicerant, callidi temporum et sapiemes credebantur ; sic couverso statu, neque alia re liomaua quam si unus impcritet, base conquiri tradique in rem fuerit, quia pauci prudentia DEGREE OF M.A., F.T. 1871. CCXxi honesta ab deterioribus, utilia ab noxiis discernunt, plures aliorum eventis docentur.

5. Translate explain and refer to the context— {a) rrpoapaXuvrtc oi 'AOyvalot, diroaifiwtrdvTwv tKtlvuiv Kal •KpotfifiaXovrwy, tripav^vavv ditoXXvovat. {b) ijZet AwptaKoe TroXf uoe KOI Xotfibe lift avrtji. {c) rvtpXbe dvyp, otKtt Si Xki ivt TranraXoio-irr], {d) Non adesse ut olim earn copiam omissa con- farreandi adsuetudine. {e) Praebitumque oleum equiti ac senatui Grceca facilitate. (/') Dubinin an quassita morte, auditos in funere ejus Marcite gemitus semet incusantis quod causa exitii marito fuisset.

6. Give the meaning- and the etymology of tipyvy, •Kapiittptatn, irportfiiviafxa, y(tifxwv} ambitiosUS, caiig-ula, incolumis, sobrinus.

7. Compare the style of Thucydides with that of Tacitus.

8. Construct a Table shewing tbe connexion between Augustus and the three following Emperors. EXAMINATION PAPERS,

SCHOOL OF LANGUAGE AND LOGIC.

MILL, Logic. MANSELL, Prolegomena. CAMPBELL, Rhetoric. DONALDSON, Varronianus and New . Cratylus.

1. Explain fully tbe meaning of tbe following:— " This fivefold division is an exhaustive classifica­ tion of all things that can be believed or tendered for belief."—MILL, Book I. c. 5.

2. Explain and criticize the view involved in the following:—" This character of necessity ascribed to the truths of mathematics and the peculiar certainty involved in them is a delusion."

3. Explain clearly the Method of Agreement, and the bearing upon it of the Plurality of Causes.

4. Exemplify Mill's "Fallacies of Simple Inspection."

5. Explain the following statement:—'' Language is in its earliest operations a sign not of concepts but of intuitions."—MANSELL, ch. II.

6. Explain clearly the following:—"All analytical judgments are necessary but they cannot be said to be in necessary matter."—MANSELL, ch. V.

7. Give a brief summary of Campbell's views summed up thus, "So much then for tbe vivacity pro­ duced by conciseness, and for those blemishes in style which stand in opposition to it, tautology, pleonasm, and verbosity." DEGREE OF M.A., F.T. 1871. CCXxiii

8. State with Campbell's criticisms the four hypo­ theses which have been propounded to account for our finding pleasure in representations that excite pity or other painful feelings.

9. What is the etymology assigned by Donaldson to aurelius, cassis, gradivus, ueptunus, petorritum, proedium. •

10. With reference to the Etruscans, Donaldson says, " Every available source of grammar and philo­ logy tends to confirm and reconcile the otherwise divergent and contradictory statements of ancient history." Explain this fully.

11. State Grimm's law of the interchange of mutes in certain Indo-European languages, and give some examples of the law as tabulated by Donaldson.

12. Explain and illustrate the following statement:— " Greek grammarians distinguish two sorts of Combinations Kara avvQiatv, and *rara TrapaOicriv." CCXS1V EXAMINATION PAPERS,

EXAMINATION FOR THE DEGREE OF DOCTOR OP MEDICINE.

LOGIC.

1. " Invariable sequence is not synonymous with causation unless the sequence besides being in­ variable is unconditional." Explain this fully.

2. Show tbe value of the method of Residues and relate an example of its employment.

3. Contrast generally observation and experiment.

4.' How are legitimate hypotheses to be distinguished from illegitimate ?

5. Explain Mill's view of the Evidence of the Law of Universal Causation.

6. Explain how it is that the physical truths which are the subject of geometry can be deduced from so few premises.

7. Characterise and exemplify malobservation.

8. What are Fallacies of Generalization ? Shew how they grow out of bad classification.

9. Explain and illustrate the Fallacy of Petitio Prin- cipii. 10. State the principal requisites of a Philosophical Language. DEGREE OF M.D., F.T. 1871. CCXXV

11. Discuss the question, are natural groups given by type or by definition ?

12. Explain the true mode of Natural Classification.

PUINCIPLES AND METHODS OF OBSERVA­ TION AND REASONING IN MEDICAL INQUIEY.

James Robertson, M.A., M.D.

1. Explain the different methods that may be followed in Clinical Inquiry.

2. What is the nature and use of Theory in Medicine ? Show that it is not only useful but also necessary • for investigation and advancement in Medicine.

3. State the common sources of error in estimating the efficacy of any remedy or mode of treatment.

4. How is the effect of a remedy or mode of treat­ ment to be ascertained ?

5. Explain the nature of the numerical and analogical methods of research.

A3 CCXXV1 EXAMINATION PAPERS,

PHYSIOLOGY OF BEAIN AND NERVOUS SYSTEM.

Professor Halford.

1. "To deal with mind apart from the consideration of the matter through the changes of which it is manifested is truly no less vain and absurd than it would confessedly be to attempt to handle electricity and gravitation as forces apart from the changes in matter by which alone we know them. Instead of mind being a wondrous entity, the independent source of power and self-sufficient cause of causes, an honest observa­ tion proves incontcstably that it is tbe most dependent of all the natural forces."

2. " Tt is strange to see how some, who confidently base their argument for the existence of a God on the ground that everything in nature must have a cause, are content, in their zeal for free will, to speak of the will as if it were self-determined and bad no cause. As thus vulgarly used, the term Will has no definite meaning, and certainly is not applicable to any concrete reality in nature, where, in the matter ot will, as in every other matter, we perceive effect witnessing- to cause, and varying according as the cause varies."

By what anatomical, physiological, and pathological evidence can these statements be upheld ? DEGREE OF M.D., F.T. 1871. CCXXV11

MENTAL PATHOLOGY, MENTAL THERAPEU­ TICS, AND MENTAL HYGIENE.

1. Describe the principal morbid appearances in the Brain and Membranes in acute insanity; in chronic insanity; in general paralysis.

2. State the more important differences between Ec­ centricity and Insanity.

3. Give the leading signs of General Paralysis in its • earliest stages.

4. Describe an ordinary case of Epileptic Mania, its origin, progress, treatment, and probable result.

5. What are the chief indications for using opium or stimulants or both in the treatment of acute insanity ?

6. What is the prognosis in Puerperal, Metastatic, Hysterical, and Senile Insanity ?

THEORY AND PRACTICE OF MEDICINE.

James Robertson, M.A., M.D.

1. Describe the symptoms, physical signs, and patho­ logy of Emphysema of the lungs, and the treatment, regiminal, dietetic and medicinal. CCXXV111 EXAMINATION PAPERS,

2. Describe the symptoms and pathology of Angina Pectoris, and the treatment to be adopted.

3. Diagnose between Meningitis of the spinal cord and Myelitis, and direct the appropriate treatment.

4. Describe the chief varieties of intestinal obstruction according to the causes inducing it; the symptoms and signs distinguishing obstruction of the small and large intestines. Indicate the treatment to be pursued. 5. Enumerate the diseases of the liver attended with enlargement of that organ, and state the dis­ tinguishing characters of the several forms of disease. 6. What are the causes that give rise to fluid in the peritoneum, and the signs by which it is dis­ tinguished ? Diagnose between Ascites and other conditions simulating it. Indicate the treatment appropriate according to its several causes.

7. Describe tbe pathology of dysentery and the treat­ ment adapted to its acute and chronic forms.

CASES FOE COMMENTARY.

James Robertson, M.A., M.D. 1. T. B., ajtalis 32, laborer, was admitted into the Melbourne Hospital suffering from the follow­ ing symptoms: — Troublesome hiccough with DEGREE OF M.D., F.T. 1871. CCXX1X

frequent vomiting, the egesta being liquid, greenishj> and mucous; abdomen tense, tympa­ nitic and tender on pressure; expression of countenance, anxious; skin, hot and dry; pulse, 108, small, weak. History states that he has .uniformly enjoyed good health until about fifteen days ago, when he felt chilly and afterwards had a distinct rigor. This was followed by severe headache and general febrile disturbance, which continued with slight intermissions. Has had professional attendance from commencement of illness. Last night was seized with severe pain and vomiting, which have continued at intervals till admission. His bowels acted yesterday. Was ordered general and local treatment. The urgent symptoms were to some extent relieved; but he sank gradually and died on the following day. From what disease did he suffer and what treat­ ment would you have adopted? Describe the post mortem appearances you would expect to find.

2. E. S., retatis 2f), domestic servant, enjoyed good health until ten months ago, when she suffered from what she terms a bilious attack (vomiting and pain over the region of the liver). Became a patient in a country hospital, where she remained three months, subject to more or less pain at the epigastrium, described as being of a burning character after meals. Recovered so far as to be able to leave, and shortly afterwards observed a tumour in the epigastric region, which has gradu­ ally increased until now. The tumour is situ­ ated behind and projects from below the ensiform cartilage. It is tense and globular, and is appa­ rently of the size of a large orange. A slight S EXAMINATION PAPERS,

degree of pain is felt on manipulation, and per­ cussion elicits a dull sound. Occasionally pain of a severe character occurs in paroxysms. In the recumbent posture pulsation is distinct when pressure is made in the antero posterior direction, but no lateral expansion can be detected. A dis­ tinct bruit de sotiffet is audible on auscultation. When the patient assumes the erect posture, the volume of the tumour is apparently increased, but both pulsation and bruit disappear. Com­ ment on the nature of the disease, diagnosis, prognosis, and treatment.

THIRD HONOUR EXAMINATION.

SCHOOL OF LANGUAGE AND LOGIC.

Translate, with brief explanatory notes if you judge them necessary— (a) wg fit cimKtro irrl rov "AXvv Trora/iov b Kpoiaog, rb ivOevrtv, wg pt:v iyw Xkyw; Kara rag ioucrag yttyvpag dttfSi- j3aat rov arparbv wg bt b TroXXbg \byog 'EXXyvwv, QaXijg ot b WiXijwog bttfjifiartt diropkovrog ydp Y.pniaov OKwg oi dtuf3iiatrat rov Trorapbv b rrrparbg (ov ydp Si) tlvai KW rovrov rov xpbvov rag yttpbpag ravrug), Xeytrat rraptovra rov Ba\yv tv TO} arparo~kb(ii. iroiyaat avr rov rrorapbv i£ dpto-repijg XW'S viovra roii arparov Kal ex fcgii/; |U££iv- •KotTfO'aL bt wot' dvutOtv rov crparorrkbov df/^aptvov, Stw- p»Xa fia6h]v bpvnattv dyovra nqvotibka, 'i>Kwg dv rb arpa- roirtSov idpvptvovKara ywrov Xdfiotravry Kara ri)v Stwpvxa tKrpairuptvog tic rwv dpxatwv ptkOpwv, Kal avng rrapajiti- ~ Piiftevog rb arpanntbov ig rd dpxfiia lafidWof Hart, lirti THIRD HONOUR EXAMINATION, F.T. 1871, CCXXX1

TE Kal iaxiaQy rdxivra 6 TTOTapbg, dptporkpy Siaparog eye- VETO, OL Se Kal TO TrapaTrap Xkyovat Kal TO dpxaiov pke9pov dTTO%ijpaj'9ripaL' dXXd TOVTO fiev oi) Trpooiep.ac KOJQ ydp oTriGio TropevoLtEVOL Steflyaap avrov ;

(6) (SaaiXev, ETreiSt) dXr)9rfty Staxpyaaa9ai Trdr'Tug fie KE- Xevetg, raura Xeyopra rd fit) \pevS6fitv6g TIQ varepoy VTTO trtv dXtoaErat, Ty 'EXXdSt TTEVII] ftep aiti Kore avvrpotpog avvean, apery Se tTracror tort, d-rro TE aoqtirjg KaTepyatjfikvrj Kal vdfiuv hxvpov' ry otaxptwfikvy y 'EXXdg, ryv TE irtp'tr\v dTrapvvETai Kal n)v SeoTro-yvvyv. aivku) pkv i'vv iravrag "EXXtji'ag TOVQ Trept KE'IPOVQ TOVQ AutptKovg xioP0VQ oiKypk- VOVQ' Epxofiat Se Xe^tov ov irepl Trdprtop TOVTSE roi-g Xoyovg, dXXd irEpt AaKeSatfiopitop fioupajv Trpwra //ev, on OVK tart OKwg ICOTE

(c) To SE SI) {.terd TOVTO tTn9vpto vplv XPV^It(pSijaat, to Kara- \py

( a9tpog' diTEv9vi'EL SE ppor&p TOVQ r dyvwftoavpav Ttf.tujprag Kai pt) rd 9EQV av^ovrag ovv fiaivofispqi, S6%oi. CCXXXii EXAMINATION PAPERS,

KpvTTTtvovai Sk TrotKtXwg Sapbv XP^V0V TroSa Kal 9yptt/atv rbv daeirrov. ov ydp Kpeiaaov TTOTE TLOP vopotv yiyptoaKEiv XPV Kai ptXtrdv. KOv

(e) Kal tp9kypa Kal avepoev tp96pt}pa Kal darvvopovg bpydg eStSd%aro Kal SvaauXtov Trdytov vTratOpeia Kal Svaopfipa (pevyetv (SkXy TravrOTTopoQ aTTopog ITT' ovSev epxtTai TO pkXXop' "AtSa pbvov tpev^tv OVK ETrd^erat' voatop S' dpt)xdvtiip (fivydg ZvpTretypaarat. aov og rdS' epdet.

(/") EP. ^Xtflui' totKag roig irapovai Trpdypaai. TIP. xKtSijj ; xXtSitiPTaQ u>Se rovg tpovg cyoi ixBpovg ISotpt' Kal ak S' kv rovrotg Xkyto, EP, y Kapk ydp rt Zvpfyopcug ETrainq,; HP, a7rXt£ Xoyoj TOVQ Txdprag kx9aipu Oeobg, oaoi Tra96vreg EV KaKovai p' tKSiKtog. EP. KXVW a* kyuj pspyvtW* ov apiKpdp voaov. IIP. voaolp* dv, EI voaypa TOVQ kx9povg arvyeXv, EP. elrjg ipopyrbg OVK dp, Et TTpdaaoig KOXIOQ. TTP. topoi. EP. TOSE ZEvg TOVTTOQ OVK tTrtffrarat. TTP. aXX' tKOtSdaKEi 7rdv9' b yrjpdaKwv xp^vog, EP, Kal pt)p av y OVTTLO aoxppoi'tip t-rriaTaaai. TIP. ak ydp TrpoayvStop OVK dv bv9' virypEryv, EP. kpelp totKag OVSEP aiy XPV^ft trariip. UP. Kal pt)p oipeiXiuv y dv rivoLp auroj xaPtv- THIRD HONOUR EXAMINATION, F.T. 1871. CCXXxiii

(ah)KEg dyavol "Qirdaav o'lKaS' iovrt epy (3ovXy TE VOOJ ret EITTLO & oaaa rot alaa Sbpotg tvt iroiyrdtatv K*/(V dpaaxio-9ai' av Sk TErXdptvai Kai dpdyKy, MJJO£ T

SCHOOL OF LANGUAGE AND LOGIC.

Translate with such brief notes as you may judge necessary— {a) Ominis etiam tibi causa absit C. Flaminii me- moria : tanieu ille consul demuin et in provincia et ad exercitum ccepit furere: hie priusquam peteretconsulatum,deindein petendoconsulatum, nunc quoque consul, priusquam castra videat aut hostern, insanit: et qui tantas jam nunc procellas proslia atque acies jactando inter togatos ciet, quid inter anna tain juventutem censes facturum et ubi extemplo res verba sequitur ? atqui si hie, CCXXX1V EXAMINATION PAPERS,

quod facturum se denuntiat, ex'emplo pugnaverit, aut. ego rem militarem belli hoc genus hostern hunc ignoro, aut nobiliur alius Trasumeno locus nostris cladibus erit. Nee ploriandi tempus adversus iiruim est, ut ego contemnendo potius quam adpetendo gloriam inodum excesserim : sed ita res se bahet, una ratio belli gerendi adversus Hannibalem est qua ego gessi, nee eventus modo hoc docet—stultoruni iste magister est.—sed eadem ratio, quie fuit futuraque, donee res eaidem manebunt, immutabilis est.

{b) Quoero, si luec emptoribns venditor non dix- erit, a'desque verdiderit |)luris miilto, quam se venditiiruin yiutarit, num id injuste, aut improbe fecerit ? 'Il!e vero,' inquit Autipater. 'Quid est enim ahud, erranti viain non monstrare, quod Athenis exsecratioiiilms puhlicis sanctum est, si hoc non est, eniptorcm pati mere et per errorcm in maximain fraiidein incurrere ? Plus etiam est, quam viam non monstrare : nam est scientem in errorem alterum inducere.' Diogenes contra : ' Num te em ere coegit, qui ne hortatus quidem est ? Hie, quod non placcbat, proscripsit; tu, quod placebat, cmisti. Quod si qui proscri- bnut, villain bonam beneque ajdificatam, non existimantur fef'ellisse, etiam si ilia nee bona est nee tedificata ratione ; multo minus, qui domurn non laudarunt. Ubi enim judicium eraptoris est, ibi fraus venditors qua? potest esse ? Sin autem dictum non omne praistandum est, quod dictum non est, id pra?standum putas ? Quod vero stul- tius, quam venditorem ejus lei, quam vendat, vitia nan-are? Quid autem tarn absurdum, quam si domini jussu, ita prasco pra;dicet: domum pes- tilentem vendo ?' THIRD HONOUR EXAMINATION, F.T. 1871. CCXXXV

(c) PY. Ubi tu es ? AR. Eccum: edepol, vel elephanto in India quo pacto pugno prasfregisti brachium. PY. Quid ? brachium ? An. Illud dicere volui, femur, py. At indiligenter iceram. AH. Pol si quidem connixus esses, per corimn, per viscera, • perque os elephant! brachium tramitteres. PY. Nolo istajc hie nunc. AR. Ne hercle operae pretium quidem, mihi te enarrare : tuas qui virtutes sciam. —Venter creat ornnis has aiiumnas: auribus peraurienda sunt, ne dentes dentiant, et adsentandum est, quidquid hie mentibitur. PY. Quid illud, quod dico ? AR. Hem, scio iam, quid vis dicere. Factum hercle; memini fieri, PY. Quid id est? AR. Quidquid est, PY. Habos tabellas ? AR Visropare? Ilabeo, et stiliim. PY. Facete advortis animum tuum ad animum rneum. AR. Novisse mores me tuos meditate'decet, curamque adhibere, ut- prasvelim, quod tu velis. PY. Ecquid meministi AR. Memini. Cen­ tum in Cilicia et quinquaginta, centum Sycolatronida?, triginta Sardi, sexaginta Macedones, sunt homines, tu quos occidisti uno die. PY. Quanta istaec hominum summa est ? AR. Septem millia. PY. Tantum esse oportet; recte rationem tenes, AR. At nullos habeo scriptos; sic memini ta- men. PY. Edepol memoria est optuma. AR. Offa me monet. CCXXXV1 EXAMINATION PAPERS,

{d) Prima? frugiparos fetus mortalibus segris dididerunt quondam praeclaro nomine Athena? et recreaverunt vitam legesque rogarunt, et prima? dederunt solacia dulcia vita?, cum genuere virum tali cum corde repertum, omnia veridico qui quondam ex ore profudit; cujiis ct extincti propter divina reperta divolgata vetos jam ad ccelum gloria fertur. nam cum vidit hie ad victum qua? flagitat usus omnia jam ferine mortalibus esse parata et, proquam posset, vitam consistere tutam, divitiis homines et honore et laude potentis afflucre atque bona gnatorum excellere fama, nee minus esse donii cuiquam tamen anxiacorda, atque animi ingratis vitam vexare sine ulla pausa atque infestis cogi sa?vire querellis, intellegit ibi vitiinn vas efficere ipsum omniaque illius vitio corrumpier intus qua? conlata foils et commoda cumque venirent; partim quod fluxuin pertusumque esse videbat, ut nulla posset, ratione explerier umquam ; partim quod tajtro quasi conspurcare sapore omnia cernebat, qmecumque receperat, intus.

{e) At pater Anchises penitus convalle virenti Inclusas aniinas superumque ad lumen ituras Lustrabat studio recolens, onmemque suorum Forte recensebat numerum carosque nepotes, Fataqiie fortunasque virum moresque manusque. Isque ubi tendentem adversum per gramina vidit iEnean, alacris palmas utrasque tetendit, Effusmque genis lacrimal, et vox excidit ore : Venisti tandem, tunque exspectata parent! Vicit iter durum ? datur ora tueri, Nate, tua, et notas audire et reddere voces ? Sic equidem ducebam animo rebarque futurum, THIRD HONOUR EXAMINATION, F.T. 1871. CCXXXVii

Tempora dinuinerans, nee me mea cura fefellit. Quas ego te terras et quanta per aequora vectum Accipio ! quantis jactatum, nate, periclis ! Quam metui, ne quid Libya? tibi regna nocerent!

if) Vixere fortes ante Agamemnona Multi; sed omnes illacrimabiles Urgentur ignotique longa Nocte, carent quia vate sacro. Paullum sepulta? distat inertia? Celata . Non ego te meis Chartis inornatum silebo, Totve tuos patiar labores Impune, Lolli, carpere lividas Obliviones. Est animus tibi Reruinquc prudens et sccundis Temporibus dubiisque rectus, Vindex avara? fraudis, et abstinens Ducentis ad se cuncta pecunia?, Consulque non unius anni Sed quoties bonus atque fidus Judex honestum pr.Btulit utili, Rejecit alto dona nocentium Vultu, per obstantes catervas Explicuit sua victor anna.

SCHOOL OF LANGUAGE AND LOGIC.

1. Translate explain and refer to the context— {a) Karatppovyaag TI)V TvpavviSa, yyttpe rpiTyv ardatv. (b) d\\' ou Karairpol^tat dirorpiirwv TO \ptbv yeviadat. (c) ovSiv Si Sttvov /n>) iv i/jtol trry. CCXXXV111 EXAMINATION PAPERS,

{d) aXijdtg; dXX' ov T6vS'"OXvinrov 'ia6' on •%atpwv tri \jjoyoto-t Stvvdattg ifii, {e) yiyvwo-Kt aavrov Kal fitOapfxoaat Tp6novg viovg. {f) dXX' tfiiryg KaXd/jiyv yi a oto/iat titropowvra ytyvwaKtiv. {g) Natis sepulcro ipse est parens. {h) Misera? ad parietem sunt fixse clavis ferreis. (i) Nonduin illi flavum Proseipina vertice crinem Abstulerat. (J) Cave sis ne bubuli in te cottabi crebri crepent. {k) Latinis actis feriis, sacrificio in uionte perfecto. {I) Signifer, statue signum, hie manebimus optime. fw) Qui ter amplum Geryonen Tityonque tristi Compescit unda. (M) Non Sicula Palinurus unda.

2. Discuss the various readings or interpretations of {a) irarpog irapaSo^dg tig 6' bfiyXtKag f^povw KtKriifitO', ovStlg avra KarafidXXtt Xoyog oiiO cl St ixKpuiv TO aotpov tvpyrat tpptvuiv. {b) KohStlg ivupyyg dXX' iiptvyt TO f-tt) tlSivat. (c) fiy fiov rrpoKiiSov fidaaov wg ifxol yXvKii, {d) Namque aliud terris, aliud legionibus istis Eventum dici poterit quodcunque erit actum. {e) Ac celeri ferme percunt fulmina lapsu. {f) Prudens Oceano dissociabili Terras abscidit. {g) Militia simul Fessas cohortes addidit oppidis. THIRD HONOUR EXAMINATION, F.T. 1871. CCXXxix

3. Discuss fully the meaning and the etymology of dva^aiTt^tiv, doXXt'jg, tKartpdt, i)cpotiSi]g, rrtSapatog, irpovo-tXovfitvog; ojquus, columna, interpres, ob- , scums, scaptensula, testa.

4. Explain and exemplify the following metres— Ionic a minore, Glyconic, Paroemiac, Longer Sapphic, Hendecasyllable, Adonic.

5. What are the Aryan cognates of the following words—viTVog, tifia, trrog, yXwpdg; signum, cruor, fruor, flamma ? Illustrate all the various forms of the roots which you quote.

SCHOOL OF LANGUAGE AND LOGIC.

1. "In the word SiSwpt are exemplified two radically different principles of change." Explain this fully.

2. " La liaison du sens et du mot n'est jamais neces- saire, jamais arbitraire, toujours elle est motived." Explain this fully.

3. Give the derivation of the following words, and trace the Aryan roots involved in them —dXaivw, /jtnvrra,Trtpdw; aurora,civis,holera; book,hammer, timber; age, berger, larme.

4. " Neque e plane neque i auditur." Who said this, and of what sound ? Explain the statement from the history of the Latin language. CCxl EXAMINATION PAPERS,

5. What are the substitutes for v in Greek ? Re­ garding one of these Peile differs from Miiller. State and examine the words in which it is sup­ posed to occur.

6. " We have thus traced the modern Teutonic dia­ lects back to four principal channels." Name and give some account of each of these.

7. How did these words " genitive case" come into Grammar ?

8. " Among monosyllabic roots it is necessary to dis­ tinguish primitive, secondary, and tertiary roots." Explain and exemplify.

9. Give with dates a concise history of the growth of Greek Tragedy.

10. Give with dates a brief account of these authors and of their works:—Apollonius Rhodius, Aris­ totle, Eupolis, Lucian, Moschus, Theocritus, Xenophon.

11. Discuss the value of Livy's History.

12. State briefly with dates what you know of the fol­ lowing Latin authors and their works :—Catullus, Ennius, Hyginus, Lucan, Plautus, Seneca. THIRD HONOUR EXAMINATION, F.T. 1871. CCxll

SCHOOL OF LANGUAGE AND LOGIC.

1. Translate into Greek Iambics— Hail, Holy Light! offspring of Heaven firstborn, And of the Eternal, co-eternal beam,. May I express thee unblamed ? Since God is light, And never but in unapproached light Dwelt from eternity, dwelt then in thee, Bright effluence of bright essence uncreate ! Or hear'st thou rather, pure ethereal stream Whose fountain who can tell ? Before tbe sun, Before the heavens thou wert, and at the voice Of God, as with a mantle did'st invest The rising world of waters dark and deep, W?on from the void and formless infinite.

2. Translate into Greek Prose— When the king of the Persians was on a journey within the boundaries of his empire, it was the custom for gifts to be offered him by all the Persians. Of those who were engaged in agri­ culture some presented oxen or sheep, others corn or wine; whilst tbe poorer class brought milk, cheeses, dates, and other fruits of trees which grew on their lands. All these offerings were made by them individually to the king, as he marched or rode past them, not under the name of tribute but as a free gift. Now a certain Persian, whose name was Sinae- tas, had fallen in with Artaxerxes, surnamed Mnemon, at a distance from his cottage, and had nothing to offer the king; he was loth, however, that the king should seem to pass unbonoured by him. And so he made the best of his way to a / CCXlii EXAMINATION PAPERS,

river that flowed hard by, and taking up water in the hollow of each hand, offered it to the king enhancing his present as far as he could with some loyal and well-chosen expressions. Artax- erxes was wonderfully pleased both with the gift and tbe good feeling and expression of the giver, and holding it a no less kingly act to receive small presents with a good grace than to confer large ones himself, said that he willingly accepted that present of water, and was quite as much pleased with it as he could have been with the most costly gift. Subsequently he sent the man a considerable sum of moneys with a Persian dress and a golden goblet, in which he might driuk the water drawn from the river.

SCHOOL OF LANGUAGE AND LOGIC.

1. Translate' into Latin Hexameters— When the gay sun first breaks the shades of night. And strikes the distant eastern hills with light, Colour returns, the plains their livery wear, And a bright verdure clothes the smiling year; The blooming flowers with opening beauties 8 loW And' grazin' • g flocks„ their milky fleeces show ; The barren cliffs with chalky fronts arise, And a pure azure arches o'er the skies. But when the gloomy reign of night returns, Stript of her fading pride, all nature mourns ; The trees no more their wonted verdure boast, But weep, in dewy tears, their beauty lost. THIRD HONOUR EXAMINATION, F.T. 1871. CCxliii

No distant landscapes draw our curious eyes, Wrapt in night's robe the whole creation lies ; Yet still, even now, while darkness clothes the land, We view the traces of the Almighty hand.

or into Latin Elegiacs— Suns that set, and moons that wane Rise, and are restored again : Stars that orient day subdues, Night at her return renews. Herbs and flowers, the beauteous birth Of the genial womb of earth, Suffer but a transient death, From the winter's cruel breath: Zephyr speaks ;, serener skies Warm the glebe, and they arise. We, alas! earth's haughty kings, We that promise mighty things, • Losing soon life's happy prime, Droop, and fade in little time. Spring returns, but not our bloom, Still 'tis winter in. the tomb.

. Translate into Latin Prose in the style of Livy— Being at the beginning- of the charge on a horse as courageous as became such a master, he made so furious speed to set upon a company of Scotch lancers, that he was singly engaged and mortally wounded, before it was possible for his regiment, though as brave men as ever drew sword, and too affectionate to their colonel to be slack in follow­ ing him, to come up time enough to break the fury of that body, which shamed not to unite all their force against one man; who yet fell not 12 CCxliv EXAMINATION PAPERS,

among them, but being faint and all covered with blood, of his enemies as well as his own, was carried off by some of his own men, while the rest, enraged for tbe loss of their dear colonel, fought not that day like men of human race; but deaf to the cries of every coward that asked mercy, they killed all, and " would not that a captive should live to see their colonel die; but said the whole kingdom of Scotland was too mean a sacrifice for that brave man." His soul was hovering to take her flight out of his body, but that an eager desire to know the success of that battle kept it within till the end of the day, when the news being brought him, be cleared his dying countenance, and said, " I now rejoice to die, since God hath let me see the overthrow of this perfidious enemy; I could not lose my life in a better cause, and I have the favor from God to see my blood avenged."

SCHOOL OF LANGUAGE AND LOGIC.

1. Refer to the context and discuss the various readings and conjectural emendations in the following— {a) Are of a most select and generous cheff in that. {b) The readinesse is all, since no man ha's ought of what he leaves, what is't to leave betimes.

2. Explain fully and refer to the context— {a) To think what lenten entertainment the players shall receive from you. THIRD HONOUR EXAMINATION, F.T. 1871. CCxlv

(6) Ay, sir; but 'while the grass grows' the pro­ verb is something musty, (c) They swell with pride and must be titled Gods, Great benefactors of mankind, deliverers. {d) Built by Emathian or by Parthian hands The great Seleucia, Nisibis, and there Artaxata, Teredon, Ctesiphon. {e) From Arachosia from Candaor east and Margiana. (f) And thus do we of wisdom and of reach, With windlasses and with assays of bias, By indirections find directions out. {g) 0 goodly golden chaine, wherewith yfere The vertues linked are in lovely wize. {h) Nine hundred Paternosters every day And thrise nine hundred aves she was wont to say. {i) At which the bushy teade a groome did light And sacred lamp in secret chamber hide.

3. Discuss the meaning and the etvmology of alabas­ ter, amice, andvile, bisson, debel, eyas, forray, mister, orison, recreaunt, scape, wassail.

4. Give y-our idea of the character of Polonius.

5. To what sources was Spenser mainly indebted for the Faery Queene ?

6. Criticize the style of Milton. Quote any passage you remember in illustration of your remarks.

7. Discuss briefly the question of Phonetic Spelling as applied to the English Language. CCxlvi EXAMINATION PAPERS, *

8. Contrast briefly the capabilities of the English with those of the Greek Tongue;

9. Give a brief account of the English Metrical Ro­ mance.

10. Give with dates some account of these English authors and their works—Beowulf, Boswell, Cole­ ridge, Donne, Geraldus Cambrensis, Gray, Man- devil, More, Wiclif, Wordsworth.

11. Trace the history of the English Drama to Shake- spere.

12. What do you know- of the date, the authorship and the subject of the Ancrcn Rewle, tho Morte d'Arthur, the Novum Organon, Junius' Letters, the Vision of Piers' Ploughman, Percy's Reliques ?

SCHOOL OF LANGUAGE AND LOGIC.

1. Two distinct views of the meaning of a Disjunctive Proposition have been maintained. Which do you adopt and why ?

2. What are Dr. Wheweli's objections to the ' Four Methods' ? How may they be answered ?

3. Distinguish accurately Classification, Nomencla­ ture, Terminology. THIRD HONOUR EXAMINATION, F.T. 1871. CCxlvii

4. State generally the various views maintained con­ cerning the nature of Inference.

5. What is the method of Concomitant Variations ? By what other name is it sometimes known ? Give examples illustrating its use.

6. 'Contrast Observation and Experiment.

7. State Mill's view of the Evidence on which the Law of Causation rests.

8. Exemplify the various Rhetorical methods whereby Energy is attained.

9. " Now the kinds of Rhetoric are three in number." Explain and illustrate this statement of Aris­ totle's.

10. " A sensibility to Beauty does not necessarily im­ ply the power of Taste." How is this assertion proved by Stewart ?

11. " Wordsworth was more of a poet or bard than of a lyrist or minstrel." Explain the distinction here drawn by Mr. Masson.

12. " And thus prose and .verse are presented as two circles, not entirely separate as some would have them, but intersecting through a large portion of both their areas." Explain the view thus illus­ trated by Mr. Masson. CCxlviii EXAMINATION PAPERS,

SCHOOL OF NATURAL SCIENCE.

ELEMENTS OF CHEMISTRY AND MINERALOGY, AND ELEMENTS OF STRUCTURAL AND SYS­ TEMATIC BOTANY. Professor McCoy.

1. Describe the chemical and physical characters as fully as you can of all the " Monad " elementary bodies.

2. Write down for as many gaseous elements as you can the precise numerical relations between the combining volume, combining weight and specific weight, with the changes produced by changes of temperature.

3. Give as many characteristic angles as you can resulting from the most commonly occurring different ratios of the variable elements in crystals of Trimetric minerals.

4. Cubic minerals are found frequently in the forms II.K.o. How many faces would bound such forms, and II being the greater, what would be the characteristic angles for the two or three most common ratios of H and K ?

5. Describe the structure and physiology^ of all the parts of a Clematis engaged in reproduction.

6. Define as many as you can of the great systematic groups of plants. THIRD HONOUR EXAMINATION, F.T, 1871. CCxlil

ELEMENTS OF COMPARATIVE ANATOMY AND ZOOLOGY. Professor McCoy. 1. Give the special and general homologies of all the bones of the nasal segment in the skull of a perch, a crocodile, and a tiger, with the relations of each to the adjoining bones. 2. Describe, in as full detail as you can, the nervous system in each of the classes of animals. 3. Characterise as many as you can of the orders of the Hamatocrya by their chief anatomical and external characters. 4. Describe the intimate structure, succession, and general characters of the teeth in each of orders of Vertebrata possessing them. 5. Describe the circulation in each of the orders of the Radiata, and define them briefly by their other characters.

GEOLOGY AND PALAEONTOLOGY.

Professor McCoy. 1. Describe all the igneous rocks, with their constitu­ tion and relations of position to other rocks. 2. Describe, in as full detail as you can, the whole of the beds between the base of the lower green- sand and the top of the white chalk, with the characteristic fossils of each. IS Ccl EXAMINATION PAPERS,

3., Describe the mountain limestone, the Nerinoean limestone, and the Woolhope limestone forma­ tions, giving the characteristic fossils of each.

4. What genera of fossils would enable you to deter­ mine the age of {a) lower siluriah, {b) wealden', (c) muschelkalk ?

5. What would be the probable geological age of the rocks containing {a) Baculites, (b) Goniatites, {c) Ampyx, {d) Amplexus, (e) Ctenacanthus, (f) Anoploiherium ?

FOURTH HONOUR EXAMINATION IN LAWS.

LAW.^PAET III.

Mr, Billing.

1. Suppose a plea of tender to be put in with a plea of set-off to the same cause of action, would it be good ? 2. In an action for knowingly keeping a mischievous • animal which injured the plaintiff, state what is put in issue under " not guilty."

3. To support an action of detinue what description of right must the plaintiff have to the possession of the goods ? Give the answer fully. FOURTH HONOUR EXAMINATION, F.T. 1871. Ccli

4. As regards actions for defamation, has a writer in a newspaper greater privilege than another per­ son in commenting on the public acts or writings of others ?

5. What is the difference between the measure of damages in trespass for taking goods and in an action for converting them ?

6. What is the measure of damages in an action for not delivering goods under a contract of sale ?

7. What is the difference between a penalty and liquidated damages ?

8. Mention some cases in which notice of action is required. Under what plea is the want of it usually taken advantage of ?

9. Give some instances where declarations would be admissible as part of the Pes Gestce.

10. What difference is there as regards the affidavits to ground a summons for interrogatories before plea and after plea ?

11. Enumerate the cases in which a jury may give damag-es in the nature of interest.

12. What is the law as to the specific delivery of chattels under the provisions of the Common Law Procedure Statute ? Cdii EXAMINATION PAPERS,

LAW.—PAET IV.

Dr. Dobson.

Cite any cases which you may remember as authorities in support of your answers.

1. Upon what principle does the doctrine of mar­ shalling depend ?

2. Mention the three classes of cases in which the doctrine of marshalling assets is most frequently brought into exercise.

3. Upon what principle is the doctrine of contribution among co-sureties founded ? Does it make any difference whether the sureties are bound by different instruments or are all jointly liable under the same instrument ?

4. Where sureties are bound by different instruments for distinct portions of a debt due from the same principal and the suretyship of each is a separate and distinct transaction, does the right of contri­ bution arise ?

5. Will the Court enforce the performance of a voluntary agreement to constitute a party cestui- que-trust ? Suppose the legal conveyance has been actually made constituting the relation of trustee and cestui-que-trust, but without con­ sideration, will the Court decree specific per­ formance in favour of the volunteer ? FOURTH HONOUR EXAMINATION, F.T. 1871. Ccliii

6. A person who was entitled to stock standing in the names of two trustees gave instructions to his attorney to prepare a settlement of it for the benefit of A, B, and O, and to procure from the trustees a transfer for the purposes of settlement. The settlement was prepared and a power of attorney for the transfer of the stock executed by both trustees; but the intended settlor died without having seen the settlement and before the stock was actually transferred. Give your opinion as to whether or not a trust was con­ stituted for A, B, and C.

7. What is the principle on w-hich the Court grants relief against penalties and forfeitures ? 8. Upon what principle does it depend that a pur­ chaser for valuable consideration, without notice of a prior equitable right, obtaining the legal estate at the time of his purchase, is entitled to priority in equity as w7ell as at law ? 9. A testator directs real estate to be sold and the produce of the sale to be applied for a purpose which either wholly or partially fails. The real estate is sold. To whom does the residue of the money arising from the sale go ? 10. A testator directs land to be sold and the proceeds given to a charity in contravention of the Statute of Mortmain. What is the effect of this in equity as regards—(i.) the charity, (ii.) the heir, (iii.) the next of kin ? 11. Illustrate and explain the doctrine of election. Upon what principle is it founded ? Does the doctrine apply to needs as well as to wills ? Ccliv EXAMINATION PAPERS,

12. A person elects against a will by which property of greater value than his own is left to him. Does he forfeit the whole of the benefit intended for him, or is he only obliged to compensate in value the claimant whom he has disappointed ?

13. Define satisfaction. In what three classes of cases do questions of satisfaction usually arise ?

14. Under what circumstances will a Court of Equity enforce the specific performance of a parol con­ tract within the Statute of Frauds ?

15. Define an express trust, a constructive trust, and an implied trust. How does a constructive trust differ from an implied trust ?

16. " He who seeks equity, must do equity." Explain and illustrate this maxim.

17. What is a donatio mortis causa ? How is it con­ stituted, and how does it differ from a legacy ?

18. What is the nature of the jurisdiction of equity in matters of account ?

19. The personal estate constitutes the primary and natural fund for the payment of debts and lega­ cies. Give the exceptions to this rule.

20. Trace the growth of the present jurisdiction of the Court of Chancery in the case of idiots and lu­ natics. FOURTH HONOUR EXAMINATION, F.T. 1871. Cclv

CONSTITUTIONAL AND LEGAL HISTORY.

Professor Heam.

1. What is the most ancient English oath of alle­ giance and where is it found ? 2. What is the difference between liege homage and simple homage and when was the distinction fully established ?

3. What parts of the Law of Procedure are specially connected with the liberty of the Subject ?

4. What is the doctrine of the Common Law as to religious disabilities, and on what occasion was this doctrine so declared ?

5. State the circumstances of the admission of Jews to the House of Commons.

6. What were Lettres de Cachet, and on what occa­ sion was an attempt made to introduce them into England ?

7. What was Evocation, and on what occasion was an attempt made to introduce it into England ?

8. What remedy (if any) exists against the Governor of a colony for acts of State done by him ?

9. What remedy (if any) exists against the Governor of a colony for personal obligations incurred by him ? Cclvi EXAMINATION PAPERS,

10. What was the old remedy which the practice of granting a new trial superseded, and what was the date of the change ?

11. State the various remedies which the Subject has at Common Law against the Crown and the cases in which they are respectively applicable.

12. State the principles established by the cases Leach v. Money, Wilkes v. Wood, and Entick v. Car- rington, respectively.

ROMAN CIVIL LAW.

Dr. Dobson.

1. What was the effect of a testator bequeathing property which he knew to belong to another ? What was the effect of a bequest to a creditor of his debt ?

2. Titius, Seio debitor! suo ita legavit:—"Do lego Seio denarios decern. Item dono illi quidquid sortis et usurarum nomine mihi debebat" : prae- terea generaliter damnavit ha?redes, fideique eorum commisit, uti darent restituerent unicuique quidquid ei legasset. Postea Seius aliam praaterea pecuniam a Titio mutuatus est. Qusero an hiec quoque pecunia qua? post testamentum factum data esset Seio, legata intelligitur ? Answer this question with reasons. FOURTH HONOUR EXAMINATION, F.T. 1871. Cclvii

3. Define obligatio. Distinguish between natural and civil obligations.

4. How were obligations arising ex contractu created ? Give an example of each class.

5. Explain the meaning of the term obligationes quasi ex contractu.

6. Cum quis absentis negotia gesserit, ultro citroque inter eos nascuntur actiones qua? appellantur negotiorum gestorum. Quas ex nullo contractu proprie nasci manifestum est: quippe ita nascun­ tur ista? actiones si sine mandate, quisque alienis negotiis gerendis se obtulerit; ex qua causa, hi quorum negotia gesta fuerint etiam ignorantes obligantur. Compare the English law on this subject. What must the negotiorum gestor prove under the latter system in order to gain his case ?

7. Explain the meaning of the expression " litem suam facere."

8. What are innominate contracts ? To what class of contracts did they belong, whence their name, and what was the nature of their difference from other contracts ?

9. Distinguish between the systems of actiones legis, formula?, and cognitiones extraordinaria?, and herein shew the respective functions of the judex and magistratus in each stage.

10. Define mandatum and mutuum. Give the formula of an actio mutui. Is mutuum an English bail­ ment ? Cclviii EXAMINATION PAPERS,

11. What is the distinction between contracts (and actions), bonee fidei and strict! juris ? Under what circumstances was the actio in factum prtBScriptis verbis resorted to ?

12. Name the different kinds of societates. In which kind was there unlimited liability ? What de­ scription of societas is the main source of the English law of partnership ?

13. What were the mutual duties of locator and con­ ductor ? Explain the actions Serviana and quasi Serviana.

14. A, under a mistake of fact, pnys money to B which was not due. Was an obligatio created between the parties ? What elements of a contract proper are there in the transaction ? Is money so paid recoverable in England on the ground of an implied contract ?

15. What is agnatio ? Explain the expression agna- tionis jus omnibus modis capitis diminutione perimitur.

16. Explain usucapio, stating its necessary- conditions and the original cause of its institution. Com­ pare it with prascriptio both as to theory and practical effect.

17. Explain pactum, conventio, pollicitatio, contractus; and shew the influence of the definition of obli­ gatio upon the Roman law of contract. FOURTH HONOUR EXAMINATION, F.T. 1871. cdix

GENERAL JURISPRUDENCE.

Professor Hearn.

Explain and comment upon the following passages in Mi\ Austin's work :— 1. "Each of the three terms signifies the same notion, but each denotes a different part of that notion and connotes the residue."—I. 10.

2. " The opinion of the party wliich abhors judge- made law springs from their inadequate con­ ception of the nature of commands."—lb. 24.

3. " Positive moral rules, wliich are laws properly so called, are of three kinds."—lb. 124.

4. " Laws metaphorical though numerous and differ­ ent have the following common and negative nature."—lb. 157. ,

5. " Hence it inevitably follows that the law obtain­ ing between nations is not positive law."—lb. 177.

6. " As meaning monarchical power limited by po­ sitive law, the name limited monarchy involves a contradiction in terms."—lb. 197.

7. " Political or civil liberty is not more worthy of eulogy than political or civil restraint."—lb. 242.

8. " To every legal right there are three separate parties."—lb. 255. Cclx EXAM. PAPERS: FOURTH HON. EX. F.T. 1871.

9. " Most of the names which seem to be names of acts are names of acts coupled with some of their consequences."—IT. 85.

10. " The objection to laws ex post facto is deducible from those general principles with regard to intention and inadvertence which I am en­ deavouring to explain."—lb. 175.

11. " No man has a right of servitude in a thing of which he is the owner."—III. 27.

12. " Every privilege properly so called is a strictly personal privilege."—lb. 93.

By Authority: Jons FEIIHES,Government Printer, Melbourne. J/

Library Digitised Collections

Author/s: University of Melbourne

Title: University of Melbourne Calendar 1871 - 1872

Date: 1871 - 1872

Persistent Link: http://hdl.handle.net/11343/23366

File Description: 25_Appendix

Terms and Conditions: Terms and Conditions: Copyright in works deposited in the University of Melbourne Calendar Collection is retained by the copyright owner. The work may not be altered without permission from the copyright owner. Readers may only, download, print, and save electronic copies of whole works for their own personal non-commercial use. Any use that exceeds these limits requires permission from the copyright owner. Attribution is essential when quoting or paraphrasing from these works.

Terms and Conditions: To request permission to adapt, modify or use the works outside of the limits of these terms and conditions, please complete the permission request form at: http://www.unimelb.edu.au/copyright/information/fastfind/externalrequest.html